Guia do estudante matemática (2017)

Page 1


APRESENTAÇÃO

Um plano para os seus estudos Este GUIA DO ESTUDANTE MATEMÁTICA oferece uma ajuda e tanto para as provas, mas é claro que um único guia não abrange toda a preparação necessária para o Enem e os demais vestibulares. É por isso que o GUIA DO ESTUDANTE tem uma série de publicações que, juntas, fornecem um material completo para um ótimo plano de estudos. O roteiro a seguir é uma sugestão de como você pode tirar melhor proveito de nossos guias, seguindo uma trilha segura para o sucesso nas provas.

1 Decida o que vai prestar

O primeiro passo para todo vestibulando é escolher com clareza a carreira e a universidade onde pretende estudar. Conhecendo o grau de dificuldade do processo seletivo e as matérias que têm peso maior na hora da prova, fica bem mais fácil planejar os seus estudos para obter bons resultados. COMO O GE PODE AJUDAR VOCÊ O GE PROFISSÕES traz todos os cursos superiores existentes no Brasil, explica em detalhes as características de mais de 260 carreiras e ainda indica as instituições que oferecem os cursos de melhor qualidade, de acordo com o ranking de estrelas do GUIA DO ESTUDANTE e com a avaliação oficial do MEC.

2 Revise as matérias-chave

CAPA: 45 JUJUBAS

CALENDÁRIO GE 2016 Veja quando são lançadas as nossas publicações MÊS Janeiro Fevereiro

GE HISTÓRIA

Março

GE ATUALIDADES 1

Para começar os estudos, nada melhor do que revisar os pontos mais importantes das principais matérias presentes no Ensino Médio. Você pode repassar todas as disciplinas ou focar só em algumas delas. Além de rever os conteúdos, é fundamental fazer exercícios para praticar.

Abril

COMO O GE PODE AJUDAR VOCÊ Além do GE MATEMÁTICA, que você já tem em mãos, produzimos um guia para cada matéria do Ensino Médio: GE QUÍMICA, Física, Biologia, História, Geografia, Português e Redação. Todos reúnem os temas que mais caem nas provas, trazem muitas questões de vestibulares para fazer e ainda têm uma linguagem fácil de entender, permitindo que você estude sozinho.

Junho

3 Mantenha-se atualizado

O passo final é reforçar os estudos sobre atualidades, pois as provas exigem alunos cada vez mais antenados com os principais fatos que ocorrem no Brasil e no mundo. Além disso, é preciso conhecer em detalhes o seu processo seletivo – o Enem, por exemplo, é bastante diferente dos demais vestibulares. COMO O GE PODE AJUDAR VOCÊ O GE Enem e o GE Fuvest são dois verdadeiros “manuais de instrução”, que mantêm você atualizado sobre todos os segredos dos dois maiores vestibulares do país. Com duas edições no ano, o GE ATUALIDADES traz fatos do noticiário que podem cair nas próximas provas – e com explicações claras, para quem não tem o costume de ler jornais nem revistas.

PUBLICAÇÃO

Maio

GE GEOGRAFIA GE QUÍMICA GE PORTUGUÊS GE BIOLOGIA GE ENEM GE FUVEST

Julho

GE REDAÇÃO

Agosto

GE ATUALIDADES 2

Setembro

GE MATEMÁTICA GE FÍSICA

Outubro

GE PROFISSÕES

Novembro Dezembro Os guias ficam um ano nas bancas – com exceção do ATUALIDADES, que é semestral. Você pode comprá-los também nas lojas on-line das livrarias Cultura e Saraiva. FALE COM A GENTE: Av. das Nações Unidas, 7221, 18º andar, CEP 05425-902, São Paulo/SP, ou email para: guiadoestudante.abril@atleitor.com.br

GE MATEMÁTICA 2017

3


CARTA AO LEITOR

ANTHONY BEHAR/SIPA

8 EM CADA 10 APROVADOS NA USP USARAM

O valor de conhecer

N

a política e na vida, a ignorância não é uma virtude.” A frase foi dita pelo presidente dos Estados Unidos, Barack Obama, na cerimônia de formatura de uma turma da Universidade Rutgers. Na ocasião, Obama criticava o então candidato à presidência Donald Trump. Mas a ideia se encaixa no dia a dia de qualquer um. Vive melhor quem acompanha de perto as transformações de seu tempo. Para você, vestibulando, o raciocínio é mais válido do que nunca. Para ir bem nas provas de vestibular, é fundamental saber interpretar as notícias. E, para isso, é importante, entre outras coisas, dominar aspectos básicos de diversas ciências, particularmente da matemática. A matemática está oculta em muitas das notícias que você vê na TV, na internet ou nos jornais. Em geofísica, explica como ocorrem os terremotos; em economia, ajuda a interpretar gráficos; em geografia e meio ambiente, é essencial para compreender a relação entre o crescimento da população e o estoque de recursos naturais do planeta. Ou seja, saber essa ciência exata não conta pontos apenas nas provas de matemática, mas também nas de outras áreas, e também na redação. Nós, editores do GUIA DO ESTUDANTE MATEMÁTICA VESTIBULAR + ENEM, entendemos que quem enxerga o lado prático da matemática aprende mais rápido. Por isso pinçamos para esta edição alguns dos grandes assuntos da atualidade como ponto de partida para trabalhar com os conceitos matemáticos básicos. O conteúdo foi preparado pelo professor Fabio Marson Ferreira, do Colégio Móbile, de São Paulo. A primeira decisão acertada que você pode tomar agora é mergulhar neste guia e se preparar para um futuro de sucesso. A redação

4 GE MATEMÁTICA 2017

SELO DE QUALIDADE GUIA DO ESTUDANTE PARA FORMANDOS O presidente dos Estados Unidos, Barack Obama, na cerimônia de formatura da Universidade Rutgers

O selo de qualidade acima é resultado de uma pesquisa realizada com 351 estudantes aprovados em três dos principais cursos da Universidade de São Paulo no vestibular 2015. São eles: � DIREITO, DA FACULDADE DO LARGO SÃO FRANCISCO; � ENGENHARIA, DA ESCOLA POLITÉCNICA; e � MEDICINA, DA FACULDADE DE MEDICINA DA USP 8 em cada 10 entrevistados na pesquisa usaram algum conteúdo do GUIA DO ESTUDANTE durante sua preparação para o vestibular Entre os que utilizaram versões impressas do GUIA DO ESTUDANTE: 88% disseram que os guias ajudaram na preparação. 97% recomendaram os guias para outros estudantes.

TESTADO E APROVADO! A pesquisa quantitativa por meio de entrevista pessoal foi realizada nos dias 11 e 12 de fevereiro de 2015, nos campi de matrícula dos cursos de Direito, Medicina e Engenharia da Universidade de São Paulo (USP).

� Universo total de estudantes aprovados nesses cursos: 1.725 alunos. � Amostra utilizada na pesquisa: 351 entrevistados. � Margem de erro amostral: 4,7 pontos percentuais.


SUMÁRIO

Sumário

POTÊNCIA E LOGARITMO

Matemática VESTIBULAR + ENEM 2017

78 80 82 86 92

Abalos semelhantes, prejuízos diferentes Países mais pobres, com infraestrutura precária, sofrem mais com terremotos Potenciação As propriedades da multiplicação de um número por ele mesmo repetidas vezes Funções e equações exponenciais As expressões nas quais a variável é o expoente de um número e seus gráficos Funções e equações logarítmicas Como encontrar o expoente de uma potência Como cai na prova + Resumo Questões comentadas e síntese do capítulo

TRIGONOMETRIA

94

ÍNDICE REMISSIVO

8

Onde você encontra nesta edição os principais conceitos FÓRMULAS E CONCEITOS

10

Os danos colaterais do aeromodelismo bélico Os drones usados contra terroristas fazem muitas vítimas entre civis inocentes 96 Triângulos e a circunferência trigonométrica Relações entre ângulos nas figuras de três lados 100 Funções trigonométricas As expressões que definem seno, cosseno e tangente e seus gráficos 102 Como cai na prova + Resumo Questões comentadas e síntese do capítulo

As expressões matemáticas e os conceitos mais importantes

PROGRESSÕES NÚMEROS E OPERAÇÕES

14 16 19 24 28

Devem e pagam quando puderem A dívida dos estados com a União Números e conjuntos Os conceitos básicos para qualquer cálculo Razão e proporção Relações entre grandezas Juros Os cálculos básicos que envolvem o custo do dinheiro Como cai na prova + Resumo Questões comentadas e síntese do capítulo

104 Disparidade econômica evidente na demografia As consequências da explosão demográfica na África e redução da população na Europa 106 PA As progressões que evoluem pela soma de uma razão 108 PG As sequências que crescem ou decrescem de maneira exponencial 110 Como cai na prova + Resumo Questões comentadas e síntese do capítulo

COMBINATÓRIA E PROBABILIDADE GEOMETRIA

30 32 36 38 40 46 52 58

Júpiter recebe um bisbilhoteiro A sonda Juno, da Nasa, chega ao planeta gigante Ponto, reta e plano Os elementos essenciais das figuras geométricas lineares Plano cartesiano O quadriculado que permite localizar qualquer ponto Gráficos As diversas maneiras de representar a variação de grandezas Polígonos Medidas de lado e área de quadrados, retângulos, trapézios e triângulos Cônicas As curvas abertas e fechadas que não têm arestas e suas equações Sólidos O volume de prismas, cilindros, cones e pirâmides Como cai na prova + Resumo Questões comentadas e síntese do capítulo

ÁLGEBRA

60 62 68 70 76

A juventude e o desemprego Os jovens são os mais afetados na hora de buscar trabalho Função e equação de 1º grau A expressão que define uma reta Posições relativas de retas As funções que descrevem retas perpendiculares, concorrentes ou paralelas Função e equação de 2º grau As parábolas e suas expressões matemáticas Como cai na prova + Resumo Questões comentadas e síntese do capítulo

6 GE MATEMÁTICA 2017

112 Contra toda a probabilidade A decisão do Reino Unido de deixar a União Europeia 114 Combinatória As diferentes maneiras de arranjar e combinar elementos de um conjunto 117 Probabilidade Como calcular as chances de ocorrer um evento 120 Como cai na prova + Resumo Questões comentadas e síntese do capítulo

MATRIZES

122 Sai a TV analógica, entra a digital A partir de 2018 todas as cidades receberão apenas sinais digitais 124 Conceitos e propriedades Como funcionam as matrizes 126 Determinantes Números que facilitam cálculos em diversas áreas 128 Como cai na prova + Resumo Questões comentadas e síntese do capítulo RAIO-X

130 As características dos enunciados que costumam cair nas provas do Enem e dos principais vestibulares SIMULADO

132 46 questões e suas resoluções, passo a passo


ÍNDICE REMISSIVO

Conteúdo matemático Em ordem alfabética, os termos que remetem aos diversos conceitos abordados nesta edição

da hipérbole.............................................. 51 da parábola ............................................... 71 de 1º grau...................................................65 de 2º grau............................................. 71, 72 da reta .........................................................65 sistemas de ................................................66 Escala de redução ............................................... 20 Richter .......................................................89 Eventos......................................................... 118

F

P Parábola ..........................................50, 70 a 75 Permutação ........................................115 a 116 Plano .........................................................32, 33 cartesiano ...........................................36, 37 Poliedros ........................................................53 Polígonos ..............................................40 a 45 Inscritos e circunscritos ...................... 48 Potenciação .........................................80 a 85 Ponto...................................................... 32 e 45 Porcentagem .......................................... 22, 23 Princípio de Cavalieri .................................55 Prismas.................................................... 53, 54 Probabilidade.....................................117 a 119 Proporção ................................................19, 20 Progressão aritmética (PA) ............. 106, 107 Progressão geométrica (PG)........... 108, 109

Ângulos .......................................33, 34, 98, 99 Área ........................................................ 41 a 44 do círculo ................................................. 46 de polígonos .................................... 41 a 44 de sólidos ......................................... 53 a 55 Arranjos ....................................................... 116 Árvore das possibilidades ................. 114, 115

Fatorial ..........................................................115 Fórmula de Bhaskara ..................................72 Funções análise de sinal .........................................65 conceitos ............................................. 62, 63 domínio ......................................................67 exponenciais .................................... 82 a 85 logarítmicas ........................................90, 91 de 1º grau .......................................... 63 a 65 de 2º grau.......................................... 70 a 75 trigonométricas ..............................100, 101

B

G

Bhaskara, fórmula de ..................................72

Gráficos ................................................. 37 a 39

C

H

Capital ........................................................... 24 Cavalieri, princípio de ................................55 Cilindros ........................................................55 Círculo........................................................... 46 Circunferência.....................................46 a 48 equação da .................................................47 inscrita e circunscrita ............................ 48 trigonométrica................................. 97 e 98 Combinação ................................................ 116 Cone ................................................................56 Conjuntos ...................................................... 17 numéricos .................................................. 18 Cosseno ....................................................97, 99 lei dos cossenos........................................99

Hipérbole.................................................50, 51

Razão ..............................................................22 Regra de três ................................................. 21 Reta .........................................................33 a 35 coeficiente angular ................................ 64 coeficiente linear ................................... 64 equação da ................................................65 posição relativa ..................33 a 35, 68, 69

I

S

Inequações ....................................................66

Sistemas de equação....................................66 Seno ....................................................... 97 a 99 lei dos senos .............................................99 Sólidos geométricos ........................... 52 a 57

A

D Determinantes ....................................126, 127

E Elipse ......................................................49 a 51 excentricidade ........................................ 50 Equações da circunferência.....................................47

8 GE MATEMÁTICA 2017

J Juros ...................................................... 24 a 27

L Logaritmo ..............................................86 a 91

M Matrizes ...............................................124, 125 Média ............................................................ 118 Mediana ....................................................... 118 Moda ............................................................. 118 Montante ...................................................... 24

N Notação científica ........................................ 81

Q Quadrantes ........................................ 37, 97, 98

R

T Tangente ........................................... 97, 98, 99 Teorema de Pitágoras .............................. 44, 45, 98 de Tales....................................................35 Trapézio ........................................................ 42 Triângulos ............................... 42 a 45, 96, 97 na circunferência trigonométrica ...97, 98 retângulos ................................. 44, 45, 98 semelhança de................................. 96, 97

V Volume .................................................... 56, 57 equivalência de ......................................56


FÓRMULAS E CONCEITOS

Para não esquecer

Equação: Se o centro estiver nas coordenadas C (0, 0): xQ2 + yQ2 = r2

Uma lista de conceitos e fórmulas desta edição

Se o centro não coincidir com (0, 0): (xQ – xC)2 + (yQ – yC)2 = r2

ANÁLISE COMBINATÓRIA

ELIPSE

LOGARITMOS

Permutação simples: P nS = n!

Equações:

Logaritmo do produto: logb (a . c) = logb a + logbc

2 x2 y + = 1 sempre com a > b a2 b2

Permutação com repetição: a, b, c,...

Pn

=

n! a! b! c!...

Q x – m V2 Q y – n V2 + = 1, sempre com a 2 b a2 b2

Arranjo simples: A n, p =

n! (n – p) !

Combinação simples: C n,p =

Compostos: Mn= C . (1 + i)n

Logaritmo do quociente: a log b ( c ) = log b a – log b c

Logaritmo de potência: logb(an) = n . logba e logb(bn) = n

FUNÇÃO DE 1º GRAU

Mudança de base do logaritmo:

f(x) = y = a . x + b, em que

A n, p n! = p! (n–p) !p!

Simples: J = C . i . n

1 log b ( a ) = – log b a

c Excentricidade: e = a

Arranjo com repetição: A rn, p = n p

JUROS

log c a =

• a é o coeficiente angular da reta:

ÁREA DE FIGURAS PLANAS

3y (y A – y B ) (y B – y A ) a = 3x = = (x A – x B ) (x B – x A )

MATRIZES

Retângulo: A = base . altura Quadrado: A = lado . lado = lado

2

Losango:

log b a log b c

• b é o coeficiente linear da reta é o valor de y quando x = 0

Diagonais: diagonal principal

diagonal secundária

A = A11 A21 A12 A22

diagonal m aior . diagonal m enor A= 2

Raiz da função é o valor de y no ponto em que a reta cruza o eixo x:

Trapézio:

b y = a. x + b & 0 = a.x + b & x = – a

Matriz identidade:

FUNÇÃO DE 2º GRAU

1 0 0 I= 0 1 0 0 0 1

A=

(base maior + base m enor) . h 2

Paralelogramo: A = base . altura Triângulo: A=

base . altura 2

Círculo: A = π . r 2

Forma geral: y = a . x2 + b . x + c Forma fatorada: y = a . (x – x1) . (x – x2) Forma canônica: y = a . (x – xV) + yV 2

Fórmula de Bhaskara x=

–b !

b 2 – 4 . a .c 2 .a

CIRCUNFERÊNCIA

Coordenadas do vértice da parábola:

Comprimento: P = 2 . π. r

xv= – b 2 .a

10 GE MATEMÁTICA 2017

D yv = – 4 . a

Soma de matrizes: a11 + b11 Aij + Bij = a21 + b21 ... aij + bij

a12 + b12 a22 + b22 ... aij + bij

a13 + b13 a23 + b23 ... aij + bij

Multiplicação por um número: k . Aij =

k . a11 k . a21 ... k . aij

k . a12 k . a22 ... k . aij

k . a13 k . a23 ... k . aij


Multiplicação de matrizes: Os elementos da matriz P produto de A1 . A2 são obtidos pela multiplicação dos elementos de cada linha de A1 pelos elementos correspondentes de cada coluna de A2. Depois, os resultados são somados.

PROBABILIDADE Eventos independentes: P (A + B) = P (A) . P (B)

União de dois eventos: P (A , B) = P (A) + P (B) – P (A + B)

PA E PG Termo geral de uma PA: an = a1 + (n – 1) . r, para n ≥ 2 Soma dos termos de uma PA: Sn =

n . (a 1 + a n) 2

Termo geral de uma PG: an = a1 . qn - 1, n ≥ 2 Soma dos termos de uma PG finita: Sn =

a 1 . (q n –1) para q ! 1 q– 1

Soma dos termos de uma PG infinita: a1 lim S n = – q – 1 n"3

Média aritmética: É a soma de todos os valores dos elementos de um conjunto dividida pelo número total de elementos do conjunto. Média ponderada: Leva em consideração o peso de cada elemento do conjunto. Mediana: É a medida central de uma lista de medidas colocadas em ordem crescente, ou decrescente. Moda: É o valor que mais aparece em uma série de dados.

TRIÂNGULOS Teorema de Pitágoras: c2 = a2 + b2 Razões trigonométricas:

POTENCIAÇÃO

sen a =

cateto oposto a a hipotenusa

Notação científica: n = a . 10x, em que 1 ≤ a < 10

cos a =

cateto adjacente a a hipotenusa

Propriedades: • am . an = am + n

tg a =

• am : an = am – n

Lei dos senos:

• (am)n = am . n

c a b sen a = sen b = sen c

1 b a–b = S a X

Lei dos cossenos:

•a

m n

=

n

a 2 = b 2 + c 2 – 2 .b . c. cos a

am

VOLUME DE SÓLIDOS

• (m.n) b = m b . n b b

m m S n X = b n b

cateto oposto a a cateto adjacente a a

• a0 = 1, desde que a ≠ 0

4 Esfera: v = 3 . r . r 3 Prism a: v = A base . h

1 Pirâm ide: v = 3 . A base . h Cilindro: v = r . r 2 .h 1 Cone: v = 3 . r . r 2 .h


1

NÚMEROS E OPERAÇÕES CONTEÚDO DESTE CAPÍTULO

Conjuntos numéricos .....................................................................................16 Razão e proporção ...........................................................................................19 Juros .....................................................................................................................24 Como cai na prova + Resumo .......................................................................28

Devem, mas pagam quando puderem

Aumento de gastos e redução de receitas arruínam as finanças dos estados e levam governadores a negociar com o governo federal a dívida com a União

O

s estados brasileiros e o Distrito Federal estão com as contas no vermelho. Algumas mais, outras menos, as 27 unidades da federação chegaram a 2015 com a contabilidade negativa, grande parte delas incapaz de pagar fornecedores e quitar dívidas. E, no geral, o maior credor dos estados é a União. Em maio de 2016, São Paulo devia ao governo federal mais de 220 bilhões de reais, Minas Gerais, 80 bilhões, Rio de Janeiro e Rio Grande do Sul, 57 e 52 bilhões, respectivamente. Sem dinheiro, os governos paralisam obras, atrasam o pagamento dos salários de servidores e não conseguem repor materiais básicos em hospitais e postos de saúde. Em dezembro de 2015, o Rio de Janeiro decretou estado de emergência na saúde pública e durante cinco meses amargou greves de professores. Diversos fatores contribuem para o saldo negativo nas contas dos estados. Redução em tarifas públicas derruba a arrecadação do Imposto sobre Circulação de Mercadorias e Serviços (ICMS), importante fonte de recursos estaduais. A valorização do salário mínimo, que é corrigido acima da inflação desde 2003, engorda a folha de pagamento, que também tem sido inchada nos últimos anos por novas contratações e reajustes salariais concedidos pelos governos estaduais, em contrapartida ao aumento das receitas.

14 GE MATEMÁTICA 2017

A atual dívida com a União foi contraída entre 1997 e 1999, quando, para salvar os estados, o governo federal assumiu o pagamento aos credores, concedendo aos governadores prazos e índices de correção mais favoráveis. No entanto, as regras e a situação econômica do país mudaram. Hoje, os valores das parcelas são atualizados segundo a variação da taxa básica de juro (Selic) e corrigidos pelo mecanismo de juros compostos – o que aumenta o valor da dívida de maneira exponencial. Os governadores pedem a troca dos juros compostos por juros simples. Mas o governo federal não cede. A substituição faria com que o Tesouro Nacional deixasse de receber 313 bilhões de reais. Enfim, as partes chegaram a um acordo em junho de 2016: o prazo para quitação foi expandido, e as parcelas serão pagas com abatimentos gradativamente menores: na primeira, o desconto será de 94,5%, na segunda, 89%, e assim por diante, retrocedendo FORA DAS ESCOLAS a cada mês 5,5 pontos Professores da rede estadual percentuais. de ensino do Rio de Janeiro Porcentagens, juros permaneceram em greve simples e compostos entre março e julho de 2016. são alguns dos temas Os docentes pedem aumento que você revê neste salarial e reclamam do capítulo. atraso nos pagamentos


CELSO PUPO/FOTOARENA

GE MATEMÁTICA 2017

15


ISTOCK

NÚMEROS E OPERAÇÕES NÚMEROS E CONJUNTOS

Conceitos fundamentais As ferramentas básicas usadas em todas as operações, da simples contagem aos cálculos mais complexos

16 GE MATEMÁTICA 2017

A

ssim como qualquer campo do conhecimento – física, química, história ou geografia –, a matemática tem também sua própria linguagem, composta de símbolos e conceitos. O primeiro e mais importante deles são os números. Sem eles, não seria possível contar, medir, ordenar e classificar. Não se sabe ao certo que povo desenvolveu a ideia abstrata de número. Mas os historiadores têm como certo que o conceito surgiu da necessidade de contar objetos e seguir um calendário. O sistema de contagem deve ter se iniciado com o uso dos dedos, há milhares de anos, e de pedras, uma para cada unidade. Depois vieram pequenas placas de argila –, cada uma delas também representando uma unidade. Os incas criaram os quipus, um sistema de cordas e barbantes com nós.

Os numerais, ou algarismos – os símbolos gráficos que representam os números –, teriam aparecido bem mais tarde, com a escrita. E a uma certa altura da história, o comércio criou a necessidade de se registrar e comunicar a contagem de mercadorias e seus valores. Antropólogos têm registro de ossos, pedras e pedaços de madeira de pelo menos 5 mil anos com marcas escavadas com o que eles supõem tenham sido os primeiros numerais. Atribui-se aos egípcios a invenção dos primeiros símbolos numéricos mais formais, na forma de hieróglifos. Os romanos criaram os algarismos romanos: I, V, X, L, C, D e M. Hoje a matemática faz uso, no mundo todo, dos algarismos indoarábicos: 1, 2, 3... 10, 11, 12... Acredita-se que esses algarismos tenham sido criados na Índia, também há milhares de anos.


SAIBA MAIS ALGARISMOS ROMANOS

O sistema de notação por algarismos romanos – que empregamos hoje apenas para classificar e ordenar elementos, como nos capítulos de um livro – dispensa o número zero. Nele, as letras I, V, X, L, C, D e M simbolizam quantidades básicas: 1, 5, 10, 50, 100, 500 e 1000, respectivamente. A posição e o número de vezes em cada um desses símbolos é repetido definem dezenas, centenas e milhares.

ALGARISMOS ARÁBICOS 1, 2, 3, 4

I, II, III, IV

5, 6, 7, 8

V, VI, VII, VIII

9, 10, 11... 19, 20

IX, X, XI... XIX, XX

50

L

54

LIV

100

C

111

CXI

500

D

591

DXCI

1000

M

1008

MVIII

VIAGEM NO TEMPO Milênios se passaram desde a criação dos algarismos indoarábicos, na Índia. Mas até hoje, por mais avançada que seja a tecnologia, são estes os algarismos que usamos no dia a dia

Os números utilizados em contagens são chamados números concretos – cada número representa certa quantidade de “coisas” reais. O zero, que representa a ausência, o nada ou o vazio, não é um número concreto, mas um numeral de posição. Dependendo do local em que o zero é colocado, os numerais anteriores ou posteriores assumem diferentes valores. Por exemplo, no sistema decimal, que tem como base o 10, o numeral 1 representa uma unidade. Mas, seguido de um zero (10), são dez unidades; e 0,1 representa um décimo de uma unidade.

Conjuntos

A teoria dos conjuntos é uma área da matemática que você não precisa conhecer com profundidade para o Enem. Mas seus conceitos são fundamentais para compreender enunciados e, assim, chegar

à resposta correta das questões. Conjunto, você sabe: é um grupo de elementos: • o conjunto formado pelos números nas faces de um dado é D = {1, 2, 3, 4, 5, 6}; • já o conjunto dos números pares de um dado é P = {2, 4, 6}; • e o conjunto dos ímpares é I = {1, 3, 5}. Os conjuntos também podem ser representados pelo diagrama de Venn. Os diagramas para os conjuntos D, P e I, acima, são: D I

1 3 5

2 P 4 6

ALGARISMOS ROMANOS

Observe o diagrama e repare: • O número 6 pertence (∈) aos conjuntos D e P. Então, 6 ∈ D e 6 ∈ P; • Mas o número 3 não pertence (∉) a P. Então, 3 ∉ P; • Todos os elementos de P e de I estão contidos (⊂) em D. Então, P ⊂ D e I ⊂ D. • No sentido inverso, D contém (⊃) P e I. Então, D ⊃ P e D ⊃ I. Podemos fazer diversas operações entre conjuntos: • A união (∪) é a combinação dos elementos dos conjuntos. No nosso exemplo, I ∪ P = {1, 2, 3, 4, 5, 6} = D. • A intersecção (∩) é o conjunto formado por elementos comuns aos conjuntos. No caso do exemGE MATEMÁTICA 2017

17


NÚMEROS E OPERAÇÕES NÚMEROS E CONJUNTOS

plo dos números pares e ímpares de um dado, o conjunto da intersecção entre I e P é um conjunto vazio (nenhum número é ímpar e par ao mesmo tempo): I ∩ P = Ø

Conjuntos numéricos

Os números também podem ser agrupados em conjuntos: • O conjunto dos números naturais (N) é N = {0, 1, 2, 3...}. Repare que este conjunto é infinito.

ATENÇÃO

NA PRÁTICA

O conjunto C resultante da união de A e B contém os elementos que se encontram em A ou em B. Já o conjunto D resultante da intersecção de A e B contém os elementos que se encontram ao mesmo tempo em A e em B.

• O conjunto dos números irracionais (I) é o dos números que não podem ser obtidos da razão entre dois números inteiros. O π é um número irracional. A raiz de alguns números também é um número irracional – por exemplo, √2 e √3. A união entre todos os conjuntos de números acima forma o conjunto dos números reais (R). No diagrama de Venn, essa união é representada assim:

A resposta é o conjunto S = {x ∈ R | x <

I

R

Q

A teoria dos conjuntos é frequentemente utilizada em álgebra, principalmente em inequações, e em probabilidade (veja os capítulos 3 e 7).

18 GE MATEMÁTICA 2017

}.

2. Quais os valores de x (x ∈ Z*) que atendem às duas condições abaixo?

(I) x – 3 ≤ 1 <3 (II) – 1 <

Se x ∈ Z*, então x é um número inteiro, diferente de zero. Essa condição restringe os valores do conjuntos solução SI e SII. Resolvendo as inequações (I) e (II): • Para a condição I:

TOME NOTA SÍMBOLOS DA TEORIA DOS CONJUNTOS SÍMBOLO { }

SIGNIFICADO Conjunto

x–3≤1 → x≤1+3→ x≤ 4 SI = {... -4, -3, -2, -1, 1, 2, 3, 4}

• Para a condição II: tervalo {-1, 1, 2}

Pertence, não pertence

Então,

Tal que

–1 <

Contém

Está contido

Intersecção de conjuntos

União de conjuntos

Conjunto vazio

N

Conjunto dos números naturais Conjunto dos números inteiros

∈, ∉

Z N Z

1. Encontre os valores de x que tornam verdadeira a expressão 2x – 5 < 0. Expressões matemáticas como esta, em que em vez do sinal de igual (=) temos sinais de maior (>), menor (<), menor-igual (≤) ou maior-igual (≥), são chamadas inequações. E são resolvidas como equações (veja no capítulo 3).

2x – 5 < 0 2x < 0 + 5 → x <

• O conjunto dos números inteiros (Z) reúne os números naturais e seus opostos: Z = {..., -3, -2, -1, 0, 1, 2, 3, ...}. Este também é um conjunto infinito. • O conjunto dos números racionais (Q) é a união dos números inteiros e as frações resultantes da divisão entre quaisquer deles: Q = { | a ∈ Z e b ∈ Z*}. Traduzindo: o conjunto Q é formado pelos números obtidos pela divisão de a por b, tal que ( � ) a pertence ao conjunto dos números inteiros e b pertence ao conjunto dos inteiros com exceção do zero (Z*). O número b não pode assumir o valor zero porque a divisão por zero não é definida.

CONJUNTOS NUMÉRICOS

Q

R

Conjunto dos números racionais Conjunto dos números irracionais Conjunto dos números reais

*

Exceto o zero

I

+/–

São válidos valores positivos e negativos

e

deve estar no in-

→ – 2 < x;

< 3 → x < 3 . 2 → x < 6.

SII = {-1, 1, 2, 3, 4, 5} O valor que atende a ambas as condições é a intersecção dos conjuntos SI e SII – ou seja, o conjunto cujos elementos pertencem aos dois conjuntos, ao mesmo tempo:

I ∩ II = {-1, 1, 2, 3, 4}

Pelo diagrama de Venn: I -3 -4 ...

-2 -1 1 2 3 4

II

I ∩ II

5 6


4FR/iSTOCK

NÚMEROS E OPERAÇÕES RAZÃO E PROPORÇÃO

Valores que conversam entre si A proporção entre grandezas é usada tanto na confecção de mapas quanto no cálculo da concentração de gases do efeito estufa na atmosfera

U

m dos principais domínios da matemática é usar a lógica para estabelecer relações entre valores e grandezas. Relações entre grandezas são aquelas em que o valor de uma grandeza varia, dependendo do valor de outra. Fazemos relações entre grandezas em diversas atividades do cotidiano, como a energia elétrica consumida a cada dia e a conta que chega no final do mês, ou a proporção entre os ingredientes de uma receita. A principal razão entre grandezas é aquela que envolve o conceito de proporção, quando uma grandeza cresce ou decresce proporcionalmente a outra: quanto mais tempo você passa no banho, maior é a quantidade de água gasta. E se uma barra de chocolate for dividida entre amigos, quanto maior o número de amigos, menor será o pedaço que caberá a cada um.

ORDEM NATURAL Quando brota, um ramo de samambaia cresce numa curva que segue a chamada proporção divina

GE MATEMÁTICA 2017

19


REPRODUÇÃO

NÚMEROS E OPERAÇÕES RAZÃO E PROPORÇÃO

Diretamente proporcionais

Algumas grandezas mantêm uma relação diretamente proporcional. Isso ocorre quando uma grandeza cresce e a outra também cresce. No banho, o volume de água consumida cresce em proporção direta ao tempo em que o chuveiro permanece ligado. Veja: Um chuveiro libera 12 litros de água por minuto. Quantos litros uma pessoa gasta num banho de 5 minutos? Podemos construir uma tabela com valores da quantidade de água gasta em função do tempo de duração de um banho: Tempo (min) Volume de água (L)

1

2

3

4

5

12

24

36

48

60

Repare: quanto mais tempo se passa no banho, mais água se consome. E esse consumo aumenta de maneira proporcional: para 1 minuto, 12 L, para 2 minutos, 2 . 12 L = 24 L, e assim por diante. Em 5 minutos, o consumo é de 5 . 12 L = 60 L. Em resumo, se dobrarmos o tempo de banho, a quantidade de água consumida também dobra; se o tempo for triplicado, o gasto de água também é triplicado.

ARTE SOB MEDIDA As figuras que Michelangelo desenhou e pintou na Criação de Adão, no teto da Capela Sistina, seguem um ideal de proporções do corpo humano

Inversamente proporcionais

Duas grandezas são inversamente proporcionais quando uma cresce e a outra cai, sempre uma em proporção à outra. Veja o exemplo: Todas as provas em sua escola valem 100 pontos. Mas as provas podem ter diferentes números de questões. Assim, cada questão terá um valor

NA PRÁTICA

RORAIMA AMAPÁ

A PROPORÇÃO NAS ESCALAS

Uma das principais aplicações práticas da noção de proporção é a confecção de mapas. Todo mapa representa uma realidade reduzida. E essa redução obedece à regra de proporção, nas medidas lineares (distâncias). Veja o mapa ao lado: Repare na indicação da escala, no canto inferior direito. Cada trecho do tamanho do segmento ali representado vale 459 km. A proporção se mantém, também, na área, só que elevada ao quadrado. Um quadrado de 459 km de lado tem área de 459 . 459 = 210 681 km2. Se dobrarmos o tamanho dos lados do quadrado para 918 km, teremos uma área quatro vezes maior: A = 918 . 918 = 842 724 km2.

AMAZONAS

PARÁ

CEARÁ RIO GRANDE DO NORTE PARAÍBA PERNAMBUCO ALAGOAS SERGIPE

MARANHÃO PIAUÍ

ACRE

TOCANTINS

RONDÔNIA

BAHIA

MATO GROSSO GOIÁS MINAS GERAIS

MATO GROSSO DO SUL SÃO PAULO

ESPÍRITO SANTO RIO DE JANEIRO

PARANÁ SANTA CATARINA RIO GRANDE DO SUL ESCALA

0

20 GE MATEMÁTICA 2017

459 KM


diferente, dependendo da prova. Quanto maior o número de questões, menor o valor de cada questão. Para 100 questões, o valor de cada uma é de 1 ponto. Já numa prova de 50 questões, cada uma deve valer 2 pontos, e assim por diante. Numa tabela, temos: Número de questões 1 Valor de cada questão 100

2

4

5

10

50

25

20

10

NA PRÁTICA REGRA DE TRÊS

Seu chuveiro deixa cair 12 L de água por minuto. Quanto você economizará de água se reduzir em 30 segundos o tempo do banho? A regra de três:

1 min – 12 L 30 seg – x L

Repare que, à medida que a quantidade de questões aumenta, o valor de cada uma diminui de maneira proporcional. Quando uma das grandezas dobra, a outra cai pela metade; quando uma cai para 1/4, a outra é quadruplicada.

Antes de resolver a regra de três, vamos uniformizar as unidades minuto e segundo. Precisamos adotar uma única. Sabemos que 1 minuto tem 60 segundos, então 30 segundos valem 0,5 minuto. Montando de novo a regrinha de três:

Regra de três

1 min – 12 L 0,5 min – x L

Qualquer relação de proporcionalidade direta entre grandezas pode ser encontrada pela regra de três. Para isso, basta conhecer um valor e a relação entre dois outros valores (a e b). Veja: a – b x – y Lemos: a está para b assim como x está para y. Para encontrar a proporção entre esses valores, multiplicamos em cruz: x.b=a.y Se você conhece a, b e x, descobre o valor de y:

Fazendo a multiplicação em cruz, obtemos:

1 min – 12 L 0,5 min – x L 1 . x = 0,5 . 12 → x = 6 L A cada 30 segundos de redução do tempo de banho, são economizados 6 L de água. Da mesma forma, você pode descobrir pela regra de três quantos minutos dura um banho em que são consumidos 40 litros de água. Novamente multiplicando em cruz:

1 min – 12 L x min – 40 L 12 . x = 40 . 1 → x =

A regra de três também funciona para grandezas inversamente proporcionais. Com uma diferença importante: neste caso, não multiplicamos em cruz, mas linha a linha. No exemplo das provas acima, se para 100 questões cada uma vale 1 ponto, quanto valerá cada questão se a prova for composta por apenas 40 questões? Montando a regra de três: Para 100 questões cada uma vale 1 ponto Para 40 questões cada uma vale x pontos Assim, 1 . 100 = 40 . x x = 100 : 40 = 2,5 pontos

→ x=

min

Transformando minuto em segundo, ficamos com

x=

. 60 seg x =

600 3

x = 200 seg ≅ 3 min 20 seg Uma regra de três pode ser construída a partir de qualquer par de valores relacionados. No caso do chuveiro, chegaríamos ao mesmo tempo de 3 min 20 seg se partíssemos do consumo, por exemplo, em 2 minutos. Veja:

2 min – 24 L x min – 40 L x . 24 = 2 . 40 =

=

min ≅ 3 min 20 seg

Este é o valor de cada questão numa prova com 40 questões. GE MATEMÁTICA 2017

21


NÚMEROS E OPERAÇÕES RAZÃO E PROPORÇÃO

Razão

Em alguns casos, a proporção entre duas grandezas é expressa como razão – a divisão de dois números, a por b. Nesse caso, a razão pode receber um nome especial. É o caso de porcentagem, densidade ou partes por milhão (abreviadamente, ppm).

NA PRÁTICA DENSIDADE

Densidade

Densidade é uma grandeza física – o valor obtido da divisão da massa pelo volume de um material. A densidade de uma substância ou mistura é dada pela razão d = m/V, em que m é a massa e V, o volume. A unidade de medida para densidade pode ser g/cm3, g/L ou kg/L. A densidade de qualquer substância é medida em laboratórios e utilizada como forma de avaliar o nível de pureza do material. Por exemplo, quando técnicos da ANP (Agência Nacional do Petróleo) fazem fiscalização nos distribuidores ou postos de combustível, eles medem a densidade de amostras da gasolina ou do etanol dos tanques e das bombas. Se tiver havido acréscimo de água ou outra substância qualquer, a densidade se altera – o que compromete a qualidade do combustível.

Porcentagem

A porcentagem também pode ser calculada por regra de três. Esse tipo de cálculo aparece quando se deseja comparar uma parte com o todo. É fácil entender. Veja: • Você tem um inteiro – digamos uma barra de chocolate. • Se dividimos essa barra em cem pedaços menores, a barra inteira representa todas as 100 partes – ou seja, a razão 100/100; • Uma única parte representa 1 parte sobre 100 – ou seja 1%; 2 partes, 2/100 = 2%. E assim por diante. Daí a palavra “por cento”.

O QUE ISSO TEM A VER COM A FÍSICA A densidade de um material depende de seu estado físico, da temperatura e da pressão a que ele está submetido. Mas não depende da quantidade ou da massa. Ou seja, se 1 kg de determinada substância ocupa um volume de 2 L, então, no mesmo estado físico e nas mesmas condições de temperatura e pressão, 2 kg ocuparão 4 L.

Quando a quantidade de soluto é muito menor que o volume total da solução, ou da mistura, em vez de porcentagem costuma-se usar a unidade partes por milhão (ppm). Nas questões relacionadas ao aquecimento global, a medida de concentração dos gases do efeito estufa na atmosfera é dada nessa unidade (veja o Saiu na imprensa na pág. ao lado).

22 GE MATEMÁTICA 2017

A primeira coisa a fazer é uniformizar as unidades. Você sabe que 1 L = 1 000 ml. Então, temos: 0,8 g – 1 mL x g – 200 000 mL Esta é uma relação diretamente proporcional. Multiplicando em cruz, temos: x . 1 = 0,8 . 200 000 → x = 200 000 . 0,8 → x = 160 000 g Transformando g em kg e mL em L, novamente, temos que 200 L de etanol têm massa de 160 kg.

NA PRÁTICA PORCENTAGEM

Uma caixa d’água com capacidade 2 000 litros contém 260 litros de água. Qual a porcentagem do volume da caixa ocupado por essa água? O “inteiro” (100%) é a capacidade total da caixa: 2 000 L. Queremos descobrir a quantos por cento correspondem os 260 L de água que ela contém. Pela regra de três, temos

Concentração

A concentração de uma solução é uma grandeza química que mede a proporção entre a quantidade de soluto e a quantidade total de solução, em massa (mg/kg) ou em volume (cm3/m3 ou L/106L).

Sabendo que a densidade do etanol é de 0,8 g/mL, qual a massa de 200 litros do combustível? A densidade – a razão entre a massa e o volume – permanece constante se a medida for feita à mesma pressão e temperatura, não importa se trabalhamos com 1 mL ou 1 000 L de etanol. Por outro lado, a relação entre volume e massa é diretamente proporcional. Então, podemos montar a regra de três: 0,8 g – 1 mL x g – 200 L

O QUE ISSO TEM A VER COM A QUÍMICA A concentração é tema do estudo de misturas e soluções. A concentração pode ser dada em termos de massa, de volume e, também, mol (número de átomos, moléculas ou íons).

2 000 L – 100% 260 L – x% 2 000 . x = 260 . 100 → E se a caixa for reabastecida até ficar com 520 L de água? De novo, a regra de três:

2 000 L – 100% 520 L – x% Fazendo as contas, chegamos ao resultado: 520 L de água correspondem a 26% da capacidade total da caixa. Repare: 520 L são o dobro de 260 L. Do mesmo modo, 26% é o dobro de 13%. Essa relação de proporção é válida para qualquer valor dado em porcentagem.


NA PRÁTICA

SAIBA MAIS

CONCENTRAÇÃO

A PROPORÇÃO DIVINA

Estima-se que 0,00014% do ar, em volume, é composto de metano, um gás inflamável, resultante da digestão de matéria orgânica. Veja que esse valor em porcentagem é muito baixo. Este é um caso em que convém aumentar a base de cálculo de porcentagem para partes por milhão (ppm). Veja:

0,00014% =

Por mais que pareça livre e desordenada, a natureza tem muitas formas que obedecem a regras rígidas de proporção. A espiral de uma folha de samambaia crescendo, como a da foto na pág. 19, por exemplo, segue uma curva que se abre unindo vértices opostos de quadrados cada vez maiores. As medidas dos lados desses quadrados seguem sempre a mesma sequência de proporção: 1, 1, 2, 3, 5, 8, 13... O mesmo acontece com a concha dos caracóis.

Queremos saber quanto 0,00014/100 representa em 1 milhão. 1 milhão é um número grande, que pode ser escrito como uma potência: 106 (veja potências no capítulo 4). Pela regra de três:

0,00014 – x 100 – 1 . 106 DANJIMENO/iSTOCK

100 x = 0,00014 . 106 → x = 1,4 Portanto, 0,00014% equivalem a 1,4 partes por milhão. E em 106 L de ar existe 1,4 L de metano.

5

SAIU NA IMPRENSA EL NIÑO ELEVOU CONCENTRAÇÃO DE GÁS DO EFEITO ESTUFA A NÍVEL RECORDE EM 2016 O fenômeno El Niño aumentou este ano a emissão de dióxido de carbono (CO2 ) na atmosfera, de acordo com um estudo publicado esta semana na revista Nature Climate Change. Por isso, 2016 terminará como o primeiro ano em que a concentração do gás será superior a 400 partes por milhão (ppm) (...) “A concentração de CO2 devido à ação humana está aumentando a cada ano, mas desta vez o El Niño deu um empurrão. Os ecossistemas tropicais estão mais quentes e secos, reduzindo sua absorção de carbono e aumentando os incêndios florestais” – comenta Richard Betts, autor principal do estudo (...) A tendência de aumento das emissões de gás de efeito estufa em Mauna Loa começou a ser estudada desde 1958 (...) Suas primeiras medidas registraram em torno de 315 ppm de dióxido de carbono. Sessenta anos mais tarde, o índice tem aumentado, em média, a uma taxa anual de 2,1 ppm (...) Atualmente o CO2 em Mauna Loa está acima de 400 ppm (...)

3

1 2

1 8

Esta é a sequência de Fibonacci. Nela, cada número é a soma dos dois termos que o antecedem: 2 é a soma 1 + 1; 3 é a soma de 2 + 1; 5 é a soma 3 + 2, e assim por diante. Além disso, a divisão de um termo por seu antecessor sempre dá um número próximo a 1,6. E quanto mais à frente da sequência estiverem os termos, mais a proporção se aproxima desse valor. Essa proporção, chamada proporção áurea ou divina, foi adotada por pintores e escultores, como o italiano Leonardo da Vinci, em seu quadro mais famoso, Mona Lisa.

O Globo, 13/6/2016 GE MATEMÁTICA 2017

23


BRUNO VEIGA

NÚMEROS E OPERAÇÕES JUROS

O custo do dinheiro Juro é o valor que se paga a mais por um valor emprestado, ou que se recebe por um investimento

24 GE MATEMÁTICA 2017

J

uro é um conceito do mundo financeiro que está presente no dia a dia de empresas, governos e cidadãos. Por exemplo, os governos pagam juros por empréstimos feitos no exterior (dívida externa); as indústrias pagam juros quando financiam a compra de equipamentos; o consumidor paga juros aos bancos se entrar no cheque especial e os investidores recebem juros por aplicações financeiras, como depósitos na caderneta de poupança. Juro é o custo do dinheiro, uma porcentagem do valor original emprestado, que o devedor deve pagar depois de certo período. É como se o tomador do empréstimo pagasse um aluguel pelo dinheiro que lhe foi cedido. A quantia emprestada (ou investida), sobre a qual incidem os juros, é o capital. E o capital acrescido de todos os juros chama-se montante.

MAIS FÁCIL, MAS MAIS CARO Quando parcelam o preço de um produto, as lojas cobram uma quantia a mais, a cada mês – os juros


A taxa de juros é o valor, em porcentagem, a ser pago a cada dia, mês ou ano, até a quitação total da dívida – ou o valor, também em porcentagem, que o aplicador recebe por um investimento.

Juros simples

São lançados sobre a quantia original, numa taxa fixa a cada período. Não importa em quantos dias, meses ou anos o empréstimo será pago, a taxa de juros será sempre a mesma e será sempre calculada sobre o capital inicial. Veja o exemplo: Sua classe planeja uma viagem de formatura, por um pacote turístico que custará a cada aluno R$ 1 200,00. Alguns de seus colegas não dispõem dessa quantia. Então, a agência de viagens propõe que o valor seja dividido em seis parcelas – a 1ª delas, paga 30 dias depois da compra –, com juros de 5% ao mês. Ao dividir o pagamento, a agência está financiando a viagem – ou seja, emprestando dinheiro a quem não consegue pagar pelo pacote, à vista. Por esse empréstimo, a agência cobra juros. Se o valor do pacote (R$ 1 200,00) é dividido em seis vezes, a cada mês o viajante deve pagar R$ 200,00. Só que, por esse parcelamento, a agência cobra 5% a cada mês sobre o valor inicial da dívida, os R$ 1 200,00:

A cada mês, então, o viajante deverá pagar R$ 60,00 a mais, além dos R$ 200,00. Ao final dos seis meses, terá pago seis prestações de R$ 260,00. Isso significa que o pacote turístico terá saído não mais por R$ 1 200,00, mas por R$ 1 560,00. Ou seja, o pacote saiu 30% mais caro. Veja: 1 200 – 100% 1 560 – x% x = 130% Desses 130%, 100% correspondem ao valor original do pacote de viagem e 30%, ao acréscimo de R$ 60,00 mensais durante seis meses. O total de juros simples é dado por: J = C . i . n, em que: • J são os juros; • C é o capital; • i é a taxa de juros; • n é o número de períodos (que podem ser dias, meses ou anos).

NA PRÁTICA JUROS SIMPLES

Um produto custa R$ 3 500, para pagamento em três prestações. Para pagamento à vista, a loja dá um desconto de 10%. Caso o comprador pague em uma única parcela 30 dias depois da compra, o preço sofrerá um acréscimo de 8%. Responda: a) Quanto o comprador deve desembolsar em cada uma dessas situações? b) A taxa de juros do cheque especial é de 12,5% ao mês. Vale a pena o comprador gastar R$ 1 500 do cheque especial para fazer a compra à vista, com desconto? a) À vista: com o desconto de 10%, o produto custa 90% do preço de tabela. Pela regra de três, temos: 3 500 – 100% x – 90% 100 . x = 3 500 . 90 → x = 315 000 / 100 → x = R$ 3 150 Este é o preço do produto à vista. O comprador economiza R$ 350. Para pagamento 30 dias após a compra: acréscimo de 8% sobre o valor original. De novo, pela regra de três, temos 3 500 – 100% x – 8% x = 3 500 . 8 / 100 → x = R$ 280 Somando essa diferença ao preço original, o comprador pagará R$ 280 a mais, ou seja, R$ 3 780. b) Supondo que o comprador reponha os R$ 1 500 do cheque especial em um mês, o montante que ele pagará corresponde ao capital emprestado acrescido de 12,5% desse valor: 1 500 – 100% x – 12,5% x = 187,50 reais Somando esses R$ 187,50 ao valor do produto com desconto: 3 150 + 187,50 = R$ 3 337,50. Este é o montante. Ainda com os juros altos do cheque especial, o valor de R$ 3 337,50 é menor do que o valor pago 30 dias depois da compra (R$ 3 780). Nesse caso, vale a pena avançar no negativo.

O montante (M) é dado por: M=C+J=C+C.i.n M = C . (1 + i . n) GE MATEMÁTICA 2017

25


NÚMEROS E OPERAÇÕES JUROS

Juros compostos

Ao final dos quatro meses de aplicação do Juros simples, você viu, é uma taxa fixa por capital de R$ 800,00, seu colega terá juntado mês, sempre sobre o valor original do finan- um montante de R$ 886,50. ciamento ou empréstimo (o capital). Já juros Abatendo essa quantia dos R$ 1 200,00 (valor compostos são aqueles que incidem sobre o do pacote), ele precisará financiar R$ 313,50. montante de cada mês – ou seja, são juros O rendimento da aplicação é mensal, então calculados sobre valores que já têm juros em- o período é 1 mês; o número de períodos é o butidos. A taxa é sempre a mesma, mas o valor número de meses em que o capital permaneceu aplicado: 4. Repare que o montante ao final de que ela representa varia. Voltando ao exemplo da viagem de formatura, cada período se transforma no capital do mês que vimos há pouco: a fim de pagar pela viagem seguinte. É sobre esse capital – agora engordado de formatura, um aluno preferiu fazer uma pou- – que incidirá a taxa de juros de 2,6%. pança e depositou, em julho, R$ 800,00 numa A fórmula para o cálculo do montante em aplicação financeira que rendia 2,6% ao mês. A juros compostos é: passagem será comprada em novembro. Veja na Mn = C (1 + i)n , em que: tabela abaixo quanto ele conseguirá acumular • M é o montante (valor final, depois de aplicanesses quatro meses. dos todos os juros); • C é o capital (o valor inicial sobre Saldo no fim Saldo inicial Rendimento o qual incidem os juros); do período no período Período no período • i é a taxa de juros; (montante) (juros) (capital) • n é o período em que os juros incidem sobre o capital. Julho 800,00 800,00 . 2,6% 800,00 + 20,80 = 820,80 Em juros compostos, n é expoente Agosto 820,80 820,80 . 2,6% 820,80 + 21,34 = 842,14 da taxa. Por isso se o capital aumenta, o novo montante também aumenSetembro 842,14 842,14 . 2,6% 842,14 + 21,90 = 864,04 ta num ritmo cada vez mais rápido Outubro 864,04 864,04 . 2,6% 864,04 + 22,46 = 886,50 – mesmo com a taxa de juros igual.

TOME NOTA A taxa Selic subiu de 12,25% para 12,75% entre fevereiro e março de 2015. Isso não significa que a taxa tenha subido 0,50% nesses dois meses. A taxa teve uma alta de 0,5 ponto percentual.

SAIBA MAIS TAXA DE JUROS NO BRASIL

No Brasil, a taxa de juros cobrada pelos bancos é baseada na taxa Selic – uma taxa básica, estabelecida pelo Banco Central. Se a Selic sobe, os bancos também elevam a taxa cobrada em financiamentos, empréstimos e cheque especial. As autoridades monetárias usam dessa lógica para controlar a quantidade de dinheiro que circula pelo mercado, o nível de consumo e a inflação. Quando a ideia é incentivar o consumo, o Banco Central baixa a taxa Selic; se quer reduzir o consumo, aumenta a taxa. O aumento da taxa de juros tem dois efeitos: de um lado, as pessoas compram menos porque, para financiar a compra, pagarão juros mais altos. De outro lado, as indústrias também reduzem a compra de equipamentos, porque o financiamento custa caro. Com isso, as empresas deixam de crescer e de contratar mão de obra. No sentido inverso, quando a taxa cai, as indústrias investem e voltam a contratar, e o consumidor compra mais – a economia se aquece. Mas aí entra outro fator: o risco de elevar a inflação. Inflação é o aumento no preço de produtos e serviços, provocado pela queda no valor da moeda do país. Entenda: se no mês passado 1 quilo de laranjas saía por R$ 3,50 e este mês custa R$ 4,50, cada real que você tem na carteira passou a valer menos.

26 GE MATEMÁTICA 2017

EVOLUÇÃO DA TAXA DE JUROS Taxa Selic, em % ao ano

13,75%

15

14,25%

12,25% 11%

10,50%

12

8,50% 9

6

J A S O N D J F M A M J 2013

2014

J A S O N D J F M A M J J A S O N D 2015

GANGORRA FINANCEIRA O Banco Central manobra a taxa básica de juros tentando manter a economia em movimento e a inflação sob controle. Elevar a taxa é um dos mecanismos para combater a inflação. Com taxas altas, as pessoas reduzem as compras a prazo. O consumo cai e, para vender, o comércio e a indústria seguram os preços – a inflação fica sob controle. Mas, produzindo e vendendo menos, as lojas e fábricas contratam menos. Se a economia desacelera muito, o Banco Central volta a baixar a taxa. Fonte: Banco Central do Brasil


NA PRÁTICA

SAIBA MAIS

JUROS COMPOSTOS

SPREAD BANCÁRIO

Uma aplicação financeira promete remunerar em 1,8% ao ano o capital investido. Se você aplicar R$ 2 000,00 quanto terá depois de dois anos? Este cálculo é de juros compostos porque no segundo ano os juros de 1,8% devem incidir sobre o capital inicial já acrescido dos juros do primeiro ano. Então Mn = C . (1 + i)n, em que:

C = R$ 2 000 i = 1,8% = n = 2 anos

M = 2 000 . (1 + )2 M = 2 000 . 1,036324 M ≈ 2 072,65 Depois de dois anos de aplicação, a 1,8% ao ano, você terá R$ 2072,65 – ou seja, R$ 72,65 de juros.

Os juros que são pagos aos bancos são sempre maiores que as taxas da Selic. Isso porque as instituições financeiras incorporam o chamado spread bancário. O spread é a diferença entre o que um banco paga como rendimento de investimentos de seus correntistas e o que recolhe de juros para emprestar dinheiro. Nem todo o spread é lucro. Incluem-se ali, também, outros valores, como o risco estimado de inadimplência (falta de pagamento) dos tomadores de empréstimo e os custos administrativos da instituição (veja o gráfico abaixo).

O QUE COMPÕE O SPREAD BANCÁRIO*

8%

Resíduo (inclui o lucro do banco)

4%

14%

Risco de inadimplência

37%

37%

NA PRÁTICA

Custos administrativos Tributos e taxas pagos pelo banco Depósito compulsório (que os bancos são obrigados a fazer no BC) * Valores arredondados

JUROS COMPOSTOS

A fatura do cartão de crédito de João, em março, era de R$ 1 200,00. Desse total, João só pôde pagar R$ 800,00. Sabendo que os juros cobrados pelo cartão são de 15% ao mês, responda: a) Quanto João deve pagar, se quitar o restante da dívida no mês seguinte, abril? b) E se ele deixar para quitar o restante da dívida em maio? a) João pagou R$ 800,00 do total de R$ 1 200,00 que devia. Ficou devendo R$ 400,00. Se pagar em abril, os 15% a mais representam juros simples sobre os R$ 400,00 devidos em março. Simples regra de três: 400 – 100% x – 15% x = 60 João pagará R$ 60,00 a mais se quitar a dívida em abril – ou seja, R$ 460,00. b) Se ele deixar para quitar os R$ 400,00 em maio, o cálculo é de juros compostos – a cada mês a taxa de 15% incide sobre o valor devido naquele mês. De março a maio são dois meses. Então: Mn = C . (1 + i)n C = 400; i = 15/100; n = 2 )2 → M = 400 . 1,3225 → M = 529 M = 400 . (1 + Se adiar a quitação da dívida para maio, a dívida original, de R$ 400,00, se transformará em R$ 529,00.

A GORDA FATIA DO LUCRO Estes são os componentes do spread bancário – a diferença entre as taxas de juros que os bancos cobram de quem toma empréstimo ou financia a aquisição de bens e aquela que a instituição paga como retorno do dinheiro deixado nas aplicações financeiras. Repare que nem tudo é lucro, mas este representa uma boa fatia da pizza. Fonte: BC/FSP

SAIU NA IMPRENSA PARA COPOM, QUEDA DA INFLAÇÃO ESTÁ COM VELOCIDADE 'AQUÉM DA ALMEJADA' O Comitê de Política Monetária (Copom) do Banco Central, colegiado responsável por fixar os juros básicos da economia, avaliou, por meio da ata de sua última reunião, que o processo de queda da inflação no Brasil “tem procedido em velocidade aquém da almejada” e acrescentou que o “balanço de riscos” indica não haver espaço para corte de juros. Na semana passada, o Copom manteve a taxa básica de juros da economia estável em 14,25% ao ano, o maior patamar em dez anos (...) O Banco Central também avaliou, no documento, que “há riscos de curto prazo para a inflação no Brasil”. (...) A taxa de juros é o principal mecanismo usado pelo BC para controlar a inflação. Ao subir os juros ou mantê-los elevados, o BC encarece o crédito. O objetivo é reduzir o consumo no país para conter a inflação que tem mostrado resistência. Entretanto, os juros altos prejudicam a atividade econômica e, consequentemente, inibem a geração de empregos. Portal G1, 26/7/2016 GE MATEMÁTICA 2017

27


COMO CAI NA PROVA

1. (IFPE 2016) Em uma cooperativa de agricultores do município de Vitória

de Santo Antão, foi realizada uma consulta em relação ao cultivo da cultura da cana-de-açúcar e do algodão. Constatou-se que 125 associados cultivam a cana-de-açúcar, 85 cultivam o algodão e 45 cultivam ambos. Sabendo que todos os cooperativados cultivam pelo menos uma dessas duas culturas, qual é o número de agricultores da cooperativa? a) 210 b) 255 c) 165 d) 125 e) 45

RESOLUÇÃO Podemos representar a situação do enunciado por um diagrama de Venn. Veja:

125 - 45 = 80

45

cana-de-açúcar

85 - 45 = 40

algodão

Repare: • Na parte central do diagrama estão os cooperados que cultivam tanto algodão quanto cana-de-açúcar – ou seja, a intersecção dos dois conjuntos. • O texto diz que 125 cooperados cultivam cana-de-açúcar e outros 85 que cultivam algodão – ou seja, entre esses 125 e 85 estão, também, os cooperados que cultivam tanto cana quanto algodão. Por isso, tiramos 45 (intersecção) dos dois lados do diagrama. Somando as quantidades que restam, temos: 45 + 80 + 40 = 165 Resposta: C

2. (CFTMG 2016) Numa fábrica de peças de automóvel, 200 funcionários

trabalhando 8 horas por dia produzem, juntos, 5 000 peças por dia. Devido à crise, essa fábrica demitiu 80 desses funcionários e a jornada de trabalho dos restantes passou a ser de 6 horas diárias. Nessas condições, o número de peças produzidas por dia passou a ser de a) 1 666 b) 2 250 c) 3 000 d) 3 750

RESOLUÇÃO: A produção diária é diretamente proporcional ao número de funcionários e à quantidade de horas que eles trabalham por dia. Um aumento ou uma redução em qualquer uma dessas variáveis produzem um aumento ou diminuição proporcional na produção. Chamando de P a produção diária de peças, de F a quantidade de funcionários e de t a quantidade de horas trabalhadas, temos P = k . F . t, em que k é uma constante de proporcionalidade – ou seja, o valor que vai determinar a proporção entre o número de funcionários e o de peças produzidas. Pelo enunciado sabemos que 5 000 = k . 200 . 8 k = 5 000 → k = 5 000 200 . 8 1 600 Simplificando a fração, ficamos com k = 25 peças por funcionários . hora 8 Se o ritmo de produção é o mesmo, o valor de k não muda. O número de empregados caiu de 200 para 120 (80 foram demitidos); e a jornada

28 GE MATEMÁTICA 2017

de trabalho foi reduzida de 8 para 6 horas. Portanto, a produção também deve cair. E cai na mesma proporção de k = 25 peças por funcionários . hora. 8 Temos, então: p = 25 . 120 . 6 = 2 250 peças 8 Resposta: B

3. (UEG 2016) Com a alta da inflação e para não repassar aos clientes o

aumento dos gastos na produção de suco de laranja, um empresário decidiu que no próximo mês 10% do volume desse suco será composto por água, volume que atualmente é de apenas 4%. Se hoje são consumidos 10 000 litros de água no volume de suco de laranja produzido, mantendo-se a mesma quantidade produzida, no próximo mês a quantidade de água consumida no volume desse suco será de a) 10 000 litros b) 12 500 litros c) 16 000 litros d) 25 000 litros

RESOLUÇÃO Sabemos que, antes do período de inflação, 4% do volume de suco produzido representava 10 000 litros de água. Com isso, montamos a regra de três para descobrir o volume total de suco (incluindo a água) produzido em um mês: 10 000 → 4% x → 100% x = 250 000 L de suco puro produzido ao mês. A quantidade de água subiu para 10% do volume de suco – ou seja, 10% sobre o total da produção mensal, de 250 000. A água consumida no mês seguinte será de 25 000 litros. Resposta: D

4. (Uerj 2016) Na compra de um fogão, os clientes podem optar por uma das

seguintes formas de pagamento: • à vista, no valor de R$ 860,00; • em duas parcelas fixas de R$ 460,00, sendo a primeira paga no ato da compra e a segunda 30 dias depois. A taxa de juros mensal para pagamentos não efetuados no ato da compra é de: a) 10% b) 12% c) 15% d) 18%

RESOLUÇÃO Se o comprador optar por pagar em duas vezes, não incidirão juros sobre a primeira parcela. Pagando R$ 460,00 no ato da compra, restam como dívida R$ 860,00 – R$ 460,00 = R$ 400,00. Esse valor deve ser quitado na segunda parcela. Mas esta, por sua vez, se mantém no valor da primeira parcela (R$ 460,00). Ou seja, o comprador pagará R$ 60,00 acima do valor original da dívida. Esses R$ 60,00 são os juros cobrados sobre os R$ 400,00. Agora, simples regra de três 400 → 100% 60 → x% Multiplicando em cruz, temos 400 x = 60 . 100 x = 6 000 = 15% 400 Resposta: C


RESUMO

5. (Epcar 2016) O dono de uma loja de produtos seminovos adquiriu, parcela-

damente, dois eletrodomésticos. Após pagar 2/5 do valor dessa compra, quando ainda devia R$ 600,00, resolveu revendê-los. Com a venda de um dos eletrodomésticos, ele conseguiu um lucro de 20% sobre o custo, mas a venda do outro eletrodoméstico representou um prejuízo de 10% sobre o custo. Com o valor total apurado na revenda, ele pôde liquidar seu débito existente e ainda lhe sobrou a quantia de R$ 525,00. A razão entre o preço de custo do eletrodoméstico mais caro e o preço de custo do eletrodoméstico mais barato, nessa ordem, é equivalente a a) 5 b) 4 c) 3 d) 2

RESOLUÇÃO Vamos chamar o valor pago pelo eletrodoméstico mais caro de a e o valor do mais barato de b. Se após pagar 2/5 da compra restou uma dívida de R$ 600,00, então esse valor representa 3/5 da compra, ou seja, do valor de a + b . Sendo assim, 3 (a + b) = 600 5 Multiplicando em cruz, ficamos com a + b = 600 . 5 → a + b = 1 000 3 O enunciado diz que o comerciante vendeu o produto de valor a com lucro de 20%. Pelo raciocínio de porcentagem, temos: 100% – 1 → 20% = 0,2 20% – x Mas esses 20% (0,2) são de lucro e, portanto, devem ser somados aos 100% do preço de compra. Ficamos, então, com (1 + 0,2) . a = 1,2 . a O mesmo raciocínio para o valor b, de venda do segundo produto. Só que agora a venda foi com prejuízo de 10%. Então, 100% – 1 → 10% = 0,1 10% – x

Números e operações PROPORÇÃO E RAZÃO Duas grandezas são diretamente proporcionais se uma cresce e a outra também, no mesmo ritmo; inversamente proporcionais são as grandezas que, quando uma cresce, outra diminui, sempre proporcionalmente. Algumas grandezas são expressas como razão de duas grandezas: densidade (massa/ volume) e concentração (% de soluto sobre total da solução). JURO é o custo do dinheiro, cobrado em empréstimos e financiamentos, ou pago aos investidores. Capital é a quantia sobre a qual recaem os juros. Montante é a quantia total depois da incidência de juros sobre o capital. Juros simples são lançados sobre o capital, numa taxa fixa a cada período (dia, mês ou ano). A fórmula: J = C . i . n. Nos juros compostos, o montante de cada período transforma-se no capital do período seguinte. A fórmula: Mn = C . (1 + i)n. CONJUNTOS O conjunto C = A U B (união de A e B) contém os elementos que se encontram em A ou em B. O conjunto C = A ∩ B (intersecção de A e B) contém os elementos que se encontram em ambos os conjuntos, ao mesmo tempo. SÍMBOLO { } ∈, ∉

SIGNIFICADO Conjunto Pertence, não pertence

Tal que

Contém

Está contido

Como esses 10% foram de prejuízo, esse valor deve ser subtraído do valor de compra: (1 – 0,1) . b = 0,9 b.

Intersecção de conjuntos

União de conjuntos

O enunciado informa que o resultado das duas vendas foi suficiente para pagar o restante da dívida (R$ 600) e ainda rendeu ao comerciante R$ 525.

Conjunto vazio

N

Conjunto dos números naturais

Z

Conjunto dos números inteiros

Q

Conjunto dos números racionais

I

Conjunto dos números irracionais

R

Conjunto dos números reais

*

Exceto o zero

Você se lembra: para resolver duas equações com duas variáveis, montamos o sistema de equações: a + b = 1 000 (I) 1,2 a + 0,9 b = 1 125 (II) Definimos o valor de uma variável em função de outra. Assim, isolando a variável a da primeira equação, obtemos: a = 1 000 – b (III). Substituindo (III) na equação (II), temos: 1,2 (1 000 – b) + 0,9 b = 1 125 1 200 – 1,2 b + 0,9 b = 1 125 – 0,3 b = – 75 b = 75 /0,3 → b = 250

+/–

São válidos valores positivos e negativos

CONJUNTOS NUMÉRICOS

N Z

Substituindo o valor de b em (III), temos que a = 1 000 – 250 = 750. Assim, a razão pedida é a = 750 = 3 b 250 Resposta: C

I

R

Q

GE MATEMÁTICA 2017

29


2

GEOMETRIA CONTEÚDO DESTE CAPÍTULO

Ponto, reta e plano ..........................................................................................32 Plano cartesiano ..............................................................................................36 Gráficos................................................................................................................38 Polígonos ............................................................................................................40 Cônicas ................................................................................................................46 Sólidos .................................................................................................................52 Como cai na prova + Resumo .......................................................................58

Júpiter recebe um bisbilhoteiro A sonda Juno, da Nasa, chega à órbita do maior planeta do Sistema Solar, prometendo desvendar segredos da origem dos demais planetas

P

ara os norte-americanos, o dia 4 de julho é tradicionalmente coroado com chuvas de fogos de artifício, em comemoração à Independência dos Estados Unidos. Este ano, uma equipe de engenheiros da agência espacial norte-americana (Nasa) teve sua festa particular, sem fogos, mas com muitos gritos e aplausos. O motivo: a sonda Juno chegara a Júpiter. Depois de viajar por cinco anos, percorendo uma trajetória cheia de voltinhas, de quase 3 bilhões de quilômetros, a sonda passou 35 minutos fazendo uma série de manobras para entrar na órbita do planeta. Qualquer desvio, e a nave seria atraída e destruída pela incrível gravidade do gigante gasoso. Juno vai dar 37 voltas em torno de Júpiter, durante um ano, estudando sua atmosfera, seu campo magnético e gravitacional. Os pesquisadores esperam com isso desvendar detalhes da formação do Sistema Solar. E, de quebra, compreender a dinâmica de sistemas extrassolares – planetas em torno de estrelas distantes. De todos os corpos que giram ao redor do Sol, Júpiter é de longe o maior, tanto em tamanho quanto em massa. Seu diâmetro é dez vezes maior que o da Terra, e sua massa, 318 vezes. Mas o mais interessante para a missão científica de Juno é que Júpiter é, provavelmente, o pri-

30 GE MATEMÁTICA 2017

meiro a ter se formado, há cerca de 4,6 bilhões de anos, capturando poeira e gases resultantes da explosão de uma estrela que existia, no lugar onde hoje está o Sol. Os cientistas não sabem ainda o que se esconde debaixo dos milhares de quilômetros de nuvens, sequer se Júpiter tem um núcleo sólido, como a Terra. A resposta pode vir da missão Juno. Esta não é a primeira vez que uma sonda bisbilhota Júpiter. Em 1995, a sonda Galileu, também da Nasa, entrou, pela primeira vez, em órbita do planeta, e lançou uma sonda filha, que mergulhou durante quase uma hora em sua atmosfera. Dessa missão, os cientistas descobriram que mais de 90% da atmosfera joviana é composta de hidrogênio. Antes de ser vaporizada, a sondinha registrou temperaturas de 300 graus Celsius e ventos de mais de 600 quilômetros por hora. Como os demais planetas do Sistema Solar, Júpiter descreve uma VISITANTE XERETA órbita elíptica em torno A sonda Juno chegou a do Sol. Elipse é um dos Júpiter em 2016 e vai passar temas deste capítulo. um ano enviando dados Aqui você vê, também, sobre a composição e a o cálculo de área e de dinâmica da atmosfera, a volume das principais magnetosfera e o campo figuras geométricas. gravitacional do planeta


JPL-CALTECH/NASA

GE MATEMÁTICA 2017

31


2

GEOMETRIA PONTO, RETA E PLANO

Só duas dimensões Retas e ângulos são os elementos essenciais das figuras geométricas lineares

G

eometria é a área da matemática que estuda o espaço e as figuras que ocupam esse espaço – suas formas, suas dimensões e as relações que podem ser estabelecidas entre elas. O espaço estudado pela geometria pode ser plano ou tridimensional.

Plano, ponto e reta

Plano é definido como um objeto geométrico que tem apenas duas dimensões: comprimento e largura. O elemento mais simples de um plano é o ponto, uma entidade que não tem dimensões. Bastam três pontos para definir um plano. O segundo elemento mais simples é a reta – um conjunto de infinitos pontos, enfileirados, sempre em uma mesma direção e nos dois sentidos. Ou

32 GE MATEMÁTICA 2017

seja, qualquer reta tem comprimento infinito, mas não tem largura. Para definir uma reta precisamos de apenas dois pontos. Os geômetras adotam algumas convenções, que você deve conhecer: • pontos são normalmente batizados com letras maiúsculas: A, B, C, O...; • retas são geralmente indicadas por letras minúsculas: r, t, s...; • e planos costumam ser indicados por letras do alfabeto grego: α (alfa), β (beta) e γ (gama). r P

α

ponto

reta

plano


DRAZEN LOVRIC/iSTOCK

Uma reta pode ser paralela a um plano. Nesse caso, nenhum de seus pontos pertence ao plano: C

D

s

Lembrando que toda reta é infinita, se duas retas não forem paralelas, elas se cruzarão em algum lugar. Inversamente, se a intersecção do conjunto de pontos da reta r com o conjunto de pontos da reta s for um conjunto vazio, as retas são obrigatoriamente paralelas: r // s ) r ∩ s = ∅ indica que a recíproca é O sinal verdadeira.

)

α

C e D ∈ s; C e D ∉ α s ∉ α. Portanto, a reta s é paralela a α. O REAL ACHATADO O plano, como o do papel em que o desenho ao lado é feito, admite apenas figuras de duas dimensões

Uma reta pode, finalmente, cortar o plano em um ponto qualquer.

α

B

r

α

A e B ∈ r; A e B ∈ α r ∈ α Ou seja, a reta r está contida no plano α.

s

r

α

Duas retas quaisquer r e s são concorrentes quando a intersecção entre os conjuntos de pontos de cada uma delas resulta num conjunto de um único ponto: r ∩ s = {P}.

P

A

P

t

Posições da reta em relação ao plano

Uma reta pertence a um plano se pelo menos dois de seus pontos pertencerem a esse plano. Se isso acontecer, então todos os outros pontos da reta também pertencerão ao plano. Veja:

• Concorrentes: são retas que se cruzam e têm um único ponto em comum.

Ângulos t ∩ α = P t é secante ao plano

Posição relativa de retas

Pensando na reta como um conjunto de pontos e usando a linguagem dos conjuntos, fazemos relações entre elas. Duas retas que ocupam um mesmo plano podem ser: • Paralelas: não têm ponto em comum.

Quando duas semirretas (trechos de uma reta) têm origem em um mesmo ponto e seguem direções diferentes, elas dividem o plano em duas regiões chamadas ângulos. O ponto de origem das semirretas é denominado vértice dos ângulos (O). Os ângulos, como os planos, também costumam ser representados por letras do alfabeto grego. ângulo β (não convexo)

r s

A

OA

ângulo α (convexo)

α

O

B

OB

GE MATEMÁTICA 2017

33


2

GEOMETRIA PONTO, RETA E PLANO

Duas retas que se cruzam dividem o plano em quatro regiões distintas, ou seja, em quatro ângulos. Veja: r

τ

Transversal e paralelas

Duas retas paralelas que são cortadas por uma terceira reta (transversal) formam oito ângulos que se relacionam de maneira bem específica. Acompanhe na figura as explicações no texto a seguir.

θ φ

λ

λ

τ

Os ângulos λ e τ são opostos pelo vértice; θ e φ também são opostos pelo vértice. Ângulos opostos pelo vértice são congruentes (têm a mesma medida). Retas perpendiculares são retas concorrentes que se cruzam formando quatro ângulos congruentes, cada um deles medindo 90° (ângulo reto).

λ

θ

τ

φ

• Ângulos alternos são pares de ângulos que estão em lados diferentes (alternados) da reta transversal. Dois ângulos alternos têm medidas iguais. Os alternos são internos quando ficam entre as retas paralelas. Na figura, são alternos internos os pares τ/θ’ e λ’/φ.

t

s

O

são sempre congruentes. No caso das duas paralelas cortadas por uma transversal, são opostos pelo vértice os pares λ/φ, θ/ τ, λ’/φ’ e θ’/τ’.

λ' τ'

θ

r

φ θ'

Ângulos alternos externos são aqueles que estão na região externa das retas paralelas (acima ou abaixo delas). São alternos externos os ângulos λ/φ’ e θ/τ’.

s

φ'

• Ângulos colaterais são aqueles que ocupam o mesmo lado da reta transversal. Eles também podem ser internos (entre as paralelas) ou externos. Na figura, são colaterais internos τ/λ’ e φ/θ’; são colaterais externos λ/τ’e θ/φ’.

• Ângulos adjacentes são ângulos que compartilham um mesmo lado: Entre as retas r e t, são adjacentes os pares λ/θ, τ/λ, τ/φ e φ/ θ; Entre as retas s e t, são adjacentes os pares λ’/θ’, τ’/λ’, τ’/φ’ e φ’/θ’. Os ângulos adjacentes somam 180º – ou seja, formam um conjunto de ângulos suplementares.

• Ângulos correspondentes são aqueles que se encontram do mesmo lado da reta transversal, um na região interna das retas paralelas e outro na região externa. Ângulos correspondentes são congruentes. Na figura, são pares de ângulos correspondentes λ/λ’, τ/τ’, θ/θ' e φ/φ’.

• Ângulos opostos pelo vértice, como já vimos, são ângulos que compartilham o vértice, mas não compartilham lados. Dois ângulos opostos pelo vértice

λ = θ = τ = φ = 90º

NA PRÁTICA

Por partes: • Se você prolongar as retas dos segmentos BC, DE, AB e AC, vai reconhecer a situação como a de duas retas paralelas (DE e BC) cortadas por duas transversais (AB e AC). Veja:

PARALELAS E TRANSVERSAIS Observe a figura abaixo.

A

D B

A

E

D C

Repare que existem aqui dois triângulos (ABC e ADE). E que os lados DE e BC são paralelos. O que se pode dizer sobre os valores dos ângulos de vértices em B, C, D e E?

34 GE MATEMÁTICA 2017

B

E

C

• Em relação à transversal AC, os ângulos com vértices em C e E são correspondentes e, portanto, congruentes. O mesmo ocorre com os ângulos D e B, em relação à transversal AB. Esta situação é muito importante para reconhecer a semelhança entre triângulos (veja o capítulo 5).


Mais importante que conhecer os nomes desses pares de ângulos é saber reconhecer as relações entre eles. E, para isso, você só precisa treinar a observação – reparar as semelhanças e diferenças entre dois ângulos.

NA PRÁTICA TEOREMA DE TALES

Qual a medida do segmento DF, na figura abaixo? u

Teorema de Tales

Retas transversais mantêm uma relação de proporção bem definida. E o que define essa proporção é o teorema de Tales: qualquer conjunto de retas paralelas cortadas por segmentos transversais formam nessas transversais segmentos proporcionalmente correspondentes. Mais fácil acompanhando na figura:

v

A B

D

r

2x + 1 E

5

s 3x

7

F

C

t r//s//t

Por Tales, sabemos que AB/DE = BC/EF. Então, u

5 7 = 2x + 1 3x

v

Multiplicando em cruz: A B

3

D 2

r

E s

9 C

x F

t r//s//t

Veja: • As retas r, s e t formam um feixe de retas paralelas; • As retas u e v (que não são paralelas, mas concorrentes) cortam o feixe r, s e t. Os pontos de intersecção das três retas definem os pontos A, B, C, D, E e F. Segundo Tales, os segmentos correspondentes em cada uma das retas transversais são proporcionais. Na figura, as medidas de AB e DE guardam uma razão de 3/2. Então, os segmentos BC e EF têm a mesma relação de proporção. Ou seja, BC AB DE = EF

Com isso, é possível determinar o valor de x (medida de EF): 9 3 2 = x 3 . x = 2 . 9 3 . x = 18 x = 6

5 . 3x = 7 . (2x + 1) 15x = 14x + 7 x=7 O segmento DE = 2 . x + 1 = 2 . 7 + 1 → DE = 15 O segmento EF = 3 . x = 3 . 7 → EF = 21 Por fim, o segmento DF é a soma de DE e EF: 21 + 15 → DF = 36

ATENÇÃO QUANDO TALES NÃO RESOLVE

Para resolver um problema de retas paralelas e transversais, só podemos usar o teorema de Tales quando temos as medidas de todos os segmentos de uma das retas que não são paralelas. Caso contrário, não é possível aplicar o teorema de Tales. Veja a figura: u

A

v

E

x

B

r

4

x+1 C

D 10

s

r//s Não temos a medida de nenhum dos segmentos da reta u. Mas temos as medidas de dois lados de dois triângulos: • Do triângulo AEB, conhecemos os lados AB = 4 e AE = x; • Do triângulo CED, conhecemos os lados CD = 10 e CE = x + 1 + x ; • Esses dois triângulos compartilham o ângulo no vértice E. Os ângulos em A e C são congruentes. O mesmo ocorre entre os ângulos em B e D. Portanto os dois triângulos são semelhantes. Nesse caso, usamos semelhança de triângulos (veja no capítulo 5). GE MATEMÁTICA 2017

35


GEOMETRIA PLANO CARTESIANO

ISTOCK

2

JOGO ORDENADO Um tabuleiro de xadrez, com suas 64 casas, reproduz com bastante precisão um plano cartesiano: cada casa é identificada por um par ordenado

A lógica do quadriculado

O SISTEMA COORDENADO

Um esquema engenhoso que permite localizar qualquer ponto pelo cruzamento de retas perpendiculares

O endereço de qualquer ponto é um par ordenado (x,y). Para este ponto, o valor de x é –2 e o de y, 2. Então o par ordenado do ponto A é (–2,2)

F

oi o filósofo e matemático francês René Descartes quem imaginou pela primeira vez um sistema para localizar qualquer ponto em um plano, o chamado sistema coordenado. Com isso, Descartes criou uma área nova da matemática, a geometria analítica, que reúne conhecimentos de geometria e de álgebra. O plano de Descartes é engenhoso. Veja ao lado.

5 y 4

A (-2,2)

2 Os eixos se cruzam no ponto de coordenadas (0,0). Este ponto é chamado origem do sistema coordenado

1 O (0,0) -4

-3

-2

-1 -1

Os valores nos eixos x e y são números reais – podem ser positivos, negativos, inteiros racionais ou irracionais

x 1

-2

-3

36 GE MATEMÁTICA 2017

No eixo vertical (y), os valores crescem de baixo para cima. O eixo y é o das ordenadas

3

2

3

4

Os valores no eixo horizontal (x) crescem da esquerda para a direita. Este é o eixo das abscissas


Unidades de medida

O plano cartesiano aceita qualquer unidade de medida e qualquer intervalo entre valores. Com isso, ele é ideal para a construção de gráficos que representam a relação entre diferentes grandezas. Veja dois exemplos: m 3

1 2

3

4

No gráfico acima, no eixo x estão indicados os intervalos de tempo, em segundo (s). E o y traz as medidas de distância, em metro (m). As unidades poderiam ser outras, como quilômetro e hora. 35

ºC VARIAÇÃO DE TEMPERATURA

25 10

20

30

40

Quadrantes

Os eixos dividem o plano em quatro regiões denominadas quadrantes. Os quadrantes são numerados no sentido antihorário, a partir do lado positivo do eixo x. Conhecendo o sinal das coordenadas de um ponto, é possível saber em qual quadrante ele se localiza.

Q II (-x,+y)

Q III (-x,-y)

7 6 5 4 3 2

y

QI (+x,+y)

2 1 0 -1

-1 -2

0 1

x

2 Q IV (+x,-y)

R2

R1

R3 C2

C3 D2

D3

R4 C1 C4 D1

D4 x

Repare: os valores nos eixos x e y podem equivaler a uma unidade qualquer. Se cada unidade fosse equivalente a 2 metros, por exemplo, a parede mais comprida da sala de reunião R1 a R4 mediria 6 m.

No gráfico de variação de temperatura, as unidades são graus Celsius (oC) por minuto (min). Mas poderia ser adotada outra unidade para a temperatura – Kelvin, por exemplo.

3

y

1 2 3 4 5 6 7 8 9 10

min

-2

9 8

1

30

-3

No projeto de um escritório de engenharia, o arquiteto estipulou as dimensões de três cômodos que serão construídos em sequência, em paralelo à fachada do prédio. Ele usou pontos que definem as arestas desses cômodos:

Veja a forma e a disposição dos cômodos no plano cartesiano:

s 1

DISTÂNCIAS NO PLANO CARTESIANO

• sala de reunião: R1(1,1), R2(1,3), R3(3,3), R4(4,1) • área de circulação: C1(4,1), C2(3,3), C3(6,3), C4(5,1) • sala da diretoria: D1(5,1); D2(6,3); D3(9,3); D4(9,1)

POSIÇÃO EM FUNÇÃO DO TEMPO

2

NA PRÁTICA

3

SAIBA MAIS AS COORDENADAS DO XADREZ

Um tabuleiro de xadrez, como o da foto na página ao lado, é composto de 64 casas distribuídas em oito colunas verticais e oito linhas horizontais. O tabuleiro funciona como um plano cartesiano, em que só se consideram valores inteiros e positivos. O eixo x é marcado com letras; o y, com números. E cada casa é identificada por um par ordenado, composto de uma letra e um número.

y

8

a8

b8

c8

d8

e8

f8

g8

h8

7

a7

b7

c7

d7

e7

f7

g7

h7

6

a6

b6

c6

d6

e6

f6

g6

h6

5

a5

b5

c5

d5

e5

f5

g5

h5

4

a4

b4

c4

d4

e4

f4

g4

h4

3

a3

b3

c3

d3

e3

f3

g3

h3

2

a2

b2

c2

d2

e2

f2

g2

h2

1

a1

b1

c1

d1

e1

f1

g1

h1

a

b

c

d

e

f

g

h

GE MATEMÁTICA 2017

x

37


GEOMETRIA GRÁFICOS

DIVULGAÇÃO

2

Traços que valem por mil palavras O plano cartesiano é a base de gráficos de barras e de linhas. Mas existem gráficos que dispensam os pares ordenados

38 GE MATEMÁTICA 2017

O

s gráficos são uma das maneiras mais fáceis de visualizar o comportamento de uma variável. Com eles, você percebe rapidamente a diferença entre quantidades, proporções entre valores e a evolução de uma variável ao longo do tempo. Por isso, muitos dados de pesquisas sobre economia, política, demografia e sociedade são apresentados na forma de gráficos. Existem diversas maneiras de representar uma realidade em gráfico, dependendo do que se pretende mostrar. E um mesmo assunto pode render todos os tipos de gráficos. Por exemplo, em questões relativas a energia. Matriz energética é o conjunto de fontes de energia de um país ou uma região, com a proporção entre a produção de diferentes fontes. A composição da matriz energética é importante para a discussão das questões relacionadas ao aquecimento global. Você sabe, a queima de combustíveis fósseis é uma das maiores causas do aumento na concentração de gases do efeito estufa. O Brasil tem uma das matrizes mais equilibradas do mundo, com as fontes renováveis respondendo por cerca de 40% de toda energia ofertada no país. É renovável, por exemplo, a energia hidráulica – das águas de um rio –, que gera eletricidade nas hidrelétricas. Acompanhe a seguir como o tema energia pode render diferentes gráficos.

RENOVÁVEL Das hidrelétricas saem quase 13% do total de energia ofertada no Brasil. Num gráfico, essa proporção fica bem clara


Gráfico de setores

Lembra uma pizza fatiada. Mostra a proporção entre valores, que podem ser dados em número absoluto ou em porcentagem. O tamanho de cada fatia (setor) é proporcional ao valor por ela representado. E essa proporção é definida pelo ângulo. O gráfico ao lado mostra a participação de diferentes fontes de energia na matriz energética brasileira. A maior parte da energia ofertada no país vem do petróleo (39,5%). A energia de fonte hidráulica (das hidrelétricas) representa 12,5% do total das fontes. Então, no gráfico, essa fatia deve medir o equivalente a 12,5% da circunferência total. Veja como esse tamanho foi calculado: • Uma circunferência completa tem 360o; • Para a fatia de 12,5%, simples regra de três: 360o – 100% x – 12,5% x = 360 . 12,5 / 100 x = 45o

Gráfico de barras

Ideal para comparar o comportamento de uma variável em diferentes situações. As barras, verticais ou horizontais, são desenhadas sobre valores de dois eixos, como o plano cartesiano. O gráfico ao lado mostra a participação de cada fonte de energia nos anos 1971, 2004 e 2030. No eixo x estão os anos; no y, os valores em toneladas equivalentes de petróleo (teps). Cada cor nas barras representa uma fonte da matriz de energia. Repare que o total de energia que se prevê seja consumida pelo mundo nos anos 2030 é praticamente três vezes maior do que em 1971. Note também que, apesar do aumento na oferta das fontes renováveis, a participação dos combustíveis fósseis triplica entre 1971 e 2030.

Gráfico de linhas

Outra maneira de mostrar a evolução de uma variável ao longo do tempo é por gráficos de linhas. Como exemplo, veja ao lado. O gráfico mostra a produção total de gás natural nos Estados Unidos (EUA) e a participação das principais fontes desse gás. Como o petróleo, o gás natural é um combustível fóssil. Recentemente, os EUA incrementaram a exploração do gás de xisto – uma rocha sedimentar formada da decomposição de matéria orgânica. A linha vermelha indica que a produção total de gás natural subiu 15% entre 2007 e 2011. Mas as demais linhas mostram que a produção de gás de todas as fontes caiu, ou permaneceu no mesmo patamar, entre os quatro anos. A exceção é a linha azul, de gás de xisto, que sobe em participação. Podemos concluir, então, que o aumento na produção de gás nos EUA se deveu à extração do xisto.

MATRIZ ENERGÉTICA BRASILEIRA (2013) Oferta de energia, % de participação de cada fonte primária no total

59%

41%

Energia não renovável Petróleo e derivados Gás natural Carvão mineral Urânio (Nuclear)

4 16 39,5

8,5

Energia renovável Hidráulica Lenha e carvão vegetal Biomassa de cana

12,5 13

5,5

Outras*

1 * Principalmente eólica, solar e geotérmica Fonte: Empresa de Pesquisa Energética (EPE)

60 ANOS DE ENERGIA Evolução da matriz de energia primária no mundo, em bilhões de teps 20

Petróleo Carvão Gás natural Energia nuclear Hidráulica Biomassa Outros renováveis*

15

10

5

0

1971

2004

2030** * Inclui eólica, geotérmica e solar ** Estimativa

Fonte: Agência Internacional de Energia/World Energy Outlook 2004 e 2006

PRODUÇÃO DE GÁS NATURAL NOS ESTADOS UNIDOS (2007-2012) Em bilhões de metros cúbicos Produção total de gás natural

de jazida natural

de petróleo

de gás de xisto

de carvão mineral

1000

800

698

726

759

454

480

738

806 *

600

450

398

400

369

200 170

0

60 2007

175

168 86

170

61

60

57

2008

2009

2010

119

255 177

174

53 2011

* Os números de produção total não incluem gás queimado, reinjetado ou perdido Fonte: Agência de Energia dos EUA 2013 GE MATEMÁTICA 2017

39


GEOMETRIA POLÍGONOS

ISTOCK

2

LÁBARO GEOMÉTRICO A bandeira brasileira é construída com a combinação de três figuras geométricas básicas: retângulo, losango e círculo

Desenhados com linhas retas Polígonos são figuras geométricas que têm lados e vértices

nos quais as extremidades dos segmentos se encontram, formando os vértices do polígono, e no qual os pontos em comum entre dois lados só podem ser os vértices do polígono. Veja os exemplos: B

A

C

F

E

A

o escolhermos dois pontos de uma reta, A e B, delimitamos um segmento de reta AB, de comprimento limitado. A

B r

Interligando segmentos de retas não alinhados, de dois em dois, cercamos uma região de um plano, definindo um polígono, ou figura plana. Cada segmento de reta corresponde a um lado do polígono. Os lados são representados por letras minúsculas: a, b, c,... Existem diversas famílias de polígonos, mas vamos nos concentrar nos polígonos fechados e simples – aqueles

40 GE MATEMÁTICA 2017

H

D A

G

A

B

C

B

C

E E

D G

D

F

Um polígono pode ser côncavo ou convexo. Se, ao unirmos dois pontos quaisquer de um polígono por um segmento de reta e parte desse segmento ficar para fora do polígono, então ele é chamado côncavo. Caso não exista a possibilidade de traçar um segmento que fique para fora, o polígono é convexo.

CÔNCAVO

Se todos os lados e ângulos de um polígono forem congruentes, isto é, se tiverem a mesma medida, então esse é um polígono regular. Por exemplo:

CONVEXO

Triângulo equilátero

Quadrado

Pentágono regular

Hexágono regular

Duas grandezas podem ser associadas às figuras planas: perímetro e área. O perímetro é a soma dos comprimentos dos lados. Área é a medida da superfície fechada pelo polígono. Veja no quadro no alto da página ao lado a fórmula para o cálculo da área dos principais polígonos regulares.


QUADRADO

RETÂNGULO A=a.h

PARALELOGRAMO

h

a

h a

a

a

TRAPÉZIO

TRIÂNGULO

LOSANGO diagonal m aior . diagonal m enor 2

A=

a.h 2

A=

(base m aior + base m enor) . h 2

base menor

diagonal menor

A=

A=a.h

A = a . a = a2

h

h

base maior

a

diagonal maior

NA PRÁTICA A BANDEIRA BRASILEIRA

As dimensões da bandeira brasileira seguem proporções rígidas, estabelecidas em lei federal: • Qualquer que seja o tamanho da bandeira, a altura deve ser dividida em 14 partes iguais (módulos). E o comprimento deve ter 20 desses módulos. • Os vértices do losango amarelo devem ficar a uma distância de 1,7 módulo da borda verde da bandeira. • E o raio do círculo azul é de 3,5 módulos. Considerando essas relações de proporção, calcule as áreas do retângulo verde, do losango e do círculo em uma bandeira com 7 metros de altura. Vamos chamar a altura de h, a largura de a e cada módulo de x. Acompanhe o raciocínio na figura abaixo.

Conhecemos a altura h. Então, descobrimos o valor do módulo x: 7 = 14 . x x = 0,5 m Agora encontramos as áreas de cada figura: • Para o retângulo verde: A = a . h Sabemos que o lado a deve corresponder a 20 módulos x. Então, a = 20 . x a = 20 . 0,5 a = 10 m A = 7 . 10 A = 70 m2

diagonal maior . diagonal menor 2

1,7 x

diagonal menor

h = 14 . x 3,5 . x a = 20 . x

Diagonal maior = 10 – 2 . (1,7 . 0,5) Diagonal maior = 10 – 1,7 Diagonal maior = 8,3 m A diagonal menor é calculada em relação à altura h: Diagonal menor = 7 – 2 . (1,7 . 0,5) Diagonal menor = 7 – 1,7 Diagonal menor = 5,3 m A área do losango amarelo:

• Para o losango, usamos a fórmula que relaciona as duas diagonais da figura: A=

Cada diagonal termina 1,7 módulo antes da borda da bandeira. A diagonal maior é calculada em relação ao lado a do retângulo:

A=

8, 3 . 5, 3 2

A ≈ 22 m2 • Para o círculo: A = π . r2 O raio mede 3,5 módulos: r = 3,5 . 0,5 r = 1,75 r2 ≈ 3 A = 3,14 . 3 A ≈ 9,4 m2

diagonal maior GE MATEMÁTICA 2017

41


2

GEOMETRIA POLÍGONOS

ATENÇÃO

TOME NOTA

Não se esqueça: a unidade dos lados é linear – cm, m, km, por exemplo. Já para a área, a unidade é sempre elevada ao quadrado (cm2, m2). É fácil entender por quê: no cálculo da área, multiplicamos sempre duas medidas lineares (lado por lado, diagonal por diagonal etc.). Então, as unidades também devem ser multiplicadas: m . m = m2

BASE MÉDIA DO TRAPÉZIO

Questões que envolvem trapézios caem regularmente no Enem e nos vestibulares. Vale a pena você anotar:

a c

A B

C

b Num trapézio C, chama-se base média a reta paralela às bases que une os pontos médios dos lados. Você deve guardar: • A base média de um trapézio define dois novos trapézios (A e B). • A base média tem comprimento igual à média aritmética das duas bases do trapézio. Em linguagem matemática, c = (a + b)/2 . • As alturas dos trapézios A e B são congruentes e medem metade da altura do trapézio C.

NA PRÁTICA RELAÇÃO ENTRE ÁREAS

Um modo simples e tradicional de calcular quantas pessoas existem numa multidão é fazer a relação entre áreas. Em uma manifestação popular, as pessoas ocupavam um trecho de 200 metros de uma avenida. A largura da avenida é de 9 metros. Sabendo que cada 2 metros quadrados eram ocupados por, em média, 6 pessoas, quantas pessoas participaram da manifestação? A questão pede a relação entre áreas. Desenhando a situação, temos:

2 m2 9m 200 m A área ocupada pela manifestação é um retângulo de 9 m de largura por 200 m de comprimento. Podemos considerar a largura como a altura do retângulo. A área de um retângulo é calculada por A = base . altura. Então, A = 9 . 200 A = 1 800 m2 Agora, é só verificar a proporção entre a área ocupada por uma pessoa e a área total. Simples regra de três: 6 pessoas – 2 m2 x pessoas – 1 800 m2 1 800 . 6 2 x = 5 400 pessoas x=

42 GE MATEMÁTICA 2017

Poucos lados, muitos usos O triângulo é o polígono mais simples, mas o mais versátil para diversos cálculos

T

rês é o número mínimo de lados de um polígono. Então o triângulo é o polígono mais simples. Mas as relações entre seus lados, seus ângulos e com outros polígonos tornam essa figura plana importantíssima. Tanto é que a matemática reserva uma área especialmente dedicada a ela, a trigonometria (veja o capítulo 5). Os triângulos são classificados em diferentes tipos, conforme o tamanho de seus lados (veja na página ao lado). Há, ainda, um triângulo muito especial, o triângulo retângulo, que tem um ângulo de 90o. Um triângulo retângulo pode ser isósceles ou escaleno, jamais equilátero.


ISTOCK

TRIÂNGULOS JUNINOS As tradicionais bandeiras de quermesse são triângulos isósceles, com dois lados iguais

Equilátero: os três lados iguais e os três ângulos iguais

sempre a metade da área do retângulo ou losango de medidas correspondentes. Veja ao lado. Área do triângulo retângulo: metade da área de um retângulo

Isósceles: dois lados iguais e dois ângulos iguais

Área do triângulo escaleno: metade da área de um losango

Escaleno: nenhum lado e nenhum ângulo igual

A fórmula geral para a área de um triângulo é: b.h , A= 2

em que b é o comprimento da base e h, a altura. Você não precisa decorar essa fórmula. É só perceber que um triângulo é exatamente a metade de um retângulo ou um losango. Portanto, sua área é

Área do triângulo isósceles: metade da área de um losango

A exceção fica para o triângulo equilátero, que tem todos os lados iguais. Nesse caso, a fórmula da área é: a2 . 3 , A= 4

O QUE ISSO TEM A VER COM A FÍSICA A área dos triângulos pode ser útil na análise de gráficos de movimentos retilíneos uniformemente variados (MRVU), aqueles no qual a velocidade varia de maneira constante ao longo do tempo. O gráfico abaixo mostra a variação da velocidade de um móvel em função do tempo. A área do triângulo retângulo formado entre a reta e o eixo x é igual ao deslocamento entre os instantes ti e tf. Aceleração > 0 Vf

A = deslocamento Vi O ti

tf

em que a é o comprimento de qualquer um dos lados. GE MATEMÁTICA 2017

43


2

GEOMETRIA POLÍGONOS

Triângulo retângulo

Os lados do triângulo retângulo recebem nomes especiais: • o lado maior é a hipotenusa;

NA PRÁTICA ÁREA DE TRIÂNGULOS

Uma piscina tem formato de um hexágono regular. A distância entre dois lados paralelos do hexágono é de 17 metros. Qual a área da piscina? (Considere 3 = 1, 7 ) Não é preciso conhecer a fórmula da área do hexágono. Basta raciocinar um pouco. Se traçarmos as diagonais do hexágono, unindo vértices opostos, encontramos seis triângulos equiláteros (três lados iguais). Veja:

• os dois lados menores são os catetos. São os catetos que formam o ângulo de 90o (ou ângulo reto). Veja:

Se os triângulos são equiláteros, então a medida de seus lados é igual à medida de cada um dos lados do hexágono. Conhecemos a distância entre dois lados opostos do hexágono: 17 m. Então a altura de cada um desses triângulos é 17 = 8,5 m.

2

A fórmula que relaciona a altura à medida dos lados do triângulo equilátero é h=

a.

2

3

b (cateto)

8, 5 =

a.

3 a = 8, 5 . 2 3 2

O enunciado informa que

C a (hipotenusa)

a

b A

c (cateto)

B

Este triângulo é retângulo em A – ou seja, o ângulo reto tem vértice em A. Então, chamamos a hipotenusa de a, e os catetos de b e c.

Teorema de Pitágoras

O filósofo e matemático grego Pitágoras desenvolveu um teorema que define a proporção entre as medidas dos lados de um triângulo retângulo. Em linguagem matemática, o teorema de Pitágoras diz que a2 = b2 + c2 Traduzindo: num triângulo retângulo, o quadrado da medida da hipotenusa é igual à soma dos quadrados das medidas dos catetos.

Temos então:

1. O quadrado da hipotenusa é igual à...

C

A h = 17 m

É fácil entender o que Pitágoras afirma. O quadrado de um número está diretamente relacionado à área do quadrado de lados com comprimento igual a esse número. Aplicando essa ideia, no desenho de um triângulo retângulo, temos:

c

B

2. ... soma do quadrado dos catetos

NA PRÁTICA TEOREMA DE PITÁGORAS

Um triângulo retângulo tem catetos medindo b = 3 e c = 4. Qual a medida da hipotenusa? Por Pitágoras, a2 = b2 + c2. Então a2 = 32 + 42 a2 = 9 + 16 a2 = 25 a=5

3 = 1, 7 .

Então, 17 a = 1, 7 " a = 10m

Esta é a medida de cada um dos lados. Aplicando a medida na fórmula da área do triângulo equilátero, temos: 2

a . 3 A= 4 A=

100 . 1, 7 4

A = 42,5 m2 A piscina é formada por seis triângulos. Portanto, a área total da piscina é A = 6 . 42,5 A = 255 m2

44 GE MATEMÁTICA 2017

TOME NOTA

TOME NOTA

RELAÇÃO ENTRE BASE E ALTURA

ÂNGULOS DE UM TRIÂNGULO

Repare que, para encontrar a área de um triângulo, precisamos conhecer a altura (h) e a medida dos lados. E nem sempre temos todas essas medidas. Nesse caso, a altura é estabelecida em função dos lados. No caso de um triângulo equilátero, a relação entre altura e lado é dada por a. 3 , h=

2

em que a é a medida de um lado.

Em todo triângulo, a soma dos ângulos internos é 180o.

b

90º + b+ H = 180º

H a = 90º


Pitágoras no plano cartesiano

O teorema de Pitágoras fornece a distância entre dois pontos no plano cartesiano. Veja: Para encontrar a distância entre os pontos P(-2, -1) e Q(2, 2):

TOME NOTA

NA PRÁTICA

DISTÂNCIA ENTRE PONTOS

PITÁGORAS NO TRIÂNGULO EQUILÁTERO

A figura abaixo representa a distância entre dois pontos quaisquer P1(x1, y1) e P2(x2, y2), num plano cartesiano: P2 (x2 , y2)

Localizando os pontos: 3 y

dvert = y2 - y1

Q

2

P1 (x1 , y1) dhoriz = x2 - x1

1 -2

-3

-1

0

x 0

1

3

2

-1

P

R

-2

Repare que temos um triângulo retângulo em R, ponto definido pelo par ordenado R (2,-1). A distância entre P e Q corresponde à hipotenusa. Conhecemos os pontos ordenados de P e Q. Assim, encontramos a distância entre R e cada um desses pontos, numa simples conta de subtração: • Para P e R: 2 – (–2) = 2 + 2 = 4 • Para Q e R: 2 – (–1) = 2 + 1 = 3 Sabendo a medida de dois catetos, aplicamos Pitágoras:

Repare que, na vertical, a distância é dada pelos valores no eixo y, e, na horizontal, pelos valores no eixo x.

d P1 P2 =

h

Q x 2 - x 1 V2 + Q y 2 - y 1 V2

x/2

2

NA PRÁTICA PITÁGORAS NO QUADRADO

Você não precisa decorar a fórmula para a diagonal de um triângulo. Basta raciocinar sobre o teorema de Pitágoras. Acompanhe abaixo. Qual a medida dos lados de um quadrado cuja diagonal mede 49 cm?

Aplicando o teorema de Pitágoras em cada um desses triângulos, temos: a2 = b2 + c2 Sabemos que a = x e b = x

2

Então, x 2 = h 2 + S 2 X x

d2 = a2 + a2 d2 = 2a2 d=

2a 2

d=a 2

a

x2 h2 = x2– 4

7=

2a

h=

2

7=a 2

3x 2 4 x 3 2

Esta é a fórmula que você viu na página ao lado.

7 a= 2

Considerando a = 7 / 1,4 a = 5 cm

4x 2 – x 2 4

3x 2 h2 = 4 h=

d2 = a2 + a2 49 = 2a2

2

x 2 h2 = x2–S 2 X

h2 =

O teorema de Pitágoras nos dá, também, a medida da diagonal de um quadrado. Veja:

x/2

Repare que construímos dois triângulos retângulos, cujas bases medem x

49

Pitágoras no quadrado

x

d P 1 P 22 = Q x 2 - x 1 V2 + Q y 2 - y 1 V2

Esta é a distância entre os pontos P e Q.

d

x

Aplicando Pitágoras, temos:

PQ2 = 32 + 42 PQ2 = 9 + 16 = 25 PQ = 5

a

O teorema também é útil para encontrar a altura de um triângulo equilátero. Veja que, nesse tipo de triângulo, o segmento de reta que indica a altura (h) corta a base exatamente em seu ponto médio, fazendo com ele ângulos de 90º:

2 ≈ 1,4, temos:

a GE MATEMÁTICA 2017

45


GEOMETRIA CÔNICAS

ISTOCK

2

RODOPIO ELÍPTICO No Sistema Solar, a órbita de todos os planetas em torno do Sol segue a forma de elipses, algumas mais, outras menos excêntricas

As figuras sem arestas Cônicas são curvas que nascem da intersecção de um plano com um cone

parábola

• Aumentando-se a inclinação, a elipse não mais se fecha, e a curva se transforma numa parábola. • Um plano perfeitamente na vertical cria uma hipérbole.

circunferência

Todas as cônicas podem ser representadas no plano cartesiano, por um par ordenado (x, y). A relação entre x e y é dada por uma equação. A forma mais comum é a equação reduzida, ou equação geral.

U

ma casquinha de sorvete é um cone, um sólido geométrico, ou seja, uma figura que tem três dimensões – altura, largura e espessura (veja mais a partir da página 52). Qualquer corte que você faça com um plano nas paredes do cone resulta numa figura plana – uma curva plana chamada cônica. Daí vem a definição de cônicas: são curvas obtidas da intersecção de um plano com um cone. São cônicas a circunferência, a elipse, a parábola e a hipérbole – cada uma delas com o formato definido pela inclinação em que o plano corta o cone (veja no quadro ao lado).

46 GE MATEMÁTICA 2017

elipse

hipérbole

Observe que a curva é alterada dependendo da inclinação do plano que corta o cone: • Um plano perfeitamente horizontal produz uma circunferência. • Um plano ligeiramente inclinado deforma a circunferência e cria uma elipse.

ATENÇÃO CIRCUNFERÊNCIA E CÍRCULO

A circunferência é uma curva. Não tem área. O que tem área é o círculo – a região no interior da circunferência. A área do círculo é dada por A = π . r2


Circunferência

É a curva formada por todos os pontos que estão a uma mesma distância de outro ponto – o centro da circunferência.

A letra grega π é um número irracional, que não pode ser escrito na forma de fração com numerador e denominador inteiros. Para efeito de cálculos, costumamos arredondar o valor de π para 3,14.

centro um ponto qualquer C (a,b), e que passa pelo ponto Q (x,y). A equação reduzida para esse tipo de circunferência, cujo centro não coincide com a origem do sistema cartesiano, é

Equações da circunferência

r2 = (x – a)2 + (y – b)2

Considere uma circunferência com o centro no ponto (0, 0):

C

Veja abaixo a figura:

y

r

y

Q (xQ, yQ)

yQ

x

d = 2r

O

r

xQ

• Diâmetro é o dobro do raio (2r); • O comprimento (ou perímetro) da circunferência é dado pela expressão: P=2.π.r

C

b

• C é o ponto que marca o centro; • A distância de qualquer ponto da circunferência a C é o raio (r);

Q

yQ

0

A posição de qualquer ponto Q de uma circunferência é dada por xQ2 + yQ2 = r2 Mas o centro da circunferência pode não coincidir com o ponto (0, 0). Considere uma circunferência que tem como

a

xQ

x

Repare que o cateto horizontal mede (xQ – a). E o cateto vertical, (yQ – b). Desenvolvendo r2 = (x – a)2 + (y – b)2, a equação geral das circunferências fica assim: r2 = x2 + y2 – 2ax – 2by + a2 + b2

NA PRÁTICA EQUAÇÕES DA CIRCUNFERÊNCIA

Na equação geral:

Simples aplicação da equação:

r2 = x2 + y2 – 2ax – 2by + a2 + b2 4 = x2 + y2 – 2 . 0x – 2 . (–5 y) + 0 + (–5)2 4 = x2 + y2 – 0x + 10y +25 x2 + y2 + 10y + 25 – 4 = 0 x2 + y2 + 10y + 21 = 0

Qual o raio da circunferência com centro no ponto C (0, –5) e que passa pelo ponto P (–2, –5)?

r2 = (x – a)2 + (y – b)2 r2 = [0 – (–2)]2 + [(–5 – (–5)]2 r2 = (0 + 2)2 + (–5 +5)2 r2 = 4 + 0 r= 4 r=2

Desenhe a circunferência no plano cartesiano. y

Defina as equações reduzida e geral dessa circunferência. Para encontrar as equações de uma circunferência, basta conhecer a medida do raio e as coordenadas do centro. Sabemos que o raio é 2 e que seu centro é C (0, -5). É só aplicar esses valores nas duas equações. Na reduzida: r2 = (x – a)2 + (y – b)2 4 = (x – 0)2 + (y – (–5))2

x

P (–2,–5)

C (0,–5)

GE MATEMÁTICA 2017

47


2

GEOMETRIA CÔNICAS

Inscrição e circunscrição

Circunferências inscritas em polígonos são aquelas dentro de um polígono, tocando todos seus lados. Circunferências circunscritas são aquelas que estão do lado de fora do polígono, passando por todos os seus vértices. Todos os polígonos regulares (de lados congruentes) podem ter circunferências inscritas e circunscritas. Os raios das circunferências se relacionam com as medidas dos polígonos. Veja:

Quadrado

a

NA PRÁTICA

r

POLÍGONOS INSCRITOS

r

Então: (2 . rc )2 = a2 + a2 4 . rc2 = 2 . a2 rc2 =

2 . a2 4 " rc =

2 2 . a2 4 " rc = a 2

Triângulo equilátero

A circunferência inscrita toca os três lados do triângulo. A circunscrita toca seus três vértices.

V

Um quadrado e um triângulo equilátero estão inscritos em uma circunferência. Calcule o comprimento dos lados do triângulo equilátero, sabendo que a área do quadrado é 100 cm2. Primeiro, preste atenção no enunciado, para não se confundir: se os polígonos estão inscritos, então a circunferência está do lado de fora, circunscrevendo os polígonos. A situação descrita é esta:

l

rc a

ri

a

M

C

rc

M Circunferência I

ri

C

M rc

l

C

Circunferência II

Na figura acima: • O polígono regular é um quadrado; • O quadrado está inscrito em uma circunferência e circunscreve outra, de raios ri e rc diferentes; • O ponto C é o centro do quadrado e das duas circunferências; • O ponto M é o ponto médio do lado do quadrado. Para a circunferência inscrita: • Ela toca o lado do quadrado no ponto médio de cada um dos lados do quadrado; • O raio ri é a distância do centro ao ponto médio do polígono (CM). Essa distância se chama apótema (segmento de reta que une o centro a um dos lados de um polígono regular, sempre perpendicular a ele). O apótema sempre une o centro ao ponto médio do lado. Então, sabemos que r i = a2 Para a circunferência circunscrita: • Ela toca os vértices do quadrado; • Encontramos rc aplicando o teorema de Pitágoras no triângulo formado por dois lados do quadrado e sua diagonal. E essa diagonal corresponde ao diâmetro da circunferência (2 . rc ). Veja a seguir:

48 GE MATEMÁTICA 2017

O raio da circunferência inscrita (ri ) equivale à distância do centro C ao ponto médio do lado do triângulo. Chegamos às medidas de ri e rc também por Pitágoras. Não é importante que você veja a demonstração. Pode apenas guardar as equações: ri =

l 3 6

e rc =

l 3 3

Basta observar as duas equações para concluir que rc = 2 . ri

Hexágono regular M

a

a 3 2

Como A = a2 a = 100 a = 10 cm O raio da circunferência que circunscreve um quadrado é dado por rc =

C rc

Para a circunferência circunscrita, rc = a

l 2 2

Então, r c =

10 2 2 = 5 2 cm

Esta circunferência circunscreve também o triângulo equilátero. A equação que relaciona o lado do triângulo com rc é: rc =

ri

Também neste caso, você não precisa da demonstração. Pode apenas guardar as equações: Para a circunferência inscrita, ri =

O enunciado informa que a área do quadrado é A = 100 cm2.

l 3 3

Temos, então, que l 3 = 5 2 3 Isolando l, obtemos: l=

15 6 15 2 = 3 = 5 6 cm 3


Elipse

Numa definição informal, elipse é uma circunferência deformada. Veja as relações entre os elementos de uma elipse:

Se chamarmos essa distância de a, temos que A1A2= 2 . a B1

B1

A1

A2

B1 A1

O

A1

F2

c

A2 O

F1 F1

O

F1

F2

B2

A2

2c B2

a B2

a

2a

O é o centro da elipse A1A2 é o eixo maior da elipse B1B2 é o eixo menor da elipse F1 e F2 são os focos da elipse

A distância entre F1 e F2 e o centro O é sempre igual e se chama (c).

A distância entre o ponto A 1 e o centro O é sempre igual à distância do ponto A 2 ao centro O.

F2

c

A distância entre o ponto B 1 e o centro é sempre igual à distância entre o ponto B2 e o centro. Então, a distância entre B1 e B2 = 2 . b B1

Como F1O = F2O, então a distância entre F1 e F2 = 2 . c (distância focal)

b 2b O

F1

A1

F2

A2

b

B2

As distâncias de um ponto qualquer da elipse a F1 e F2 , somadas, é um valor constante.

NA PRÁTICA Q x – m V2

EQUAÇÃO DA ELIPSE

a2

Escreva a equação reduzida desta elipse.

+

Q y – n V2 b2

Substituindo os valores de m e n, encontramos a equação da elipse dada:

8 y

Q x – 2 V2

7 6

42

5

+

Q y – 5 V2 32

3 9

2

-3 -2 -1

x 0 1

2

3

4

5

6

7

F1

A1

y

5

a

-3 -2 -1

b

3 2

x 0 1

2

3

4

5

6

7

8

B2

Repare que C não coincide com a origem do sistema (0,0). Nesse caso, temos de considerar a diferença entre as corodenadas. Chamando as coordenadas de C de m e n, temos:

x 0 1

2

3

4

5

6

7

Como a circunferência, uma elipse também pode ser desenhada num plano cartesiano, ou seja, sobre os eixos x e y:

8

y B1

Agora o eixo maior (2a) está na vertical (ou seja, sua medida é dada pelo eixo y). Então os denominadores da equação mudam de lugar:

Q x – m V2 b

2

+

Q y – n V2 a

2

= 1, sempre com a > b

Substituindo os valores, novamente, temos:

Qx – 2V

2

3

2

Qy – 5V

2

+

A2

A elipse no plano cartesiano

4

1 0

F2

dQ2

dP1 + dP2 = dQ1 + dQ2

2

C = (2, 5)

O

dQ1

Q

3

4

-3 -2 -1

dP2

6

7

1 0

dP1

7

8

8 y

5

P

8

Identificamos os principais pontos da elipse.

6

B1

=1

Se a elipse for vertical, a equação muda:

4

1 0

= 1,

42

=1

A1

F1

A1O = A2O = a B1O = B2O = b

O

F2

A2

x

B2

F1O = F2O = c GE MATEMÁTICA 2017

49


2

GEOMETRIA CÔNICAS

Se a elipse tiver o centro na origem do sistema cartesiano – ou seja, nas coordenadas (0,0) –, cada ponto da curva é definido pela equação:

Isso porque c é sempre maior ou igual a zero (c * 0) e sempre menor que a (c < a).

2 x2 y + = 1, sempre com a 2 b a2 b2

Para uma elipse cujo centro não coincide com a origem do sistema cartesiano, a equação reduzida leva em conta a distância entre os pontos da curva e o centro da elipse:

2 Q x – m V2 Q y – n V + = 1, sempre com a 2 b a2 b2

• x e y são as coordenadas de um ponto qualquer P; • m e n são as coordenadas do centro da elipse: O (m,n); • a é metade do eixo maior e b, metade do eixo menor. Essa equação descreve uma elipse cujo eixo maior está na horizontal. Para obter a elipse com eixo maior na vertical, basta trocar a e b de lugar na equação:

Q x – m V2 b2

+

Q y – n V2 a2

= 1, sempre com a 2 b

0

O

A2

F2

B2

Excentricidade

A excentricidade (e) de uma elipse é a razão entre a distância focal (2c) e o tamanho do eixo maior (2a). Em linguagem matemática: 2c c e = 2a = a

A excentricidade é sempre um número positivo, que varia de zero a um (0 ≤ e < 1).

50 GE MATEMÁTICA 2017

0

x1

1

x2

2

3 F

P d 1

d2

-1

O

• A reta azul é a diretriz • F é o foco da parábola • V é o vértice • Os valores x1 e x2 correspondem aos pontos em que a parábola corta o eixo x. • P é um ponto qualquer da parábola • As distâncias de P ao foco e de P à diretriz são iguais: d1 = d2. • O eixo de simetria é a reta que passa pelo foco e pelo vértice.

F

As parábolas são as curvas características dos gráficos das funções do 2º grau, ou seja, das funções cuja expressão tem a forma geral y = ax2 + bx + c. As relações entre os pontos de uma parábola você vê no capítulo 3.

F

Hipérbole

e = 0,8

F

A figura abaixo é uma hipérbole – uma curva de dois ramos, virados para lados opostos. 4

e = 0,95 O e = 0,999

x

diretriz

e = 0,5

O

4 V

e = 0,0

a A1

eixo de simetria

1

O=F

O

b

A parábola é a curva definida por pontos para os quais a distância de uma reta diretriz é igual à distância do ponto foco (F). Veja na figura: y

B1

F1

Parábola

y

3 2

F

1 F1 -4

-3

-2

P B1

A1 0 C -1 0 -1 -2 B2 -3 -4

A2 1

F2 2

3

x 4


SAIBA MAIS ÓRBITA ELÍPTICA

A órbita de asteroides e cometas em torno do Sol é elíptica, como a dos planetas. E a velocidade desses corpos varia conforme o ponto da elipse em que se encontram. Foi o alemão Johannes Kepler quem descobriu isso, no início do século XVII. Até então, imaginava-se que as trajetórias descritas pelos planetas, asteroides e cometas tivessem a forma de circunferência. Kepler não percebeu apenas que a órbita dos planetas é elíptica. Ele também notou que a velocidade de cada corpo celeste varia ao longo dessa trajetória. Kepler observou que o Sol não se encontra no centro da elipse, mas em um de seus focos. Assim, os planetas estão ora mais próximos, ora mais distantes dele. E, quanto mais próximo, mais rápido o planeta viaja. Essa variação de velocidade obedece à 2ª Lei de Kepler, ou Lei das Áreas. Segundo ela, “a linha que une um planeta ao Sol atravessa áreas iguais em intervalos iguais de tempo”. Veja abaixo o que significa a Lei das Áreas, de Kepler.

ISTOCK

Planeta no instante t1

Acompanhe na figura: • O ponto C é o centro da hipérbole. Neste caso, C coincide com a origem do sistema cartesiano: C = O = (0,0); • F1 e F2 são os focos da hipérbole; • A distância entre F1 e F2 é a distância focal da hipérbole e vale 2c; • Os pontos A1 e A2 são os vértices da hipérbole; • O segmento A1A2 é chamado eixo real da hipérbole, e vale 2a; • O segmento B1B2 é o eixo conjugado da hipérbole, e vale 2b; • Um ponto qualquer P da hipérbole tem distâncias d1 e d2 dos focos F1 e F2. A hipérbole é formada por pontos tais que a diferença entre as distâncias d1 e d2 é constante, em módulo. Quando o centro da hipérbole coincide com a origem do sistema cartesiano, a equação reduzida é 2 x2 y 2 – 2 =1

em que x é a coordenada do ponto P no eixo x, e y, a coordenada de P no eixo y.

LUZES PARABÓLICAS Os fogos de artifício fazem no céu curvas que lembram parábolas

Planeta no instante t2

1 Quando está

próximo do Sol, um planeta na posição 1 atinge a posição 2 num intervalo de tempo t2 – t1.

Sol

2 O mesmo Sol

Planeta no instante t4 Planeta no instante t3

Planeta no instante t1

Planeta no instante t2

planeta, quando está distante do Sol, viaja mais devagar. Assim, no intervalo de tempo t4 – t3 , igual a t2 – t1 , percorre uma distância menor.

3 No entanto, a

linha imaginária Planeta no que une o planeta instante t4 ao Sol atravessa, em intervalos de tempo iguais, áreas iguais. Planeta no instante t3

A maioria dos planetas segue órbita de excentricidade muito próxima de zero – ou seja, órbitas quase circulares. Mercúrio é o de órbita mais excêntrica, com excentricidade pouco maior que 0,2. Para a Terra, a excentricidade é de 0,017 e para Vênus, 0,007. Mas alguns corpos que circundam o Sol têm excentricidade muito maior. O cometa Halley, que passa por aqui a cada 75 anos, descreve uma curva com excentricidade de 0,97. GE MATEMÁTICA 2017

51


GEOMETRIA SÓLIDOS

ISTOCK

2

Geometria em 3D Área e volume dos principais sólidos – prismas, cones, cilindros e pirâmides

O

s polígonos, você lembra, são figuras de duas dimensões, relacionadas aos lados e à altura, que permitem calcular sua área. Já as figuras tridimensionais, os chamados sólidos geométricos, têm três dimensões: altura, comprimento e largura. Com essas medidas encontramos o volume de um prisma – ou seja, o espaço que ele encerra.

Tipos de sólidos

Os sólidos geométricos são divididos em duas grandes famílias: • poliedros, construídos com faces que têm formato de polígonos; • corpos redondos, que, se cortados, podem mostrar uma circunferência (ou um círculo) como secção.

Secção transversal

POLIEDRO Esta pirâmide tem quatro faces triangulares. E a base é um quadrado

52 GE MATEMÁTICA 2017

CORPO REDONDO Cortando um cone paralelamente à base obtemos um círculo

EDIFÍCIO SÓLIDO Prédios modernos, como este, da Biblioteca Central de Seattle, nos Estados Unidos, exploram a combinação de figuras sólidas, como prismas


Poliedros

Os poliedros têm, pelo menos, quatro faces. Os polígonos que formam os lados das faces do poliedro compartilham lados entre si, formando as arestas. Os pontos onde três ou mais arestas se encontram são os vértices do poliedro. Face Vértice Aresta

TOME NOTA

NA PRÁTICA

RELAÇÃO DE EULER Todo poliedro convexo obedece à relação de Euler, que define o número de vértices (V) e faces (F) em função do número de arestas (A): V+F=A+2

ÁREA DE PRISMA

Qual é a área total de um prisma regular, reto, de bases quadradas, com altura de 10 cm e aresta das bases de 3 cm? Se o prisma é regular, então os lados da base são iguais: as base são quadrados. Se o prisma é reto, então as faces laterais são perpendiculares às bases. Este é o prisma:

Os poliedros regulares são aqueles formados exclusivamente por polígonos regulares, ou seja, polígonos com lados iguais. Além disso, os poliedros regulares são formados por polígonos de mesmo formato e com o mesmo número de lados: só quadrados, só triângulos equiláteros ou só pentágonos regulares.

h = 10 cm

b = 3 cm

A área de um prisma regular e reto é A total = A bases + A faces Cada base é um quadrado. Então, A bases = a2

Tetraedro é uma pirâmide que tem como faces quatro triângulos equiláteros

O cubo é composto de seis quadrados

Cada face é um retângulo. Então, face = a . h

O octaedro é formado por oito triângulos equiláteros

Preste atenção: você tem de levar em conta que o prisma tem duas bases e quatro faces: Então, A total = 2 . 3 . 3 + 4 . 3 . 10 A total = 18 + 120 A total = 138 cm2

Prismas

São poliedros em que duas das faces, de formato idêntico, se encontram em planos paralelos. Se os polígonos das bases forem polígonos regulares, isto é, polígonos com todos os lados de mesma medida, então o prisma é chamado de prisma regular. Um prisma pode ser reto ou oblíquo:

Os prismas também podem ser classificados conforme o tipo de polígonos das bases:

PRISMA HEXAGONAL

PRISMA RETO As arestas laterais são perpendiculares ao plano da base

PRISMA OBLÍQUO As arestas laterais são inclinadas em relação ao plano da base

PRISMA TRIANGULAR

PRISMA PENTAGONAL

A área total de um prisma é a soma das áreas dos polígonos que formam as faces laterais e a área das bases. Atotal = Abase + Afaces As medidas das áreas laterais são sempre áreas de quadriláteros (polígonos de quatro lados). Se o prisma for reto, as faces laterais são retângulos.

TOME NOTA O cubo é o prisma regular e reto mais simples que existe. Um cubo tem seis lados, todos quadrados. GE MATEMÁTICA 2017

53


2

GEOMETRIA SÓLIDOS

Paralelepípedo reto-retângulo

Uma caixa de leite é um paralelepípedo reto-retângulo: um prisma reto de bases quadradas. É importante saber calcular a medida das diagonais desse tipo de paralelepípedo. Veja: D’

C’

A’

d

d = a2 + b2 + c2

D

C

a

Essa é a fórmula para encontrar a diagonal de um paralelepípedo retoretângulo de dimensões a, b e c.

b

f A

Sabemos que f 2 = a2 + b2. Então:

c

B’

• O segmento que une vértices em faces de planos paralelos (d) é a diagonal do paralelepípedo. Seu comprimento, em função das dimensões a, b e c do paralelepípedo, também pode ser determinado pela aplicação do teorema de Pitágoras, agora no triângulo DBD’: d2 = f2 + c2

B

• A face ABCD é um retângulo; • Esse retângulo tem uma diagonal f, chamada diagonal da face; • Encontramos a medida de f aplicando o teorema de Pitágoras sobre as medidas dos lados a e b desse retângulo: f2 = a2 + b2 " f = a2 + b

Pirâmides

Pirâmide é o sólido que tem uma única base, formada de um polígono qualquer, e faces laterais na forma de triângulos com um vértice em comum. Uma pirâmide também pode ser reta ou oblíqua.

PIRÂMIDE HEXAGONAL RETA

PIRÂMIDE HEXAGONAL OBLÍQUA

Pirâmides que têm como base um polígono regular (de lados de mesma medida) são chamadas pirâmides regulares. Se a pirâmide for regular e reta, então os triângulos das faces laterais serão isósceles, ou equiláteros (no caso de triângulos de lados de mesma medida). Como ocorre com qualquer sólido, o cálculo da área total de uma pirâmide é a soma das áreas da base e de cada face, no caso, cada triângulo: A total = A bases + A faces

E os dois lados são os pontos CM e MV. Veja:

NA PRÁTICA

V

ÁREA DE PIRÂMIDE

Calcule a área total de uma pirâmide reta, de base quadrada, com 8 cm de aresta da base e 3 cm de altura. Desenhando a pirâmide: C M h = 3 cm

Para o triângulo CMV, sabemos que CV = hpirâmide = 3 cm Repare que o lado CM do triângulo vale metade de um lado da base: 8 CM = 2 = 4 cm

lado da base = 8 cm

Atotal = Abase + Afaces A base é um quadrado de 8 cm de lado. Então, Abase = 8 . 8 = 64 cm2 As faces são quatro triângulos isósceles. Lembrando a fórmula da área de triângulos, temos: A triângulo =

base . altura 2

Sabemos que a base mede 8 cm (a mesma medida de um lado da base). Mas atenção: a altura da pirâmide não é a altura dos triângulos (que estão inclinados). Calculamos essa altura com um truque: criamos um novo triângulo no qual um dos lados é o eixo central da pirâmide, do centro da base até o vértice (CV).

54 GE MATEMÁTICA 2017

Repare, também, que o triângulo CMV é um triângulo retângulo. Então podemos aplicar o teorema de Pitágoras e descobrir a medida do lado VM, a hipotenusa desse triângulo: VM2 = CV2 + CM2 � VM2 = 32 + 42 = 9 + 16 = 25 cm VM =

25 " VM = 5 cm

Esta, sim, é a altura de cada triângulo das faces da pirâmide. Agora podemos calcular a área dos triângulos isósceles, pela fórmula da área de qualquer triângulo: 8.5 A triângulo = base . altura = 2 = 20 cm 2 2

Como a pirâmide tem quatro triângulos nas faces, a área das faces soma 4 . 20 = 80 cm2 Voltando ao início da resolução, a área total da superfície da pirâmide é a soma da área da base e as áreas das faces: Atotal = Abase + Afaces � Atotal = 64 + 80 = 144 cm2


Cilindro

Numa definição informal, cilindro Ê o sólido geomÊtrico com duas bases paralelas e circulares.

NA PRĂ TICA Ă REA DO CILINDRO

Uma indĂşstria lança um produto que serĂĄ vendido em latas com formato de cilindro reto, com raio da base de 3 cm e altura de 5 cm. O rĂłtulo de papel deve cobrir toda a lateral do cilindro. Qual o formato do rĂłtulo, suas dimensĂľes e sua ĂĄrea? • Lembrando da planificação de um cilindro, sabemos que a lateral corresponde a um retângulo; • Esse retângulo tem um lado igual Ă altura do cilindro (5 cm) e outro lado igual ao comprimento da circunferĂŞncia da base do cilindro. A circunferĂŞncia da base ĂŠ dada por P = 2Ď€ . r. EntĂŁo, o outro lado do retângulo mede 2 . Ď€ . 3 = 6Ď€

O’

β h

Eixo

Îą

O

Cilindros cujas paredes laterais sĂŁo perpendiculares Ă base sĂŁo chamados cilindros retos. Caso contrĂĄrio, temos um cilindro oblĂ­quo.

h

• A ĂĄrea do retângulo ĂŠ dada por Aretângulo = base . altura. EntĂŁo, A = 6Ď€ . 5 = 30Ď€ cm2. Considerando Ď€ = 3,14, a ĂĄrea do rĂłtulo ĂŠ 30 . 3,14 = 94, 2 cm2

h = 2r

2r

CILINDRO RETO 2r

CILINDRO OBLĂ?QUO

Um plano pode cortar um cilindro de diferentes maneiras. Se o corte se der por um plano paralelo às bases, a chamada secção transversal serå um círculo. Secção transversal

Sabemos que Atotal = Abases + Alateral

Secção meridiana

CILINDRO EQUILĂ TERO

h

Repare que o cilindro Ê formado pela associação de dois círculos e um retângulo.

• A ĂĄrea de cada base ĂŠ a ĂĄrea do cĂ­rculo de raio r: A = Ď€ . r2. Como existem Como nos demais sĂłlidos, a ĂĄrea total duas bases, de um cilindro ĂŠ a soma das ĂĄreas das Abases = 2 . Ď€ . r2 bases e da lateral. Fica fĂĄcil entender como se calcula essa ĂĄrea com a plani- • A ĂĄrea lateral ĂŠ a ĂĄrea de um retânficação do cilindro. Planificar significa gulo no caso de um cilindro reto (ou “abrirâ€? o sĂłlido em suas componentes de um paralelogramo, no caso de um planas. Veja o formato que um cilindro cilindro oblĂ­quo). planificado tem: Um dos lados do retângulo corresponde Ă altura do cilindro: h. A medida do outro lado ĂŠ exatamente r a circunferĂŞncia do cĂ­rculo da base. h Lembrando que o comprimento de um cĂ­rculo ĂŠ dado por C = 2 . Ď€ . r. EntĂŁo, a ĂĄrea lateral ĂŠ dada por Alateral = 2 . Ď€ . r . h EntĂŁo, a ĂĄrea total de um cilindro ĂŠ:

Se o plano for perpendicular às bases e pasar pelo eixo do cilindro (secção meridiana), a secção Ê sempre um paralelogramo. Se o cilindro for reto, a secção meridiana terå a forma de um retângulo.

r 2đ?›‘r

Atotal = Abases + Alateral Atotal = 2 . π . r2 + 2 . π . r . h Atotal = 2 . π . r . (r + h)

h

GE MATEMĂ TICA 2017

55


2

GEOMETRIA SĂ“LIDOS

Cone

Existem dois tipos de cones: o cone reto tem o eixo perpendicular Ă base. Quando o eixo ĂŠ inclinado, temos um cone oblĂ­quo. V

V

h

Eixo

Eixo

O

O

Volume de sĂłlidos

O volume de prismas e cilindros Ê dado por V = Abase . h O volume de pirâmides e cones Ê uma fração do volume de prismas e cilindro (veja ao lado). Dois sólidos têm volumes equivalentes desde que tenham mesma altura e formato e têm volumes iguais se as bases tiverem a mesma årea. O volume de um cone reto Ê igual ao de um cone oblíquo, desde que sua altura seja a mesma e sua base tenha a mesma årea. O mesmo vale entre cilindros retos e oblíquos e prismas retos e oblíquos. Fica mais fåcil entender vendo a figura:

Quando um cone reto Ê cortado por um plano paralelo à sua base (secção transversal), a visão que se tem Ê a de um círculo.

h

Secção transversal

h

A figura abaixo mostra um cone planificado. g

g 2đ?›‘r

r r

Têm volumes iguais, tambÊm, dois sólidos de diferentes formatos, desde que eles apresentem a mesma altura, a mesma årea na base e, tambÊm, a mesma årea numa secção transversal cortada à mesma altura. Este Ê o princípio de Cavalieri. Veja:

A base do cone ĂŠ um cĂ­rculo cuja circunferĂŞncia mede 2Ď€r. Assim, a ĂĄrea da base ĂŠ a ĂĄrea de um cĂ­rculo:

h

Abase = Ď€ . r2 Na lateral, a letra g representa um segmento de reta qualquer que liga um ponto da circunferĂŞncia da base do cone a seu vĂŠrtice. A superfĂ­cie lateral do cone, planificada, ĂŠ um setor circular – uma parte de um cĂ­rculo, como um pedaço de pizza, de raio g. O setor circular tem comprimento igual Ă circunferĂŞncia da base (2Ď€r). A ĂĄrea lateral do cone ĂŠ dada por A lateral = rrg

E a ĂĄrea total ĂŠ a soma das duas ĂĄreas: A total = rrg + rr = rr Q g + r V 2

Ab1

Ab2

Se Ab1 = Ab2 , entĂŁo V1 = V2

h As1 Ab1

As2 Ab2

Se Ab1= Ab2 e As1= As2 , entĂŁo V1 = V2

56 GE MATEMĂ TICA 2017

VOLUME DA PIRĂ‚MIDE E DO CONE Uma pirâmide tem um terço do volume de um prisma com a mesma base e mesma altura. Portanto, o volume da pirâmide ĂŠ dado por 1 V = 3 . A base . h

A mesma relação ĂŠ vĂĄlida entre o cone e o cilindro: V = 1/3 . Ď€ . r2 . h

CONE OBLĂ?QUO

CONE RETO

TOME NOTA


NA PRÁTICA

NA PRÁTICA

ÁREA E VOLUME DE PRISMA

EQUIVALÊNCIA DE VOLUMES

Uma caixa de 1 litro é um prisma reto, de bases quadradas e faces laterais retangulares. Sabendo que as bases têm lado igual a 5 cm, quais as dimensões de cada retângulo das faces laterais? Este é o formato da caixa:

h

Dois sólidos geométricos, um prisma de base hexagonal e um cilindro, têm o mesmo volume. Além disso, ambos têm a mesma altura. Se a aresta da base do prisma mede 10 cm, qual é a medida do raio da base do cilindro? Considere 3 . 1, 7 e r . 3, 14 . Sabemos que dois sólidos têm volume igual desde que suas alturas e bases sejam iguais. O hexágono (base do prisma) tem área equivalente a 6 triângulos equiláteros de lados 10 cm. Sua área, portanto, é:

A hexágono = 6.

10 2 3 4

= 150 3 = 255 cm 2

Se o círculo (base do cilindro) tem essa área, é fácil encontrar o tamanho do raio: 255 = π . r2 r 2 , 81 r , 9 cm

Se as bases são quadradas, a área de cada uma delas é a área de um quadrado: Abase = 5 . 5 Abase = 25 cm2 O volume, sabemos, é 1 litro. Convertendo litro para cm3: 1 L = 1 000 cm3 Substituindo esses valores na fórmula para volume, temos: V = Abase . h 1 000 = 25 . h h = 1 000 / 25 h = 40 cm

NA PRÁTICA VOLUME DA PIRÂMIDE

Calcule o volume de uma pirâmide regular, de base hexagonal, com arestas da base medindo 5 cm e altura 10 cm. Primeiro, vamos calcular a área do hexágono da base. Um hexágono regular é formado por seis triângulos de lados iguais. Neste caso, o lado de cada triângulo vale 5 cm. Sabemos também que a área do triângulo equilátero em função do lado é dada por: A triângulo equilátero =

Então, os retângulos que formam as laterais têm 40 cm de altura por 5 cm de largura.

l2 3 4

Então, A triângulo equilátero =

52 3 25 3 2 = 4 4 cm

E qual a área superficial total do prisma? Basta somar as áreas de cada polígono das bases e das laterais:

Se o hexágono tem seis triângulos, então,

Para a área de cada uma das bases quadradas: A=a.a A=5.5 A = 25 São duas bases, então a área total das bases é 50 cm2.

O volume da pirâmide é um terço do volume de um prisma de mesma base e mesma altura:

A hexágono = 6.

25 3 75 3 2 4 = 2 cm

1 V = 3 . A base . h

Portanto, Para a área lateral: A = base . altura = 5 . 40 = 200 cm2

1 75 3 V pirâmide = 3 . 2 . 10 = 125 3 cm 3

Se a base é quadrada, então só podem existir quatro retângulos. A área total deles é: 4 . 200 = 800 cm2 Por fim, somando a área das bases e das laterais, temos: A total = 50 + 800 = 850 cm2 GE MATEMÁTICA 2017

57


2

COMO CAI NA PROVA

1. (UPE 2016) A Pizzaria Italiana vende pizzas inteiras ou em porções (fatias). A tabela abaixo apresenta o número de fatias e o diâmetro, de acordo com o tipo da pizza. Tipo da Pizza

Número de Fatias

Diâmetro (cm)

Broto

6

30

Grande

8

35

Gigante

10

40

O marcador de combustível é dividido em cinco partes. Na coluna Início, vemos que o tanque continha 4/5 do volume do cilindro. Ao final, havia apenas 1/5 do volume. Portanto, o consumo na viagem foi de 3/5 do tanque. Se conhecemos a capacidade total do tanque, calculamos o volume de combustível consumido na viagem: 3 . 0,405 = 0,243m3 de óleo diesel. 5 Pela conversão de unidades, sabemos que 1 m³ = 1 000 litros. Então os 0,243 m3 consumidos durante a viagem equivalem a 243 litros. Considerando que o caminhão percorre, em média, 3 km com 1 litro de diesel, então o caminhão percorreu 3 km . 243 L = 729 km. Resposta: D

Se uma pizza Broto inteira custa R$ 27,00 qual deve ser o preço de cada fatia da pizza Gigante? a) R$ 6,50 b) R$ 4,80 c) R$ 4,50 d) R$ 3,90 e) R$ 3,50

RESOLUÇÃO Aplicando a fórmula para a área do círculo, temos • Área da pizza Broto (30 cm de diâmetro e, portanto, 15 cm de raio): Abroto = π . r2 = π . 152 = 225 π cm2 • Área da pizza Gigante (40 cm de diâmetro e, portanto, raio de 20 cm): Agigante = π . 202 = 400 π cm2 Montando uma regra de três com o valor da pizza Broto, descobrimos o preço da pizza Gigante: 225 . π → R$ 27,00 400 . π → x x = 400 . π . 27,00 225 . π Simplificando a fração (eliminando π), ficamos com x = 10 800 → x = R$ 48,00 225 Este é o preço da pizza Gigante, inteira. Se essa pizza tem dez fatias, então o preço de cada uma é R$ 4,80. Resposta: B

2. (UPE 2016) A figura abaixo representa um tanque de combustível de certa marca de caminhão a diesel. Sabendo que esse veículo faz, em média, 3 km/L e observando o marcador de combustível no início e no final de uma viagem, quantos quilômetros esse caminhão percorreu? Considere π = 3.

3. (Ulbra 2016) As retas 2x – y – 4 = 0 e 2x + 3y – 12 = 0 interceptam-se no centro de uma circunferência de raio igual a 3. Então podemos dizer que a) a circunferência possui centro no ponto (2,3) b) a circunferência corta o eixo y em dois pontos c) a circunferência corta o eixo x em um ponto d) a circunferência é tangente ao eixo x e) a circunferência é tangente ao eixo y

RESOLUÇÃO Vamos determinar o centro da circunferência, que é o ponto de intersecção das duas retas. As equações dessas retas foram apresentadas na forma geral. Isolando a variável y em cada uma das expressões, obtemos a forma reduzida, y = ax + b. Acompanhe: 2 x – y – 4 = 0 → y = 2 x – 4 (I) 2 x + 3 y – 12 = 0 → 3 y = – 2 x + 12 → y = – 2 x + 4 (II) 3 Para identificar o ponto de intersecção da reta, igualamos as equações I e II: 2x–4=– 2 x+4 3 2x+ 2 x=4+4 3 8x = 8 → x = 3 3 Esta é a coordenada x do ponto do centro da circunferência. Substituindo x = 3 em (I), obtemos a coordenada y: y=2.3–4=6–4=2 Portanto, o centro da circunferência é C (3,2). Localizando o ponto C (3,2) no plano cartesiano e considerando que o raio da circunferência é 3, temos a seguinte situação: 5

y

4 3

C (3,2)

2 1

a) 243 km

b) 425 km

c) 648 km

d) 729 km

RESOLUÇÃO O tanque é um cilindro, com raio da base 0,3 m e altura 1,5 m. Para saber o volume do tanque, temos V = π . r² . h = π . (0,3)² . 1,5 = 0,135 . π = 0,405 m³

58 GE MATEMÁTICA 2017

e) 813 km

0

x 0

1

2

3

4

5

6

7

–1

De todas as alternativas, a única verdadeira é a que afirma que a circunferência tangencia o eixo y. Resposta: E


RESUMO

4. (UFPR 2016) Temos, abaixo, a planificação de uma pirâmide de base quadrada, cujas faces laterais são triângulos equiláteros. Qual é o volume dessa pirâmide?

Geometria ÁREAS • Retângulo: A = base . altura • Trapézio:

4 cm

A=

Q base maior + base menor V .h 2

• Triângulo: A =

a) 16 √3 cm3 3

b) 16√3 cm

c) 32 cm

3

3

d) 32 √2 cm3 3

e) 64 cm3 3

a .h 2

• Círculo: A = π . r2

RESOLUÇÃO

SÓLIDOS • Relação de Euler: V + F = A + 2 (V = vértices, F = faces e A = arestas)

Para calcular o volume da pirâmide regular, precisamos determinar a área da base e sua altura. Se a pirâmide é regular, então os triângulos que constituem suas faces são equiláteros. Veja a figura abaixo:

• Volume: • Prisma e cilindros : V = ABASE . h • Pirâmides e cone: V = ABASE . h / 3

H

h

• Equivalência de volume: Têm volumes iguais sólidos diferentes, desde que tenham a mesma área de base, altura e área de secção transversal. TEOREMA DE PITÁGORAS: No triângulo retângulo, a2 = b2 + c2

r

EQUAÇÃO DA CIRCUNFERÊNCIA r2 = (x – a)2 + (y – b)2

Repare que temos um triângulo retângulo formado por: • um segmento H, que corresponde à altura da pirâmide; • um segmento r, de medida igual à metade do lado do quadrado. Sabemos que o lado desse quadrado mede 4. Então, r = 2; • um segmento h, que corresponde à altura do triângulo equilátero da face da pirâmide. Na figura você percebe, também, que os lados do triângulo equilátero que constitui cada face medem 4 (mesma medida do lado do quadrado da base). Então, pela fórmula que define a altura de um triângulo equilátero, descobrimos o valor de h: h = 4√3 = 2√3 cm 2 Com o valor de r e de h, basta aplicar o teorema de Pitágoras ao triângulo retângulo para descobrir a medida de H: h² = H² + r² (2 √3)2 = H2 + 22 H² = 12 – 4 = 8 H = 2 √2 cm

EQUAÇÃO DA ELIPSE • Eixo principal na horizontal:

Q x – m V2 Q y – n V2 + = 1, sempre com a 2 b a2 b2

• Eixo principal na vertical:

Q x – m V2 b2

+

Q y – n V2 a2

= 1, sempre com a 2 b

• Quando o centro coincide com a origem do sistema: 2 x2 y + = 1 , sempre com a 2 b a2 b2

EXCENTRICIDADE

2c c e = 2a = a EQUAÇÃO DA HIBÉRBOLE

A área da base, um quadrado de lados 4 cm, é A = 16 cm² Pela fórmula do volume da pirâmide, temos Vpirâmide = 1 . A . H = 2√2 . 16 = 32 √2 cm3 3 3 3

2 x2 y – 2 2 =1

Resposta: D GE MATEMÁTICA 2017

59


3

ÁLGEBRA CONTEÚDO DESTE CAPÍTULO

Função e equação de 1º grau .......................................................................62 Posições relativas de retas ...........................................................................68 Função e equação de 2º grau .......................................................................70 Como cai na prova + Resumo .......................................................................76

A juventude e o desemprego O Brasil vive um período de bônus demográfico, bom para impulsionar o crescimento econômico. Mas os jovens ainda têm poucas oportunidades no mercado de trabalho

N

o mundo todo, o desemprego é um importante indicador de que a economia de um país não vai bem. Assim que a economia emperra, o consumo e as exportações caem, as empresas param de investir, as contratações são interrompidas e – mais grave – começam as demissões. Depois, quando a economia melhora, o cenário para os trabalhadores é o último fator a se recuperar. E os jovens são as principais vítimas de qualquer abalo no mercado de trabalho. Em 2015, cerca de 12% da população com idade entre 18 e 24 anos não encontrava trabalho. Em março de 2016, com o agravamento da crise econômica, esse índice já superava os 15%. Na população com idade entre 14 e 24 anos a situação é ainda mais grave. Segundo o Instituto de Pesquisa Econômica Aplicada (Ipea), no primeiro trimestre de 2016 mais de 25% dos jovens dessa faixa etária estavam desempregados (veja o quadro Saiu na imprensa, pág. 67). A exclusão dos jovens – muitos deles sem qualificação – do mercado de trabalho é um sinal de alerta para o Brasil: o país está perdendo a chance de aproveitar o bônus demográfico. Bônus demográfico é um período em que o número de jovens e adultos em idade de trabalhar cresce em relação à parcela que depende desse trabalho – ou seja, quando o número de jovens e adultos é maior que o de crianças e idosos. O

60 GE MATEMÁTICA 2017

bônus é transitório. No Brasil, a proporção entre os dependentes e a PIA – a chamada razão de dependência – começará a aumentar a partir dos anos 2020. Isso significa que, daqui a quatro anos, o número de dependentes passará a crescer mais rapidamente do que o de pessoas que possam sustentá-los. Para aproveitar essa brecha de oportunidade, é preciso dar a crianças e jovens condições de participar da vida econômica do país, com políticas públicas de educação, saúde e geração de emprego. O aumento da razão de dependência é resultado da combinação, de um lado, da queda na taxa de fertilidade (nascem cada vez menos crianças) e, do outro lado, do aumento da longevidade da população (existem cada vez mais idosos). Em 2013, os idosos representavam pouco mais de 7% da população brasileira. Em 2060, eles representarão mais de 25%. No Brasil, a razão de dependên- JOVEM PROCURA cia cai desde 2007 Pessoas com idade entre segundo uma função 18 e 24 anos têm grande de primeiro grau (veja dificuldade em conseguir o gráfico na pág. 63). trabalho. No início de Função de primeiro 2016, no Brasil, a taxa de grau é um dos temas desocupação nessa faixa deste capítulo. etária era de cerca de 15%


GE MATEMÁTICA 2017

61

ZANONE FRAISSAT/FOLHAPRESS


ÁLGEBRA FUNÇÃO E EQUAÇÃO DE 1º GRAU

ANTONIO CICERO / FOTOARENA

3

A construção de uma reta Em um gráfico, cada ponto de uma reta é dado por um par ordenado, definido por uma função de 1º grau

62 GE MATEMÁTICA 2017

T

odo gráfico de linha é composto de segmentos de retas desenhados no plano cartesiano, em que cada ponto é definido por um par ordenado (x , y) – ou seja, a cada valor de x, no eixo horizontal (das abscissas) existe um único valor y, no eixo vertical (das ordenadas). Em linguagem matemática, dizemos então que y é uma função de x. Acompanhe no gráfico da página ao lado, que mostra a razão de dependência no Brasil. • Repare que entre 1970 e 2020, a reta é decrescente. Mas, a partir de então, a reta sobe. Isso ocorre porque, até 2020, os valores de y caíram e, desse ano em diante, esses valores sobem. • As retas pontilhadas em vermelho são construídas unindo apenas os pontos extremos de cada um desses dois períodos: 1970 a 2020, e 2020 a 2050.

• Repare que as retas pontilhadas não passam exatamente sobre todos pontos originais do gráfico. Então, seus valores, para cada década, não estão marcados de maneira precisa. Mas essa reta tracejada mostra a variação da dependência, entre os anos extremos – de 1970 a 2020 e, depois, de 2020 a 2050. Calculando a taxa de variação: • A razão de dependência é y, a variável; • y varia em função do tempo (década a década). Chamamos o tempo de x. Para cada valor de x temos um único valor de y; • Escolhemos pontos em que a reta coincide com os pontos originais da reta: 1970, 2020 e 2050. E, com isso, montamos a tabela com os valores correspondentes:


Esta é a queda da razão de dependência entre 1970 e 2020. Fazendo o mesmo raciocínio para o período seguinte, entre 2020 e 2050, temos: • 6x = 3 décadas. • 6y = 75,1 – 50,9 = 24,2 • Taxa de variação da dependência no período: Dy 24, 2 = 3 .8 Dx

CADA VEZ MAIS VELHA Os gráficos que mostram a proporção de jovens e idosos na população brasileira são a representação de funções de 1º grau x 1970 2020 2050

y 89,3% 50,9% 75,1%

Calculando a taxa de variação no primeiro trecho (1970-2020): A variação de x (6x) é o número de décadas entre os dois valores: 6x = 5. • y variou de 89,3 a 50,9. Então, 6y = 50,9 – 89,3 • 6y = - 38,4 • Repare que 6 y é um valor negativo – o que está correto, já que a razão de dependência caiu nesse período. •A taxa de variação entre 1970 e 2020 é a divisão de 6y por 6x: Dy - 38, 4 = 5 . - 7, 7 Dx

Interpretando os valores: • Entre 2020 e 2050, a razão de dependência deverá aumentar num ritmo ligeiramente maior que a queda registrada entre 1970 e 2020: crescimento de 8% e queda de 7%. • Ao fazer o cálculo para as taxas de variação, descobrimos a função geral que define a relação entre y e x para cada período: 6y (1970-2020) = -7 . x 6y (1970-2020) = 8 . x

As expressões de 6y que acabamos de encontrar são funções do 1º grau. Esse tipo de função tem sempre a forma f(x) = a . x + b Repare que f(x) é outra maneira de representar a variável y. O nome “função de 1o grau” vem do fato de que x é elevado à primeira potência (elevado a 1). O valor de b é o chamado coeficiente linear da reta.

Gráfico da função de 1º grau

Toda função de 1o grau tem como gráfico uma reta. Mas cada função define uma reta específica, com características próprias. Essas características dependem da relação estabelecida entre x e y, que, por sua vez, têm relação com o coeficiente angular e o coeficiente linear. Acompanhe no exemplo a seguir: Considere a função f(x) = 2 . x – 1. Para construir o gráfico dessa função, começamos montando uma tabela que atribui valores aleatórios a x e o valor correspondente de y.

RAZÃO DE DEPENDÊNCIA NO BRASIL (EM %) 95,00

89,3 75,1

76,00

50,9

57,00 38,00 19,00 0,00

1970

1980

1990

2000

2010

2020

2030

2040

2050

CADA VEZ MAIS DEPENDENTES A linha em verde mostra a razão de dependência – a proporção entre idosos e crianças (dependentes) e as pessoas em idade de trabalhar (que sustentam os dependentes), entre 1970 e 2050. Veja que a razão cai entre 1970 e 2020, para então começar a subir. Cada um dos pontos das linhas verdes representa um valor exato da razão na década correspondente. Já as retas pontilhadas não fornecem os valores exatos da razão em cada década, mas dão uma boa ideia da taxa de variação dessa razão nos dois períodos. Fonte: IBGE

GE MATEMÁTICA 2017

63


3

ÁLGEBRA FUNÇÃO E EQUAÇÃO DE 1º GRAU

x vale

Na função y=2.x–1

y valerá

-2

y = 2 . (-2) – 1

-5

-1

y = 2 . (-1) – 1

-3

0

y=2.0–1

-1

1

y=2.1–1

1

2

y=2.2–1

3

5 z 4

)3-!4*

3 2

)2-!2* .5!

.4!

.3!

1 ! .2

.2!

2!

3!

y 4!

5

)1-!.2* .3

).2-!.4*

Definimos, assim, os pares ordenados desses cinco pontos:

.4 .5

(-2, -5)

.6

2

(1, 1) (2, 3)

Ao lado você vê como esses pares ordenados se organizam no gráfico.

z!>!3!/!y!.!2

.7

(-1, -3) (0, -1)

Coeficiente linear

3

).3-!.6*

Veja que os cinco pontos interligados formam a reta da função y = 2 . x – 1. Observe que: • Todos os pontos estão alinhados, formando uma reta. Na verdade, para definir qualquer reta, precisamos de apenas dois pontos. Então

NA PRÁTICA GRÁFICO DA FUNÇÃO DE 1º GRAU Considere outra função: f(x) = – 3 . x + 4. A tabela para a construção do gráfico no plano cartesiano é: Se x vale

na função y = -3 . x + 4

então, y valerá

(x,y)

-2

y = -3 . (-2) + 4

10

(- 2, 10)

-1

y = -3 . (-1) + 4

7

(- 1, 7)

0

y = -3 . 0 + 4

4

(0, 4)

1

y = -3 . 1 + 4

1

(1, 1)

2

y = -3 . 2 + 4

-2

64 GE MATEMÁTICA 2017

Na função y = a . x + b, a letra b é o coeficiente linear da reta. Este é o valor de y quando x = 0. Ou seja, é o valor de y no ponto em que a reta cruza o eixo y. • Na função com que trabalhamos, y = 2 . x – 1, b vale -1 E no gráfico você confirma: a reta cruza o eixo x em y = -1; • No quadro Na prática, ao lado, no gráfico da função y = -3 . x + 4, b = 4. Confirme: a reta cruza o eixo y em y = 4.

Coeficiente angular Novamente, com cada uma dessas coordenadas cartesianas (x, y), localizamos os pontos e traçamos a reta:

a = Dx

:

.4

Na função y = 2 . x – 1, a = 2

9

Dy

7

Na função y = –3 . x + 4, a = –3

6

a=

5 )1-!5* 4 3

)2-!2*

2 .5!

.4!

.3!

.2!

1

y !

2!

3!

4!

.2 .3 .4

z!>!.4!/!y!,!5

2

a = Dx = 1 = 2

8

).2-!8*

Nas funções dos dois exemplos citados acima, o coeficiente a é a razão da variação de y e x – ou seja, quanto y varia a cada variação de x. Dy

21 z

2

).3-!21*

(2, – 2)

Observe que: • Desta vez, ao passarmos de um ponto a outro da tabela – por exemplo, de (– 2, 10) a (– 1, 7) –, andamos uma unidade para a direita no sentido horizontal (do eixo x) e 3 unidades para baixo no sentido vertical (do eixo y). Ou seja, para ∆x = 1, ∆y = – 3. Essa relação é válida para quaisquer pontos da reta; • No ponto em que a reta cruza o eixo y, a coordenada x tem valor 0 e a y, 4.

poderíamos ter calculado apenas os valores de y para dois valores diferentes de x; • Quando passamos de um ponto a outro, deslocamos uma unidade na horizontal e duas unidades na vertical. Ou seja: para 6x = 1, 6y = 2. Essa relação é válida para quaisquer pontos da reta; • No ponto em que a reta cruza o eixo y, a coordenada x vale 0 e a coordenada y vale -1.

.5

)3-!.3*

5

Dy =-3 Dx

O coeficiente a é o coeficiente angular da reta. Ele define a inclinação ou declividade da reta. Para encontrar o coeficiente angular de uma reta, precisamos apenas das coordenadas de dois pontos (xA, yA) e (xB, yB) e estabelecer a razão entre suas diferenças: a=

Dy Q y A –y B V Q y B –y A V = = Dx Q x A –x B V Q x B –x A V


As funçþes do 1º grau com coeficiente angular a > 0 são crescentes e suas retas formam ângulo entre 0° e 90° com o lado positivo do eixo x. y

0º≤ � ≤ 90º

Cruzamento da reta com os eixos

Uma reta inclinada – ou seja, que não seja nem horizontal nem vertical – sempre cruza tanto o eixo x quanto o eixo y. O cruzamento no eixo y se då no ponto em que x = 0 e terå coordenadas (0, b) – ou seja, x = 0 e b Ê o coeficiente linear da reta. O cruzamento no eixo x ocorre no ponto de coordenada (x,0). Veja abaixo:

x

(0,0)

y

Jå as funçþes de 1º grau com coeficiente angular a < 0 são decrescentes e suas retas formam ângulo entre 90° e 180° com o lado positivo do eixo x.

Faça a anĂĄlise de sinal da função y = 3x – 9 Para encontrar o valor de x para o qual y = 0, basta resolver a função 0 = 3x – 9 Isolando x de um lado da igualdade, temos 3x = 9 x = 3 Portanto, para x = 3 y = 0 Para analisar o sinal, fazemos um esboço do grĂĄfico da função. JĂĄ conhecemos as coordenadas de um dos pontos da reta (3,0). O coeficiente angular ĂŠ maior que zero (a > 0). Portanto, a reta ĂŠ crescente e faz um ângulo entre 0Âş e 90Âş com o eixo x.

(0, y)

x

y

Quando x > 3, todo y > 0

(x,0)

(3,0)

Equação de 1º grau

90º≤ � ≤ 180º

(0,0)

x

Aliås, lembrando: no plano cartesiano, os ângulos são medidos sempre no sentido anti-horårio, a partir do lado positivo do eixo x.

Raiz

O ponto em que uma reta cruza o eixo x se chama raiz da função. Observe que qualquer ponto do eixo x tem coordenada y = 0. EntĂŁo, para descobrir a raiz de uma função de 1Âş grau, basta calcular o valor da função para y = 0: • Para y = 2 . x – 1, se y = 0, temos: 1 0=2.x–1 1=2.x x = 2 A raiz da função y = 2 . x – 1 ĂŠ 21 . • Para y = -3 . x + 4, se y = 0, temos: 4 0 = -3 . x + 4 -4 = - 3 . x x = 3

Então, a raiz da função y = -3 . x + 4 Ê 4 . 3

Generalizando: numa função de 1Âş grau, a raiz ĂŠ dada pela expressĂŁo: b x=–a

Toda função ĂŠ uma igualdade que sĂł pode ser estabelecida se conhecermos as coordenadas de dois pontos da reta por ela definida. Uma equação ĂŠ tambĂŠm uma igualdade, mas define apenas um dos pontos do grĂĄfico. Veja a diferença com um exemplo: Qual ĂŠ o valor de x que torna verdadeira a expressĂŁo 2x + 5 = 9? O primeiro passo e isolar x em um dos lados da expressĂŁo: 2x + 5 = 9 2x = 9 – 5 2x = 4 4 x=2 x=

-2

-1

1

0

3

2

x 4

Quando x < 3, todo y < 0

EntĂŁo, a anĂĄlise de sinal indica que: A função ĂŠ positiva para todo x real maior que 3: f(x) > 0, se x > 3 | x ∈ R . E ĂŠ negativa para todo x real menor que 3: f(x) < 0, se x < 3 | x ∈ R .

2

O valor 2 para a incógnita x Ê a solução da igualdade. Ou seja, o conjunto solução da equação Ê S = {2}. Repare que encontramos apenas um par coordenado da reta: (2, 9). Então, a partir de uma função, quando isolamos a variåval x, acabamos criando uma equação.

AnĂĄlise de sinal

Fazer a anålise de sinal de uma função significa determinar os valores de x para os quais y serå negativo, e os outros valores para os quais y serå positivo. Para isso, o primeiro passo Ê encontrar o valor no qual y = 0. Acompanhe no exemplo a seguir.

ATENĂ‡ĂƒO SINAIS EM FUNÇÕES E EQUAÇÕES

NĂŁo se esqueça: ao isolar uma incĂłgnita em um dos lados de uma equação, vocĂŞ deve inverter a operação dos termos transferidos de lado. O que de um lado ĂŠ somado, do outro deve ser subtraĂ­do; o que ĂŠ multiplicado deve ser dividido. E vice-versa. Por exemplo: • x+3=9→x=9–3 18

• 4 x = 18 → x = 4 • 3x – 7 = 12 → x =

12 + 7 3 GE MATEMĂ TICA 2017

65


3

ÁLGEBRA FUNÇÃO E EQUAÇÃO DE 1º GRAU

Inequações

Existem expressões matemáticas que, diferentemente do que ocorre com funções e equações, exprimem uma desigualdade. Essas desigualdades são indicadas pelo sinais: > (maior que) < (menor que) ⩾ (maior ou igual a) ⩽ (menor ou igual a) Resolvemos uma inequação de primeiro grau seguindo os mesmos passos usados para a resolução de equações de 1º grau, com uma particularidade importante: em algumas situações o

sinal de desigualdade é invertido (veja o Atenção ao lado). Acompanhe no exemplo a seguir. Que valores de x tornam verdadeira e inequação 3x – 9 ⩾ 0, com x ∈ R ? • Isolando a incógnita x: 3x – 9 ⩾ 0 9 3x ⩾ 9 x H 3 x ⩾ 3 • Então, S = {x ∈ R | x ⩾ 3} Veja que são infinitas as possibilidades de valores para x a fim de tornar a inequação verdadeira: para qualquer valor igual ou maior que 3, a função dará sempre um resultado maior ou igual a zero.

ATENÇÃO SISTEMA DE EQUAÇÕES

Você deve se lembrar: sistema de equações é uma forma de combinar duas equações, com duas variáveis, que permite definir os valores dessas variáveis que são válidos para ambas. Para resolver um sistema de equações, podemos realizar diversas operações. • Subtração Quando qualquer um dos coeficientes (a ou b) é o mesmo, simplesmente subtraímos uma equação de outra. Veja o exemplo a seguir. Considere o sistema:

{

2a + 2b = 8 2a – 3b = 6

O coeficiente de a é o mesmo em ambas as equações (a = 2). Então podemos simplesmente subtrair uma da outra:

encontrar o número certo – o mínimo múltiplo comum (MMC) entre os coeficientes a e b de uma delas. Com um exemplo fica mais fácil entender: Resolva o sistema 2a + 3b = 8 (I) 5a + 4b = 8 (II)

{

Veja que entre as duas equações, nem o coeficiente de a, nem o de b são iguais. Vamos, encontrar o MMC entre os coeficientes de a das duas equações, 2 e 5. O MMC é 10. Isso significa que, para deixar as duas equações com o mesmo coeficiente, temos de multiplicar a primeira equação por 5 e a segunda por 2:

{

(I) [2a + 3b = 8] . 5 = 10a + 15b = 40 (II) [5a + 4b = 13] . 2 = 10a + 8b = 26

Agora podemos subtrair uma da outra: 2a + 2b = 8 _ 2a – 3b = 3 0a + 5b = 5 b=1 Para encontrar o valor de a, basta substituir o valor de b = 1 em qualquer uma das equações: 2a + 2b = 8 2a + 2 . 1 = 8 a=

8–2 2

a=3 • Multiplicação Quando as duas equações não têm em comum nenhum coeficiente, nem a, nem b, multiplicamos as equações por um número que iguale um desses coeficientes. Para isso, é preciso

66 GE MATEMÁTICA 2017

10a + 15b = 40 (I) _ 10a + 8b = 26 (II) 0a + 7b = 14 b=2 Encontrado b, definimos o valor de a substituindo b em qualquer uma das equações: 10a + 15 . 2 = 40 10a + 30 = 40 10a = 10 a=1 A solução para o sistema é a = 1 e b = 2 Sistemas de equação são úteis para definir pontos em comum em retas concorrentes.

ATENÇÃO INVERSÃO DE SINAL

Quando, numa inequação de 1º grau, o coeficiente de x for negativo (x < 0), devemos inverter o sinal de desigualdade: de > ou ≥ para < ou ≤ , e vice-versa.


Domínio de uma função

Domínio, em matemática, é o nome que se dá às condições necessárias para que uma função se torne verdadeira. Por exemplo: você sabe que não é possível dividir um número real por zero. Portanto, para que uma função do tipo f(x)= 3/(4 . x), a variável x tem de ser diferente de zero, obrigatoriamente. Mas x pode assumir valores maiores ou menores que zero. O mesmo é válido para raízes quadradas. Não é possível extrair a raiz quadrada de números negativos. Portanto, para a expressão x – 4 jamais poderá dar um número menor que zero. Então x nunca pode assumir valores menores que 4. Repare que quando definimos o domínio de uma função, estamos resolvendo uma inequação.

NA PRÁTICA INEQUAÇÕES

Determine o conjunto-solução da inequação: -3x + 6 < 0. -3x < -6 → -x < -2 → x > 2 S = {x ∈ R | x > 2} É fácil constatar que apenas os valores de x maiores que 2 tornam a sentença verdadeira. Basta substituir na inequação qualquer valor menor que 2: • Para x = 1, temos: -3x + 6 → -3 . 1 + 6 = 3 3 não é menor que zero, portanto esse valor não torna a inequação verdadeira. Mas, se, na mesma expressão, atribuirmos a x qualquer valor maior que 2, confirmamos a inequação. • Para x = 3, temos: -3 . 3 + 6 → -9 + 6 = -3 O valor -3 é menor que zero. A expressão é verdadeira.

NA PRÁTICA ANÁLISE DE SINAL

Faça a análise de sinal da função -2x + 8 = y Primeiro, encontramos a raiz da função, atribuindo a y o valor 0 – 2x + 8 = 0 – 2x = – 8 –x=–4 x=4 Então a reta passa pelo ponto (4,0). Identificamos na expressão que o coeficiente a < 0. Portanto, a reta é decrescente e faz um ângulo entre 90º e 180º com o eixo x:

NA PRÁTICA DOMÍNIO DE UMA FUNÇÃO

Determine o domínio da função y = 2x + 3 , em que x ∈ R. Não existe raiz quadrada de número negativo. Portanto, para que a expressão acima seja uma função no campo dos números reais, precisamos garantir que o radicando (2x + 3) seja estritamente positivo ou nulo (a raiz quadrada de 0 é 0). Daí temos a inequação 2x + 3 ⩾ 0. Resolvendo a inequação: 2 x + 3 ⩾ 0 → 2 x ⩾ – 3 → x ⩾ -3/2 O domínio da função é D = {x ∈ R | x ⩾ -3/2}

SAIU NA IMPRENSA y>0

DESEMPREGO AFETA MAIS OS JOVENS, DIZ ESTUDO DO IPEA (4,0)

-1

0

1

2

3

4

x 5

y<0

Portanto, sabemos que: • f(x) > 0 se x < 4 • f(x) < 0 se x > 4

6

Levantamento do Instituto de Pesquisa Econômica Aplicada (Ipea) mostra que os jovens entre 14 e 24 anos são os mais afetados pelo desemprego. No 4° trimestre de 2015 o índice era de 15,25% e passou para 26,36% no 1° trimestre deste ano (...) O estudo (...) dividiu a população entre 14 e 24 anos entre ocupados, desocupados, aqueles que só estudam e os chamados “nem-nem”, ou seja, aqueles que nem estudam nem participam da força de trabalho. A proporção de jovens ocupados vem caindo desde 2013, de acordo com a Pnadc [Pesquisa Nacional por Amostra de Domicílios Contínua, do IBGE]. Após atingir um pico de 44% no terceiro trimestre de 2012, os jovens ocupados eram 37% no primeiro trimestre deste ano. Os jovens que somente estudavam subiram de 35% em 2012 para 38,2% no último trimestre de 2014 e recuaram novamente até 36,3% no início deste ano. Por outro lado, a proporção de jovens desocupados oscilava em torno de 8% até 2015 e alcançou 13,2% em 2016. Já a parcela de jovens nem-nem oscilou em torno de 13% durante o período (...) Portal G1, 10/6/2016 GE MATEMÁTICA 2017

67


3

ÁLGEBRA POSIÇÕES RELATIVAS DE RETAS

Paralelas, concorrentes ou perpendiculares As funções que definem as relações entre retas em um mesmo plano

D

GEASUHA/ISTOCK

uas retas distintas e que ocupem um mesmo plano apresentam, uma em relação à outra, diferentes posições. Elas podem ser paralelas, concorrentes ou perpendiculares. E, em cada um desses casos, as retas mantêm entre si algumas relações algébricas.

Paralelas

São duas retas em um mesmo plano que jamais se encontram – portanto, não têm ponto em comum. Para que isso aconteça, as duas retas devem fazer, com a horizontal, ângulos de mesmo valor. Sabemos que o coeficiente angular da forma geral da equação da reta está diretamente ligado à sua declividade – ou seja, a esse ângulo com a horizontal. Então, para duas retas serem paralelas, elas devem ter a mesma declividade, ou seja, o mesmo coeficiente a. Veja: As retas abaixo são definidas por duas equações diferentes: (I) y = 2x + 3 (II) y = 2x – 2 4 y 3

y = 2x + 3

y = 2x - 2

2 1 -3

-2

-1

0 -1 -2

68 GE MATEMÁTICA 2017

x 1

2

3

4

Repare que, nas duas equações, o coeficiente angular a tem o mesmo valor: 2. Portanto, são paralelas, independentemente do coeficiente b (coeficiente linear). Conhecendo a equação de uma reta, descobrimos a equação de qualquer paralela a ela, desde que saibamos um ponto pelo qual essa segunda reta passa. Acompanhe no exemplo a seguir. Encontre a equação da reta r que passa pelo ponto Pr (2, 3) e que é paralela à reta s, cuja equação é y = (1/2) . x + 5. A forma geral da equação de qualquer reta é y = a . x + b. Duas retas paralelas têm o mesmo coeficiente a. Conhecemos esse coeficiente da equação da reta s. Então a r = a s = 1/2 Não conhecemos o valor de b para a reta s. Mas temos os valores de x e y, nas coordenadas do ponto PR (2,3) x = 2 e y = 3. Para descobrir o valor de b da reta r, basta substituir todos esses valores (a, x e y) na forma geral da equação: y=a.x+b 3 = 1/2 . 2 + b 3 – 1 = b b = 2 1 Portanto, a equação da reta r é y = 2 x + 2 . Para desenhar as duas retas, definimos dois pontos de cada reta, atribuindo diferentes valores para x e encontrando os valores correspondente de y.

AO INFINITO E ALÉM Retas paralelas, como trilhos de trem na foto acima, se vistas num plano cartesiano, mantêm relações especiais


• Para a reta r: conhecemos o ponto Pr (2,3). Para encontrar o segundo ponto, lembramos da raiz da equação: o ponto em que a reta cruza o eixo x. Nesse ponto, y vale zero. E x = – b . a Então, pela equação de 1 r ( y = 2 x + 2 ) , sabemos que

b -2 - a = 1 =-4 2

Portanto, outro ponto de r é Pr (-4, 0). • Para a reta s: não conhecemos nenhum ponto. Mas temos a equação geral: y = (1/2) . x + 5. O valor de x na raiz é dado por x = – b = – 5 = –10 . a

1 2

Um dos pontos dessa reta, então, é Ps (–10, 0).

Outra maneira de resolver este problema é com um sistema de equações (veja na pág. 66).

ATENÇÃO

Perpendiculares

NÚMEROS INVERSOS E OPOSTOS • Números inversos são aqueles em que se inverte o numerador pelo denominador: 1 9 2 é o inverso de 2 ; 3 é o inverso de 3 . 9

As retas perpendiculares também são concorrentes, mas têm algumas características especiais. Duas perpendiculares se cruzam em ângulo reto (90º). E, para que isso ocorra, o coeficiente angular de uma das retas deve ser o inverso do oposto do outro. Veja duas retas perpendiculares, representadas no plano cartesiano, com suas respectivas equações:

• Números opostos são aqueles de sinais trocados: -4 é o oposto de 4; 19 é o oposto de -19

6

y

5

y = -2x 2 +6

4

• Para encontrar um segundo ponto de s, definimos o valor de y quando x = 0, substituindo esses valores na equação da reta

1 y= 2x+1

3 2

1 1 y = 2 .x + 5 " y = 2 .0 + 5

1

O segundo ponto de s, então, é (0,5). No gráfico:

x

0 -4 5 y

-3

-22

-1

1

2

3

4

-1 y = 00,55 x + 5

4 3 2

y = 0,5 x - 2

NA PRÁTICA

1 x

0 -11

-10

-9

Concorrentes

-8

-7

-6

-5

-4

-3

-2

-1

São retas num mesmo plano que apresentam um ponto em comum. Isso sempre acontece com retas de declividades diferentes. E as coordenadas do ponto em comum satisfazem as duas equações. Acompanhe no exemplo. Quais as coordenadas do ponto de encontro das retas de equações (I) y = 2 . x – 5 e (II) y = -3 . x + 10? A questão pede que encontremos as coordenadas do ponto P(x,y) que sejam válidas para as duas equações. Então, podemos estabelecer a igualdade entre as equações nesse ponto: 2 . x – 5 = -3 . x + 10 5x = 15 x = 3 Para encontrar y, basta substituir esse valor em qualquer uma das equações: y = 2 . 3 – 5 y = 1. Repare que chegaríamos ao mesmo resultado trabalhando com a segunda equação: y = –3 . 3 + 10 = 1.

-1

1

2

3

PERPENDICULARES

Determine a equação da reta que seja perpendicular à reta definida pela função f(x) = – 3x + 2 e que passe pelo ponto (6, 1). Vamos chamar a função da reta que se pede de g(x). Se as duas retas são perpendiculares, o coeficiente de x de uma deve ser o inverso do oposto do coeficiente da outra. Em f(x) = -3x + 2, o coeficiente de x = -3. Portanto, em g(x), o coeficiente de x é 1 3

Como para qualquer reta, a equação tem este formato: g(x) = ax + b. Conhecemos o coeficiente a. E temos as coordenadas de um ponto dessa reta: (6,1). Vamos substituir tudo na expressão de g(x): g(x) = ax + b 1 gQx V = 3 . x + b 1 1 = 3 .6 + b

1 = 2 + b → b = -1 A equação da reta perpendicular a f(x) pelo ponto (6, 1) é: 1 gQx V = 3 . x – 1

GE MATEMÁTICA 2017

69


3

ÁLGEBRA FUNÇÃO E EQUAÇÃO DE 2º GRAU JATOS LUMINOSOS Erupção de um vulcão no arquipélago de Vanuatu, no Oceano Pacífico. A gravidade faz com que a lava lançada ao espaço desenhe formas que lembram parábolas

A equação que define a parábola

A

parábola é uma cônica, mesma família das circunferências, elipses e hipérboles. Uma parábola é formada pelos pontos que mantêm a mesma distância de um ponto F (foco) e uma reta r (diretriz). Veja na figura abaixo que, para qualquer ponto da parábola (P, P’ e P’’), a distância até F é igual à distância até r.

FRIZI/ISTOCK

As funções de 2º grau são importantes ferramentas para cálculos em diversos campos da física

Pontos notáveis da parábola

Normalmente, trabalhamos com as parábolas no plano cartesiano, ou seja, sobre os eixos x e y. No plano, é possível localizar os chamados pontos notáveis da parábola: y

s

x1

P”

x2

x

P

P’

F V

r

• O ponto F é o foco da parábola • A reta r é a diretriz • O ponto V é o vértice • Passando pelo vértice e pelo foco, perpendicularmente à diretriz, está a reta s, que é o eixo de simetria da parábola

70 GE MATEMÁTICA 2017

V

C

• V é o vértice. • Os pontos x1 e x2 são as chamadas raízes da parábola – os pontos nos quais a curva corta o eixo x. • O ponto C é a intersecção da parábola com o eixo y.


As equações de uma parábola

Toda parábola é definida por uma função de 2º grau. Esse tipo de função pode ser apresentado de três formas distintas.

1) Forma geral: y = a . x2 + b . x + c

É a forma mais conhecida. Observe nos gráficos a seguir como é possível deduzir uma série de informações sobre a parábola, só analisando a função que a define: 6 z 5 4 z>y .3y,4 3

3 2 z>y3.3y,2 .3

.2

1

1

3 2 3 .2 z>y .3y.2

4

y 5

.3

Observe que: • As três parábolas têm a concavidade voltada para cima. • Em todas as funções, o coeficiente a = 1. Ou seja, o termo x2 é multiplicado por 1. • A parábola azul-escura não tem raiz (a curva não cruza o eixo x); a azul-clara tem uma única raiz, que coincide com o vértice V; a rosa tem duas raízes. • Nas três funções, o valor do coeficiente c coincide com o ponto em que a curva corta o eixo y. Agora observe os gráficos de outras três parábolas: 3

z

2 1 .3

.2

.2 .3

z>.y3,3y,2 1

2

z>.y3,3y.2

.4 .5 .6 .7

z>.y3,3y.4

3

4

y 5

Veja que: • As três têm a concavidade voltada para baixo. • Todas as funções têm o coeficiente a = – 1. • A parábola azul-escura não tem raízes, a azul-clara, tem uma única raiz, e a rosa tem duas raízes. • Cada uma das parábolas cruza o eixo y no ponto que corresponde ao valor de c na função. Da comparação entre os dois conjuntos de parábolas, podemos concluir que: • O parâmetro a está relacionado à concavidade da parábola: • Se a > 0, a concavidade é para cima; • Se a < 0, a concavidade é para baixo. • O parâmetro c é exatamente o valor da coordenada y do ponto em que a parábola corta o eixo y (no ponto em que x = 0). • O número de raízes está relacionado com o número de pontos em que a parábola cruza o eixo x. • Quando a concavidade da parábola é voltada para cima, dizemos que o vértice é o ponto de mínimo, ou seja, o vértice é o ponto da parábola no qual a coordenada y tem o menor valor possível. Da mesma maneira, se a parábola tem concavidade para baixo, chamamos o vértice de ponto de máximo – aquele em que a coordenada y atinge o maior valor possível.

2) Forma fatorada: y = a . (x – x1) . (x – x2)

Neste caso, x1 e x2 são as raízes da função – ou seja, os valores de x para os quais y = 0. É claro que só se pode representar uma parábola com a função na forma fatorada se ela tiver raízes. Desenvolvendo a forma fatorada: y = a . (x – x1) . (x – x2) y = a . (x2 – x . x1 – x . x2 + x1 . x2) y = a . (x2 – x . (x1 + x2) + x1 . x2) Vamos considerar que: • x1 + x2 = S, ou seja, S é a soma das raízes. • x1 . x2 = P, ou seja, P é o produto das raízes. Substituindo S e P na última etapa do desenvolvimento da função acima, temos: y = a . (x2 – x . S + P) y = a . x2 – a . S . x + a . P GE MATEMÁTICA 2017

71


3

ÁLGEBRA FUNÇÃO E EQUAÇÃO DE 2º GRAU

Comparando essa função com a forma geral: y = a . x2 - a . S . x + a . P y = a . x2 + b . x + c Então: b = -a . S c=a.P

As coordenadas do vértice

Quando temos a função de 2º grau em sua forma geral, podemos obter as coordenadas do vértice aproveitando o fato de que toda parábola é simétrica. Observe:

b S=–a

s

y

c p= a

Isso significa que os coeficientes da forma geral têm relação direta com as raízes da função. x1

3) Forma canônica: y = a . (x – x V )2 + y V

Esta forma descreve a parábola a partir das coordenadas de seu vértice V – ou seja, das coordenadas (xV , yV) – e de mais um ponto. O parâmetro a, novamente, é a concavidade da parábola. A forma canônica é muito útil na solução de problemas em que as coordenadas do vértice são conhecidas (veja em Como é construído o M do McDonald’s, na pág. 74).

Fórmula de Bhaskara

É a fórmula que determina as raízes de uma função de 2º grau, se elas existirem. A fórmula de Bhaskara é utilizada para resolver a equação da parábola na forma geral: a . x2 + b . x + c = 0 Para isso, basta substituir os coeficientes a, b e c na fórmula: - b ! b2 – 4 . a . c x= 2 .a

O radicando de b 2 – 4 . a . c é chamado de discriminante, ou delta (6): 6 = b2 – 4 . a . c A fórmula ficaria, então: x=

–b! D 2 .a

A equação terá: • duas soluções se 6 > 0 • uma solução se 6 = 0 • ou nenhuma solução se 6 < 0

72 GE MATEMÁTICA 2017

yv

xv

x2

x

V

O eixo s é o eixo de simetria. Repare que a coordenada x do vértice V está bem no meio do segmento que une as raízes x1 e x2: xV é o ponto médio desse segmento. Então, o valor da coordenada x do vértice V é a média aritmética das coordenadas x das raízes. Por Bhaskara, sabemos que as duas raízes são: x1 =

–b+ D 2.a

e x2 =

– b– D 2.a

Se x V =

x1 + x2 , então: 2

– b + D – b– D + 2.a 2.a xv = 2 2.b – 2.a xv = 2 b –a xv = 2 b xv = – 2 . a

Se substituirmos esse valor na forma geral da função de 2º grau, obtermos yV: D yv = – 4 . a


NA PRÁTICA

NA PRÁTICA

CANÔNICA E GERAL

COORDENADAS DO VÉRTICE

Obtenha a forma geral da função quadrática representada no gráfico abaixo:

4 y 3 2 1 x

0 -4

-3

-2

-1

1

2

Determine as coordenadas do vértice da função quadrática que passa pelos pontos P(0, -2) e Q(3, 4), sabendo que uma de suas raízes é x = – 1. Primeiro, vamos obter a expressão da forma geral da função. Depois, calcular as coordenadas do vértice a partir dos coeficientes a, b e c. Repare que conhecemos três pontos da parábola: P (0, -2), Q (3,4) e o ponto correspondente à raiz. Raiz, você se lembra, é o ponto em que a parábola corta o eixo x. E nesse ponto, y sempre vale zero. Então, conhecemos um ponto R (-1, 0).

3

Do gráfico obtemos as coordenadas do vértice (V ) e do ponto (P) no qual a curva intersecta o eixo y: V(-2, 1) e P(0, 3). Substituindo as coordenadas de V na forma canônica da função, temos: y = a . (x – x V )2 + y V y = a . (x + 2)2 + 1 y = a . (x2 + 4x + 4) + 1 (I) Substituindo as coordenadas de P, ficamos com 1 3 = a . (02 + 4 . 0 + 4) + 1 2 = 4a → a = 2 Vamos substituir o valor encontrado para a na expressão (I):

x 1 2 y = 2 . Q x + 4x + 4 V + 1 " y = 2 + 2x + 3 2

Esta é a equação na forma geral.

NA PRÁTICA FORMA FATORADA

A forma geral da função de 2º grau é y = ax2 + bx + c. Substituindo cada um desses pontos nessa forma geral, ficamos com três equações: Para o ponto P (0, - 2), temos a equação (I): -2 = a . 02 + b . 0 + c -2 = 0 + 0 + c → c = -2 (I) Para o ponto Q (3,4), temos a equação (II). Aqui já substituímos o valor de c = - 2. Veja como fica: 4 = a . 32 + b . 3 – 2 → 9a + 3b = 6 (II) Por fim, para o ponto R (-1, 0), definimos a equação (III) 0 = a . (– 1)2 + b . (-1) – 2 a – b = 2 (III) Com as equações (II) e (III) montamos um sistema de equações: 9a + 3b = 6 (II) a – b = 2 (III)

As raízes da função f(x) = ax2 + bx + c são x1 = 3 e x2 = 5. O vértice da parábola de f(x) tem coordenadas (4,1). Em que ponto o gráfico dessa função cruza o eixo das ordenadas?

Multiplicamos a equação (III) por 3. Ficamos com 3a – 3b = 6

• No ponto em que a parábola de f(x) cruza o eixo y, x = 0; • Substituindo x = 0 na forma geral da função, obtemos y = c; • O enunciado fornece as duas raízes da função, que podem entrar na forma fatorada da função. Então,

Agora, somamos as equações: 9a + 3b = 6 (II) + 3a – 3b = 6 (III) 12a = 12 → a = 1

y = a.(x – x1). (x – x2) y = a.(x – 3). (x – 5) y = a.(x2 – 3x – 5x + 15) y = a . (x2 – 8x + 15) (I)

Substituindo o valor de a em (II), obtemos: 9a + 3b = 6 → 9 . 1 + 3 . b = 6 → 3b = –3 → b = – 1

Para calcular o valor de a, basta substituir as coordenadas do vértice: (4, 1):

Se encontramos os coeficientes a = 1 e b = – 1, temos a forma geral da função da parábola: y = x2 – x – 2

1 = a.((4)2 – 8.(4) + 15) 1 = a.(16 – 32 + 15) 1 = a . (-1) a = -1

Para encontrar a coordenada x do vértice V(X,Y) , usamos a fórmula xV = – b / 2a → xV = – (–1) / 2 . 1 = 1/2

Substituindo esse valor na expressão (I), obtemos: y = (-1) . (x2 – 8x +15) y = –x2 + 8x –15

Substituindo o valor de xV na função, em sua forma geral, encontraremos a coordenada yV: yV = (1/2)2 – 1/2 –2 = 1/4 – 1/2 – 2 = – 1/4 –2 = –9/4 Portanto, o vértice tem coordenadas V (1/2, –9/4).

O gráfico da função cruza o eixo y no ponto (0, -15).

GE MATEMÁTICA 2017

73


3

ÁLGEBRA FUNÇÃO E EQUAÇÃO DE 2º GRAU

Máximo e mínimo

Nas funções em que a < 0, a parábola tem concavidade voltada para baixo. Com isso, o vértice é o ponto mais alto da curva – ou seja, sua coordenada y é maior que a de qualquer outro ponto da parábola. Nesse caso, o vértice é chamado de ponto de máximo da função. Já se a função tem a > 0, a parábola tem concavidade voltada para cima. O vértice é o ponto mais baixo da curva, e sua coordenada y, a menor de todos os pontos quer a formam. Nesse caso, o vértice é chamado de ponto de mínimo da função. Os pontos máximos e mínimos das parábolas têm aplicações muito práticas (veja os quadros Na prática a seguir.)

SAIBA MAIS COMO É CONSTRUÍDO O M DO MCDONALD'S Os designers gráficos empregam várias figuras geométricas no desenho de logotipos. Observe com atenção o logo da rede de lanchonetes McDonald’s. Repare que as pernas da letra M têm a forma muito parecida com a de duas parábolas:

NA PRÁTICA PONTO MAXÍMO

Um fazendeiro deseja cercar uma área retangular para fazer um galinheiro. Ele dispõe de 16 metros de alambrado. E um dos lados do galinheiro não precisa da cerca porque é um muro. Qual é a maior área possível desse galinheiro?

O M no plano cartesiano: z

!

5

A figura abaixo mostra a situação do enunciado: muro x

4 3

x

2

y

O alambrado será usado para fechar apenas três lados do retângulo, ou seja, os 16 metros de alambrado irão cobrir dois comprimentos x (altura do retângulo) e um comprimento y (largura). Então, 2x + y = 16 y = 16 – 2x (I) A área do galinheiro é a área do retângulo: A = x . y Substituindo o valor de y encontrado em (I) na expressão da área, obtemos: A = x . (16 – 2x) A = 16x – 2x2

.2

74 GE MATEMÁTICA 2017

2

3

4

5

6

y

.2 .3

Trabalhando apenas com metade do logotipo, podemos reconhecer os pontos notáveis da parábola da esquerda: o vértice Ve e as raízes x1 e x2 da função de 2º grau que define a parábola: y

Esta é uma função de 2º grau, com coeficiente a = -2. O coeficiente é negativo, então a parábola tem concavidade voltada para baixo. E seu vértice será o ponto máximo – o ponto que corresponde à área máxima possível para o retrângulo. Calculando o valor de x no vértice: xV = – b / 2a Portanto, a área do galinheiro é máxima quando os lados x medem 4 metros. Com dois lados medindo 4 metros, para cobrir a altura do retângulo o fazendeiro consumirá 8 metros do alambrado disponível. Os restantes 8 metros cobrem o terceiro lado (largura). Então a área do retângulo é A = 8 . 4 = 32 m2

1

4

Ve

3 2 1 -1

C -1 -2

x1

1

2

x2

3

4

5

x


• O vértice tem as coordenadas V (1, 3). • O ponto x1 tem como coordenadas (0, 0). • O ponto x2 tem como coordenadas (2, 0). Repare: • A concavidade é voltada para baixo. Isso significa que o coeficiente a deve ser negativo.

geral a partir da forma fatorada. Vamos aplicar essa estratégia para obter a função da parábola da direita do logotipo do McDonald’s: z

!

5 W 4

• A parábola corta o eixo y no ponto zero. Então o coeficiente c deve ser nulo. Podemos, agora, deduzir a função que define cada um dos pontos da figura, por dois caminhos: 1) A partir das coordenadas do vértice (V) Vamos trabalhar com a forma canônica da função. Para isso, devemos conhecer as coordenadas do vértice e as de mais um ponto da parábola – o ponto x2, por exemplo. Então, temos estes dados:

3 2 .2

1

2

3 y2!

4

5 y3!

6

y

.2 .3

Só de observar a figura já sabemos que: V (1, 3) e x2 (2, 0) Substituindo as coordenadas do vértice em y = a . (x – xV)2 + yV , temos: y = a . (x – 1) + 3 2

Desenvolvendo: y = a . (x – 1) . (x – 1) + 3 y = a . (x2 – 2.x + 1) + 3 (I) Para obter o valor do coeficiente a, substituimos as coordenadas do outro ponto conhecido, x2 = (2, 0). Ou seja, x = 2 e y = 0. Então: 0 = a . (22 – 2 . 2 + 1) + 3 0 = a . (4 – 4 + 1) + 3 0=a.1+3 a = -3 Substituindo esse valor em (I), temos: y = -3 . (x2 – 2 . x + 1) + 3 Aplicando a propriedade distributiva da multiplicação, obtemos: y = -3 . x2 + 6 . x – 3 + 3 y = -3 . x2 + 6 . x Está confirmado: o coeficiente a é negativo e o coeficiente c é nulo. 2) A partir das raízes Se conhecemos as raízes de uma função de 2º grau, conseguimos obter a expressão da função em sua forma

• As raízes da função que define essa parábola são x1 = 2 e x2 = 4 • O vértice tem como coordenadas Vd (3, 3). Vamos substituir as raízes na forma fatorada da função: y = a . (x – x1 ) . (x – x2 ) y = a . (x – 2) . (x – 4) Efetuando a multiplicação indicada, temos: y = a . (x2 – 6x + 8) (I) Agora, é só substituir as coordenadas do outro ponto conhecido, no caso o vértice V (3, 3): 3 = a . (32 – 6 . 3 + 8) 3 = a . (9 – 18 + 8) 3 = a . (-1) a = -3 Substituindo esse valor em (I), temos: y = - 3 . (x2 – 6 . x + 8) y = - 3 . x2 + 18 . x – 24 Essa é a forma geral da função de 2º grau que tem como gráfico a parábola da direita do logotipo do McDonald’s. Observe que, assim como na primeira parábola, o coeficiente a desta segunda parábola também é negativo: sua concavidade é para baixo. E tem o mesmo valor (-3). Os outros coeficientes (b e c) também influem na localização de cada parábola no plano cartesiano. No caso da parábola da direita, c = - 24 significa que, se a parábola fosse prolongada, ela cruzaria o eixo y na coordenada (0, -24). GE MATEMÁTICA 2017

75


3

COMO CAI NA PROVA

1. (UEG 2015) Considere o gráfico a seguir de uma função real afim f(x). y 3

f(x)

2 1 -4

∆ = b² – 4 . a . c = (2m)² – 4 . 1 . 9 = 0 4 m² = 36 → m² = 9 m = 3 ou m = –3

-3

-2

-1

-1

x 1

2

3

4

A função afim f(x) é dada por a) f(x) = – 4x + 1 b) f(x) = – 0,25x + 1 c) f(x) = – 4x + 4 d) f(x) = – 0,25x – 3

RESOLUÇÃO Lembrando: no plano cartesiano, uma reta é definida por dois pontos, cada um deles identificado por um par ordenado (x, y). E a reta é descrita por uma função afim, que tem forma geral y = ax + b. Nessa expressão, a é o coeficiente angular da reta e b, o coeficiente linear. Vamos verificar o valor de a e b. Veja que, no gráfico, identificamos facilmente dois pontos da reta: • o ponto em que a reta cruza o eixo y: (0, 1). A ordenada deste ponto (y, que aqui vale 1) é o coeficiente linear da reta. E corresponde ao coeficiente b de f(x). Então já sabemos que a função terá a forma f(x) = ax + 1 • identificamos, também, o ponto em que a reta cruza o eixo x: (4, 0). Para encontrar o coeficiente linear (a), usamos a fórmula: a= ∆y ∆x Temos, então a = ∆ y → a = 1 – 0 = – 1 → a = – 0,25 ∆x 0–4 4 Então, a expressão de f(x) para esta reta é f(x) = – 0,25 x + 1 Detalhe: repare que o coeficiente angular da reta é negativo (a = – 0,25). Isso indica que a função é decrescente – o que se confirma pelo gráfico. Resposta: B

O enunciado diz ainda que o ponto da parábola que tem a ordenada y = 9 tem x < 0. Fazendo y = 9, temos: 9 = x² + 2 mx + 9 → x² + 2 mx = 0 x (x + 2m) = 0 Essa equação tem duas raízes: x = 0 e x = –2 m Como x < 0, só podemos ficar com a raiz x = –2 m. Para que isso seja verdade, devemos ter m = 3. Resposta: B

3. (UEMG 2016) O lucro de uma empresa é dado pela expressão matemática

L = R – C, onde L é o lucro, C o custo da produção e R a receita do produto. Uma fábrica de tratores produziu n unidades e verificou que o custo de produção era dado pela função C(n) = n² – 1 000 n, e a receita representada por R(n) = 5000 n – 2n². Com base nas informações acima, a quantidade n de peças a serem produzidas para que o lucro seja máximo corresponde a um número do intervalo a) 580 < n < 720 b) 860 < n < 940 c) 980 < n < 1 300 d) 1 300 < n < 1 800

RESOLUÇÃO Repare que as expressões para custo (C) e receita (R) são dadas em função de n. Se L = R – C, vamos substituir nessa expressão as funções que definem C e R em função de n: L (n) = 5 000 n – 2 n² – (n² – 1000 n) = –3 n² + 5 000 n + 1 000 n L (n) = –3 n² + 6 000 n Trata-se de uma função quadrática (y = ax2 + bx + c), em que a = –3, b = 6 000 e c = 0; n é a variável independente, que no gráfico corresponde ao eixo das abscissas. Os gráficos de funções de segundo grau são parábolas. Veja que o coeficiente a é negativo (–3). Então, essa parábola tem concavidade voltada para baixo. Portanto, o máximo dessa função está no vértice – é aí que se identifica o valor de n (número de peças para o máximo lucro). Veja: y

2. (Uece 2016) No plano, com o sistema de coordenadas cartesianas usual,

o gráfico da função f: R → R definida por f(x) = x² + 2 mx + 9 é uma parábola que tangencia o eixo das abscissas, e um de seus pontos com ordenada igual a 9 tem abscissa negativa. Nessas condições, o valor do parâmetro m está entre a) 1,5 e 2,5 b) 2,5 e 3,5 c) 3,5 e 4,5 d) 4,5 e 5,5

RESOLUÇÃO Se a parábola tangencia o eixo x, então ela não cruza esse eixo, apenas encosta nele, em um único ponto. Isso significa que a função tem apenas uma raiz real. Para que isso ocorra, é necessário que o discriminante da função seja nulo, ou seja, ∆ = 0. Calculando esse valor para a função dada no enunciado, temos:

76 GE MATEMÁTICA 2017

V

yv

x1

xv

x2

x

O valor de n mais alto corresponde à abscissa do vértice da parábola (xv). Pela expressão que define a abscissa do vértice, temos: xv = – b = – 6 000 = 1 000 2a 2 . (–3) Portanto, o lucro será máximo quando n = 1 000 Resposta: C


RESUMO

4. (Aman 2016) Um portal de igreja tem a forma de um arco de parábola,

conforme a figura abaixo. A medida da sua base AB é 4 m e da sua altura é 5 m. Um vitral foi colocado 3,2 m acima da base. Qual a medida CD da base, em metros? vitral

C

D

Álgebra FUNÇÃO DE 1º GRAU f(x) = y = a . x + b, em que: • a é o coeficiente angular da reta, definido pela diferença entre as coordenadas x e y de dois pontos quaisquer da reta: Dy QyA – yBV QyB – yAV = = Dx Q x A – x B V Q x B – x A V

a= B

A

• b é o coeficiente linear da reta: o valor de y no ponto em que a reta cruza o eixo y (ou seja, o ponto que tem a coordenada x = 0).

desenho ilustrativo – fora de escala

a) 1,44

b) 1,80

c) 2,40

d) 3,00

e) 3,10 • A raiz da função é o valor de y no ponto em que a reta cruza o eixo x :

RESOLUÇÃO A figura abaixo mostra o portal num sistema de eixos cartesianos. y (0, 5)

PARÁBOLA

C

A

D

–1

0

b x=–a

1

z!

V é o vértice da parábola x1 e x2 são as raízes da parábola

W

B

Acompanhe: • o ponto mais alto do portal corresponde ao vértice da parábola. Para facilitar os cálculos, colocamos o vértice sobre o eixo y. Se a altura do portal é de 5 metros, sabemos que esse ponto tem coordenadas (0, 5). • a base do portal (AB) está sobre o eixo x. Como AB mede 4 metros, os pontos em que a parábola corta esse eixo têm coordenadas A(–2, 0) e B(2, 0). A e B correspondem às raízes da equação da parábola. Obtemos a equação pela forma fatorada: y = a (x – x1) . (x – x2) → y = a (x + 2) . (x – 2) → y = a (x² – 4) Para calcular o valor de a, substituímos nesta expressão as coordenadas do vértice V (0, 5). 5 = a (0² – 4) → – 4 a = 5 → a = – 5 4 Portanto, a equação da parábola é y = – 5 x + 5 4 2

Mas, atenção: precisamos, ainda, descobrir a medida entre os pontos C e D, que definem a base do vitral. Sabemos que o vitral foi instalado 3,2 metros acima do solo – ou seja, para C e D, y vale 3,2. Substituindo esse valor na equação da parábola, temos: 3,2 = – 5 x2 + 5 → – 1,8 = – 5 x2 4 4 x² = 1,44 xC = – 1,2 e xD = 1,2 A distância entre C e D é dada pela diferença xC – xD = –1,2 – 1,2 = –2,4 Distância é uma grandeza física de valor sempre positivo. Então, trocamos o sinal: a distância CD é de 2,40 m. Resposta: C

y! y3

y2

FUNÇÃO DE 2º GRAU

• Forma geral: y = a . x2 + b . x + c • Forma fatorada: y = a . (x – x1) . (x – x2) • Forma canônica: y = a . (x – xV)2 + yV FÓRMULA DE BHASKARA x=

–b ! D 2 .a

A equação terá: • duas soluções se ∆ > 0 • uma solução se ∆ = 0 • ou nenhuma solução se ∆ < 0 COORDENADAS DO VÉRTICE b xv = – 2 . a

D yv = – 4 . a

CONCAVIDADE DA PARÁBOLA É definida pelo valor de a na função y = a . x2 + b . x + c • Se a > 0, a concavidade é para cima • Se a < 0, a concavidade é para baixo

GE MATEMÁTICA 2017

77


4

POTÊNCIA E LOGARITMO CONTEÚDO DESTE CAPÍTULO

Potenciação .......................................................................................................80 Funções e equações exponenciais .............................................................82 Funções e equações logarítmicas...............................................................86 Como cai na prova + Resumo .......................................................................92

Abalos semelhantes, prejuízos diferentes As consequências de um terremoto dependem mais das condições socioeconômicas da região afetada do que da magnitude do tremor

O

s geólogos afirmam: as consequências – mais ou menos catastróficas – de um terremoto dependem menos de sua magnitude do que das condições da ocupação humana na superfície. A ideia foi, mais uma vez, confirmada por dois episódios recentes. Em abril de 2016, Equador e Japão foram atingidos por abalos sísmicos de magnitudes similares – 7,8 e 7,3 graus na escala Richter, respectivamente. Mas, no país latino-americano, o número de mortos, feridos e desabrigados, somados, superou os 270 mil, enquanto na nação asiática o tremor afetou pouco mais de 120 mil pessoas. Essa disparidade tem, no fundo, uma razão econômica: países ricos têm mais recursos para construções mais sólidas, acesso a tecnologias que minizam os efeitos dos abalos e, também, políticas públicas preventivas, como o preparo da população para situações de emergência. Tanto é assim que no Equador, 64ª economia no ranking mundial, o prejuízo financeiro do terremoto de abril equivale a 3% de todas as riquezas geradas pelo país, a cada ano, o Produto Interno Bruto (PIB). No Japão, terceira economia mundial, tudo o que ruiu não comprometeu mais que 0,7% do PIB. Terremotos acontecem devido ao movimento das placas tectônicas – imensos blocos de rocha que flutuam sobre o magma, fluido e em eterna

78 GE MATEMÁTICA 2017

ebulição, nas profundezas da Terra. As placas podem se chocar de frente, mergulhar uma sob a outra ou deslizar em paralelo. Quando elas se enroscam, o ponto de encontro vai guardando uma quantidade cada vez maior de energia. E um dia se soltam, liberando, de uma única vez, uma quantidade imensa de energia. É essa energia, transmitida em ondas, que sobe do subsolo e sacode tudo o que está na superfície. Quanto maior a energia liberada, maior a magnitude do tremor, medida na escala Richter. Nem sempre o movimento tectônico é destrutivo. Algumas placas se afastam, abrindo fendas por onde o magma escapa, por vulcões, no geral. É o que ocorre nas bordas do Oceano Pacífico, onde o chamado Círculo de Fogo concentra centenas de vulcões ativos. Esse transbordar de lava cria novas terras. O arquipélago do Havaí, por exemplo, está em eterno crescimento, graças a esse processo geológico. BALANÇA E CAI Neste capítulo você O terremoto que assolou o revê o conceito e as Equador em abril de 2016 propriedades de loga- levou ao chão edifícios como ritmos, a base da escala este, na cidade de Portoviejo. Richter. E, também, da Em todo o país, mais de operação matemática 650 pessoas morreram e inversa – potenciação. 12,5 mil ficaram feridas


GE MATEMÁTICA 2017

79

JUAN CEVALLOS / AFP PHOTO


POTÊNCIA E LOGARITMO POTENCIAÇÃO

KTSIMAGE/iSTOCK

4

A potência de um número A operação de potenciação nada mais é que o número multiplicado por ele mesmo seguidas vezes

A

medida da magnitude de um terremoto é dada pela escala Richter, baseada em funções logarítmicas. Para entender bem como funcionam os logaritmos, vamos antes ver as operações com potências. Potenciação nada mais é que a multiplicação de um número por ele mesmo, por várias vezes: 22 = 2 . 2 = 4 23 = 2 . 2 . 2 = 8 Uma potência é sempre indicada na forma ab = c, com a & 0, em que a é a base; b é o expoente; c é a potência.

Potências de base 10

As potências de base 10 são especialmente úteis para facilitar os cálculos e estabelecer ordens de grandeza, na notação científica (veja o quadro Saiba mais, ao lado). Veja como é simples escrever um número com muitos zeros usando potência de base 10. Por exemplo: 150 000 000 000 10 casas Pelo número de zeros, percebemos que esse valor é resultado da multiplicação de 15 por 10 vezes o número 10. Então 150 000 000 000 = 15 . 1010. Repare que o número de zeros é exatamente o expoente da potência de 10.

80 GE MATEMÁTICA 2017

A ESCALA DA NANOTECNOLOGIA Potências de dez com expoente negativo são ideais para indicar o tamanho de robôs minúsculos como este, entre hemácias


Outro exemplo: 0,000 000 0037

TOME NOTA

10 casas Para escrever esse número usando potência de dez, deslocamos a vírgula no sentido inverso, da esquerda para a direita. O expoente é exatamente o número de casas em que a vírgula foi deslocada. Mas, agora, o expoente é negativo: 0,000 000 0037 = 37 . 10–10

Operações com potências

As potências têm propriedades, que definem algumas regras para operar com elas. • Soma ou subtração de potências Duas potências só podem ser somadas ou subtraídas se tiverem a mesma base e o mesmo expoente. Veja: • 3 . 106 + 4 . 106 = (3 + 4) . 106 = 7 . 106 Repare que somamos apenas as mantissas. A potência fica inalterada. • Já para a soma 7 . 10-5 + 3 . 10-3, a base é a mesma, mas os expoentes são distintos. Para fazer a soma, temos de dar às potências uma mesma ordem de grandeza (mesmo expoente). Vamos passar 3 . 10-3 para a ordem de grandeza -5. Para isso, deslocamos a vírgula duas casas para a direita: 3 . 10-3 = 300 . 10-5 Agora a soma é possível: 7 . 10-5 + 300 . 10-5 = 307 . 10-5 • Produto de potências de mesma base Ao multiplicar potências de mesma base, somamos os expoentes: am . an = a (m + n) Exemplo: 26 . 25 = 2 (6 + 5) = 211 • Quociente de potências de mesma base Na divisão de potências de mesma base, subtraímos um expoente de outro: am = a (m–n) an

Exemplo: 2 = 2 =2 =2 25 • Potência de potência Os expoentes são multiplicados: (am)n = am . n Exemplo: (32)2 = (3 . 3)2 = 92 = 9 . 9 = 81 Pela propriedade de potência de potência, chegamos mais rapidamente ao resultado: (32)2 = 3(2 . 2) = 34 = 3 . 3 . 3 . 3 = 81 6

(6 – 5)

1

• Potência de expoente fracionário São as potências em que o expoente é uma fração. Nesse caso, a potência indica uma raiz: a 3

Exemplo: 2 4 =

4

m n

23

=

n

am

EXPOENTES 1 E ZERO • Um número a elevado a 1 é igual a a. • Um número a elevado a 0 é igual a 1, desde que a ≠ 0.

• Potência de um produto A multiplicação elevada a determinado expoente é igual à multiplicação de cada fator elevado àquele mesmo expoente: (a . b)m = am . bm Exemplo: (2 . 3)2 = 22 . 32 Confira: (2 . 3)2 = 62 = 36 22 . 32 = 4 . 9 = 36 • Potência de um quociente Da mesma maneira, uma divisão (ou fração) elevada a determinado expoente é igual à divisão de cada fator elevado ao mesmo expoente:

RbW = m b a

m

am

3 3 Exemplo: S 10 X = 103 2

2 Confira, de novo: S 10 X3 = 5 3 = 125 2 10 3 = 1 000 = 125 8 23

ATENÇÃO EXPOENTES POSITIVOS E NEGATIVOS Numa potência de expoente positivo, o número é multiplicado: 52 = 5 . 5 = 25 Já o expoente negativo indica que a base é o denominador de uma fração de numerador 1: 1 1 5 -2 = 5 . 5 = 25 = 0, 04

Como regra geral, lembre-se de que 1 n a -n = S a X

SAIBA MAIS NOTAÇÃO CIENTÍFICA

Notação científica é a forma como a ciência apresenta valores. Um número está em notação científica quando aparece na forma n = k . 10 x, em que 1 ⩽ k < 10. Isso significa que k (chamado mantissa) é sempre um valor entre 1 e 9,9999999.... (indefinidas casas decimais) – jamais menor que 1 e nunca igual a 10 ou superior a isso. Alguns exemplos de transformação de um número em notação científica: • 15 . 1010, em notação científica, é escrito como 1,5 . 1011 • 37 . 10–8, em notação científica, é expresso como 3,7 . 10–9 Números assim pequenos são usados, por exemplo, para as dimensões diminutas da nanotecnologia, área da engenharia que lida com objetos do tamanho de moléculas . GE MATEMÁTICA 2017

81


POTÊNCIA E LOGARITMO FUNÇÕES E EQUAÇÕES EXPONENCIAIS

RAVENNKA/iSTOCK

4

As expressões para variações muito grandes Como resolver funções em que a variável é o expoente de um número

F

unção exponencial é aquela que tem como variável independente o expoente de uma potência. Ou seja, a variável dependente é definida em função de uma potência. Funções exponenciais têm a seguinte forma geral: f(x) = m . ax, em que: • f(x) é a variável dependente; • x é a variável independente; • m é um número real qualquer; • a é um número real maior que zero e diferente de 1. As funções exponenciais são usadas em diversas situações da economia e das ciências – no cálculo de juros compostos (veja na página 24), na estimativa de crescimento de uma população ou na redução da amostra de um elemento radioativo, que variam em progressão geométrica (veja no capítulo 6).

Funções crescentes

As funções exponenciais podem ser crescentes ou decrescentes. Exemplo de função crescente é a que descreve o crescimento de uma população de bactérias – microrganismos que se reproduzem por simples divisão celular. A multiplicação de bactérias segue uma regularidade: a partir da população inicial, é possível calcular o tamanho da população depois de determinado tempo.

82 GE MATEMÁTICA 2017

EXPLOSÃO POPULACIONAL As bactérias se multiplicam em ritmo exponencial. Por isso, uma cultura delas cresce de maneira extremamente rápida

O QUE ISSO TEM A VER COM A BIOLOGIA As bactérias são microrganismos unicelulares, que se reproduzem por mitose – cada célula se divide em duas, que carregam material genético idêntico.


Imagine que um tipo de bactéria se divida a cada 3 horas. Vamos chamar de x a quantidade de períodos de 3 horas, e de y o número de bactérias a cada etapa. Se partimos de uma única bactéria, a tabela é a seguinte: Períodos de 3 horas (x) 0 1 2 3 4 5 6 7

Nº de bactérias (y) 1 2 4 8 16 32 64 128

Repare que o número de bactérias se duplica a cada etapa (veja no infográfico ao lado). Então y varia em função de uma potência de base 2. A função que descreve o número de bactérias ao longo do tempo é: y = 1 . 2x Agora imagine que a população inicial seja de três bactérias. Cada uma se divide em duas, seguidamente. Então y continua variando em função de uma potência de base 2. Mas, como partimos de três bactérias, os valores são multiplicados por 3. A tabela ficaria assim: Períodos de 3 horas (x) 0 1 2 3 4 5 6 7

MULTIPLICAR PARA CRESCER À medida que se dividem, o número de bactérias cresce exponencialmente

Nº de bactérias (y) 3 . 20 = 3 3 . 21 = 6 3 . 22 = 12 3 . 23 = 24 3 . 24 = 48 3 . 25 = 96 3 . 26 = 192 3 . 27 = 384

Generalizando, para uma população inicial de três bactérias, a função para a variação de y é: y = 3 . 2x De modo geral, a função que descreve o crescimento das bactérias é: y = m . 2x , em que • m é a quantidade inicial de bactérias, no momento zero (x = 0); • 2 é a razão de crescimento. Como o número de bactérias sempre aumenta, esta é uma função crescente.

Etapa I

Etapa II

Etapa III

COMO ELAS SE REPRODUZEM Na etapa I, uma bactéria se divide em duas. Essas duas bactérias tornam a se dividir na etapa 2, e assim por diante. Em intervalos regulares de tempo, o número de bactérias na população sempre se duplica – ou seja, a população cresce em potência de expoente 2 (20, 21, 22, 23 ...)

Funções decrescentes

O exemplo clássico de função exponencial descrescente é a desintegração radioativa. Esse fenômeno ocorre naturalmente com os elementos químicos no final da tabela periódica, como urânio e o radônio. A desintegração obedece à lei geral: em intervalos regulares de tempo, metade dos átomos de uma amostra se desintegra, transformandose em elementos químicos mais leves (veja o infográfico na página 84). Cada um desses intervalos é chamado meia-vida. Diferentes elementos se desintegram em diferentes períodos. O urânio-238 tem meia-vida de cerca de 4,5 bilhões de anos – ou seja, uma amostra de mil átomos de urânio-238 precisa de 4,5 bilhões de anos (praticamente a idade da Terra) para se reduzir à metade. Já para o radônio-220, bastam pouco mais de 50 segundos para isso acontecer. O iodo-131, muito utilizado em exames de diagnóstico por imagem, também é um elemento radioativo. Quando o paciente passa por uma máquina de tomografia computadorizada, o iodo-131 denuncia a presença de um tumor. Como todo elemento radioativo, o iodo-131 também tem seu período de meia-vida. Se injetarmos uma amostra de 100 gramas de iodo-131 em um paciente, oito dias depois restarão no organismo apenas 50 gramas do elemento ainda radioativo. Mais oito dias, e o material radioativo terá decaído pela metade, novamente: restarão apenas 25 gramas. E assim por diante. Considerando que a massa de iodo radioativo seja y, e a quantidade de períodos de meia-vida seja a variável x, é possível montar uma tabela que indique o que resta de material radioativo no organismo do paciente depois de determinado tempo. GE MATEMÁTICA 2017

83


4

POTÊNCIA E LOGARITMO FUNÇÕES E EQUAÇÕES EXPONENCIAIS

Períodos de meia-vida

Massa (g)

x

y

0

100

1

50

2

25

3

12,5

4

6,25

5

3,125

Duplicação das bactérias: função crescente y = a . 2x Decaimento radioativo: função decrescente 1 x y = 100 . S 2 X

Repare que a diferença fundamental entre as expressões das duas funções é o valor da base da potência:

Repare que a sequência de valores para y é obtida pela divisão do valor anterior por dois. Ora, dividir por dois é a mesma coisa que multiplicar por ½. Isso significa que essa sequência é uma sucessão de multiplicações por ½. É aí que surge a potência, agora de base ½. A função que descreve o fenômeno é, então:

• Na função crescente, trata-se de um valor maior que 1; • Na função decrescente, a base da potência é um valor positivo, ou seja, maior que zero, porém menor que 1. É a lógica: dividir por dois é o mesmo que multiplicar por ½. CRESCIMENTO DE BACTÉRIAS

1 y = 100 . S 2 X

x

10 y

Note que a forma geral dessa função é a mesma que a da função para a duplicação de bactérias. Entretanto, na primeira situação, a base da potência era 2, que é maior que 1. Isso indica que o y cresce conforme x cresce – função crescente. Já no segundo caso, a base tem valor positivo, mas é uma fração, ou seja, menor que 1. Então, neste caso, à medida que x cresce, o valor de y cai – função decrescente.

9 8

Quantidade inicial: 1 bactéria

7

As bactérias se duplicam a cada x dias

5

Função: y = 1 . 2x

3

(3, 8)

6

4 (2, 4)

2

Gráficos da função exponencial

Funções crescentes e decrescentes definem gráficos diferentes. Vamos retomar os exemplos da duplicação de bactérias e do decaimento de iodo-131 e analisar cada situação.

(-1, 0,5) -5

-4

-3

-2

(1, 2)

1 0 -1

(0, 1) 0

1

2

3

x 4

-1

MEIA-VIDA DE UM ELEMENTO RADIOATIVO Em intervalos regulares de tempo, metade de uma amostra de átomos radioativos se desintegra

A amostra contém, originalmente, oito átomos radioativos

Passado determinado tempo, apenas quatro átomos permanecem radioativos

Depois de dois períodos iguais de tempo, o número de átomos radioativos caiu para dois.

METADE DA METADE O número de átomos de uma amostra radioativa cai pela metade a intervalos regulares de tempo. No infográfico você vê como essa queda ocorre. Da amostra original de 8 átomos radioativos, apenas 4, a metade, não se desintegraram em elementos mais leves, depois do intervalo de tempo t. Passado outro intervalo t, o número de átomos radioativos cai de novo pela metade: de 4 para 2. O decaimento obedece a uma função exponencial de base 1/2.

84 GE MATEMÁTICA 2017

O QUE ISSO TEM A VER COM A QUÍMICA O decaimento radioativo ocorre em átomos de elementos químicos que têm um número exgerado de prótons. Prótons têm carga elétrica positiva, e quando existem prótons demais, a força de repulsão entre eles é tão intensa que os nêutrons não conseguem manter o núcleo inteiro. Este é um fenômeno físico. Mas, para a química, conhecer esse processo é importante para o desenvolvimento de substâncias como as usadas em diagnóstico por imagem.


para igualar as bases nos dois membros da equação. Acompanhe no exemplo a seguir.

CONCENTRAÇÃO DE IODO-131 110

y

Qual o valor do expoente x que torna verdadeira a equação

100 (0, 100)

x S 1 X = 27 ? 9

90 80 70

Quantidade inicial: 100 g

60 50

(1, 50)

40

Repare que tanto 9 quanto 27 podem ser reescritos como potências de base 3: 9 = 32 e 27 = 33

Meia-vida: 8 dias Função:

30

1 x y = 100 . S 2 X

Então podemos reescrever a equação: x x S 1 X = 27 = Q 3 -2 V = 3 3 9

20 (3, 12,5)

10

x

0 -1 0 -10

1

2

3

4

5

6

7

8

-20 -30

A respeito dos dois gráficos, podemos ressaltar: • Ambas as curvas passam pelo ponto (0, m). No gráfico das bactérias, m = 1; no gráfico da concentração de iodo, m = 100. Isso ocorre quando o expoente x assume valor zero. E qualquer número elevado a zero é igual a 1. Veja: y = m . ax y = m . a0 = m . 1 Então, y = m • As curvas se aproximam do eixo x sem jamais tocá-lo, formando aquilo que em matemática se chama de assíntota: duas curvas que não são paralelas tendem a se aproximar mais e mais, mas nunca se tocam. Isso ocorre porque o valor de y nunca chegará a zero, pois a base da potência é sempre diferente de zero. • O eixo x representa o tempo. A parte pontilhada no alto do gráfico da concentração de iodo 131 mostra como seria a curva se fosse possível contar o tempo antes do início (zero).

Equação exponencial

São equações nas quais a incógnita é o expoente de uma potência. Se duas potências com bases iguais são iguais, então seus expoentes também são iguais. Veja: ax = ap x = p A estratégia para resolver uma equação exponencial é usar as propriedades das potências

Pela propriedade de potência de potência, eliminamos os parênteses no primeiro membro: 3–2x = 33 Numa igualdade entre potências, se as bases são iguais, os expoentes também devem ser iguais. Portanto, 3 -2x = 3 " x = - 2

NA PRÁTICA EQUAÇÃO EXPONENCIAL Resolva a equação S

1 x 2 X = 128

A fração 1/2 pode ser escrita como 2–1. E 128 é o mesmo que 27 Então podemos reescrever a equação assim: R

Q 2 -1 V W = 2 7 x

Pela propriedade de expoentes fracionários, sabemos que mesmo que 2 -1/2

2 -1 é o

Então podemos reescrever a equação, de novo: R 2 - 2 W = 2 7 1 x

Pela propriedade de potência de potência, multiplicamos os expoentes do 1º termo da expressão: 1

2- 2 x = 27

Temos agora duas potências de mesma base. Portanto, os expoentes também devem ser os mesmos: 1 -2x=7 x = -14

GE MATEMÁTICA 2017

85


4

POTÊNCIA E LOGARITMO FUNÇÕES E EQUAÇÕES LOGARÍTMICAS

PETROVICH9/iSTOCK

RABISCOS DE UM TERREMOTO As ondas de um tremor são registradas por um sismógrafo. A escala Richter, que mede a intensidade do abalo, é logarítmica

Definição de logaritmo

Para medidas extremas

O logaritmo de um número é a operação para encontrar o expoente de uma potência. Veja:

logba = c

Como os logaritmos permitem comparar valores que variam muito

V

ocê já viu no início deste capítulo: a energia liberada por um terremoto se propaga em ondas. Os sismógrafos captam essas ondas e indicam a amplitude delas. Quanto maior a amplitude, maior a energia liberada e, portanto, maior a intensidade do tremor. A medida dessa intensidade é dada pela escala Richter como magnitude. A cada ponto da escala, a energia liberada aumenta 31,6 vezes. A energia de um tremor de 5 pontos na escala Richter é igual àquela liberada pela explosão de 32 toneladas de dinamite. Outro abalo, de magnitude 6, libera uma energia equivalente a 1 milhão de toneladas de explosivos. Essa diferença é tão grande porque a escala Richter é uma escala logarítmica. (veja mais sobre a escala Richter na pág. 89).

86 GE MATEMÁTICA 2017

base

O QUE ISSO TEM A VER COM FÍSICA Onda é um fenômeno estudado pela física. Uma onda só transfere energia cinética para o meio, jamais carrega matéria. Existem ondas mecânicas – como as que se propagam na água quando se lança uma pedra – e ondas eletromagnéticas, como as da luz.

logaritmando

logaritmo

Existem algumas condições para que um logaritmo exista: logb a = c ) bc = a, com a & 0, b & 0 e b & 1. Uma das vantagens de compreender a relação entre logaritmo e potência é que, se você estiver trabalhando com vários valores, todos dados na forma de potências de mesma base, pode deixar a base de lado e operar somente com os expoentes. Por exemplo: Imagine um problema que exija que se multipliquem os números: 100 10 000 0,00001 1 000 000 000 000 0,01


Repare que todos os valores apresentados são potências de 10: 100 = 102 10 000 = 104 0,00001 = 10-5 1 000 000 000 000 = 1012 0,01 = 10-2 Para multiplicar esses valores, basta manter a base 10 e somar os expoentes: 2 + 4 – 5 + 12 – 2 = 11

Em primeiro lugar, vamos simplesmente aplicar a definição de logaritmo para determinar alguns deles. Assim: logba = c

)b =a c

Ora, se 25 = 32, então, pela definição, log2 32 = 5 Assim, também podemos dizer que: log2 256 = 8, pois 28 = 256. log2 1 = 0, pois 20 = 1.

Logaritmo do produto

O resultado é 10

11

Seguindo o mesmo raciocínio, a divisão desses números pode ser feita simplesmente subtraindo-se os expoentes: 100 : 10 000 : 0,00001 : 1 000 000 000 000 : 0,01 = 102 : 104 : 10-5 : 1012 : 10-2 Trabalhando com os expoentes, temos: 2 – 4 – (–5) – 12 – (–2) = –7 O resultado será 10–7

Propriedades dos logaritmos

As propriedades dos logaritmos são diretamente obtidas das propriedades das potências. Vamos considerar, nos exemplos a seguir, a sequência formada pelas potências de base 2:

20

1

21

2

22

4

23

8

4

2

16

25

32

26

64

7

2

128

28

256

29

512

210 1024 211 2048

Qual é o logaritmo em base 2 do produto de 16 por 64? O que se pede é o expoente a que devemos elevar 2 para obter o produto indicado. Em linguagem matemática: log2(16 . 64) = log2 1 024 Sabemos que: • 16 = 24 • 64 = 26 Então, 1 024 = 24 . 26 Pela propriedade das potências, para multiplicar potências de mesma base, basta somar os expoentes. Então, 1 024 = 210 Portanto, log2 1 024 = 10 Podemos generalizar: log b (a . c) = logb a + logb c No sentido inverso, a soma de logaritmos de mesma base é o logaritmo do produto dos logaritmandos: logb a + logb c = logb (a . c)

NA PRÁTICA LOGARITMO DE PRODUTO

Resolva a operação log2 (8 . 32). Pela propriedade do logaritmo de produto, log2 (8 . 32) = log2 8 + log2 32 • O expoente a que 2 deve ser elevado para se chegar a 8 é 3, pois 8 = 2 . 2 . 2. Então, log2 8 = 3 • E o expoente a que 2 deve ser elevado para se chegar a 32 é 5, pois 32 = 2 . 2 . 2 . 2 . 2. Portanto, log2 32 = 5. Então, log2 (8 . 32) = log2 256 = 3 + 5 = 8 O expoente a que 2 deve ser elevado para se chegar a 256 é 8: 28 = 256 GE MATEMÁTICA 2017

87


4

POTÊNCIA E LOGARITMO FUNÇÕES E EQUAÇÕES LOGARÍTMICAS

Logaritmo do quociente

Na divisão de potências de mesma base, o que fazemos com os expoentes é subtraí-los, certo? A mesma regra é válida para logaritmos. Acompanhe o exemplo: Se quisermos fazer a divisão 512 por 4, podemos tomar os logaritmos na base 2 para 512 (9) e de 4 (2) e subtraí-los (9 – 2 = 7). Assim, o resultado de 512 29 = = 2 9 - 2 = 2 7 = 128 4 22

Generalizamos em: a log b R c W = log b a - log b c

Logaritmo de potência

Vamos partir de logb a = c

ATENÇÃO

Se quisermos saber quanto vale logb a , podemos fazer, por conta da propriedade do produto de potências: n

logb an = logb a + logb a + logb a + ... + logb a n vezes

Temos então: logb an = n . c = n . logb a Ou: logb (an) = n . logb a

No sentido inverso, a subtração de dois logaritmos de mesma base é o quociente dos logaritmandos: a logb a – logb c = log b R c W

Quando o logaritmo e a potência têm a mesma base, o logaritmo é o expoente da potência: logbba = a Fácil de entender: na expressão acima, a é o número de vezes que b deve ser multiplicado por si mesmo (o expoente de uma potência de b), para que se obtenha o resultado a. Ora, como ba = ba, a resposta é a vezes.

NA PRÁTICA

NA PRÁTICA

LOGARITMO DE QUOCIENTE

LOGARITMO DE POTÊNCIA

27 Qual o resultado da operação log 3 S 81 X ?

Pela propriedade de logaritmo do quociente, temos:

a log b R c W = log b a - log b c

27 log 3 S 81 X = log 3 27 - log 3 81

• O expoente a que 3 deve ser elevado para chegarmos a 27 é 3, pois 27 = 3 . 3 . 3 Então, log3 27 = 3 • Já o expoente a que 3 deve ser elevado para chegarmos a 81 é 4, pois 81 = 3 . 3 . 3 . 3. Então, log3 81 = 4. 27 Portanto, log 3 S 81 X = 3 - 4 = -1

O expoente a que 3 deve ser elevado para conseguirmos o valor 27/81 é – 1: log 3 S 27 = - 1. 81 X

Qual o valor de log 7 2 401?

A primeira coisa a fazer é decompor 2 401 em números primos, pela fatoração. Basta dividir esse valor, seguidas vezes, por seus divisores que sejam números primos, em ordem crescente a começar do 2. Neste caso, o primeiro número primo que é divisor de 2 401 é 7. Então, Os números, para os dois lados da fatoração: 2 401 7 343 49 7 1

7 7 7

Portanto, 2 401 = 7 . 7 . 7 . 7 = 74 Faça a operação log7 74 Pela propriedade de logaritmo de potência, temos: logb na = n . logb a log7 74 = 4 . log7 7 log7 7 = 1 (ou seja, o expoente a que 7 deve ser elevado para resultar em 7 é 1: 7 = 71 Então, nossa operação fica assim: log7 74 = 4 . 1 = 4, pois 2 401 = 74

88 GE MATEMÁTICA 2017

Sempre que aparecer uma expressão de logaritmo em que a base não é escrita, considere que a base é 10. Assim: log 100 = log10 100 log 20 + log 57 = log10 20 + log10 57


Mudança de base

A partir de um conjunto de valores de logaritmos de certa base, podemos determinar os logaritmos de qualquer número em qualquer outra base. Na Antiguidade, vários matemáticos se preocuparam em obter uma tabela de logaritmos em base 10. Imagine que tenhamos acesso a uma dessas tabelas e queiramos calcular o logaritmo de 2 na base 3, a partir dos logaritmos de 2 e de 3 na base 10. São dados: log 2 = x log 3 = y

NA PRÁTICA A ESCALA RICHTER

Usada pela primeira vez pelo físico norte-americano Charles Richter, em 1935, a escala Richter é logarítmica – cada grau é uma potência de base 10. Ou seja, de um grau a outro, a amplitude das ondas sísmicas cresce dez vezes. Para comparar a diferença na magnitude de dois tremores, fazemos: M1 – M2 = log A1 – log A2 , em que:

Queremos calcular o valor de m: log3 2 = m

M1 = magnitude do terremoto 1

Dos dados, concluímos que 10x = 2 e 10y = 3

M2 = magnitude do terremoto 2 A1 = amplitude das ondas do terremoto 1

Queremos determinar m, tal que 3m = 2 Podemos substituir nessa última equação 3 e 2 por suas potências equivalentes em base 10. Assim: (10y)m = 10x 10y . m = 10x

A2 = amplitude das ondas do terremoto 2 Aplicando a propriedade de subtração de logaritmos: loga - logb = log S

A1 M 1 - M 2 = log S A X 2

Temos que y . m = x x

log2

Então, m = y = log3

A diferença de amplitude das ondas de um tremor de 5 pontos e de outro de 9 pontos na escala Richter: A1 5 - 9 = log S A X 2

Assim, podemos generalizar: log c a =

log b a log b c

A1 -4 = log S A X 2 A1 10 -4 = A 2

NA PRÁTICA MUDANÇA DE BASE

1 A1 = A 2 10 4

Calcule o log 9 81, mudando a base do logaritmo. Lembrando da fórmula: log b a =

log c a log c c

1 A1 10 000 = A 2

Vamos aproveitar que a base 9 e o logaritmando 81 são potências de 3. Então, temos log 9 81 =

a X , temos: b

log 3 81 log 3 9

A 2 = 10 000 . A 1

ATENÇÃO

• O expoente para elevar 3 a 81 é 4; • O expoente de 3 para se chegar a 9 é 2. 4

Portanto, temos: log 9 81 = 2 log 9 81 = 2

Fácil constatar que a resposta é verdadeira: 92 = 9 . 9 = 81

Não se preocupe: você não precisa saber de cor o valor dos logaritmos de todos os números. Valores quebrados são fornecidos no enunciado das questões da prova.

Isso significa que um tremor de 9 graus na escala Richter libera ondas de amplitude 10 000 vezes maior que um tremor de 5 graus.

GE MATEMÁTICA 2017

89


4

POTÊNCIA E LOGARITMO FUNÇÕES E EQUAÇÕES LOGARÍTMICAS

Função logarítmica

Função logarítmica é aquela que tem como forma f(x) = loga x, com a & 1 e a > 0. O domínio (D) de qualquer função logarítmica (as condições obrigatórias para que a função tenha solução) é o conjunto de números reais maiores que zero.

NA PRÁTICA

Gráfico de funções logarítmicas

Vamos comparar os gráficos de duas funções: y = log2 x e y = log1/2 x.

Repare que y corresponde a c (valor do logaritmo), 2 é a base b e x é o logaritmando a. Atribuindo alguns valores à variável x, calculamos seus logaritmos na base 2 e na base 1/2, completando as tabelas abaixo:

DOMÍNIO

Qual o domínio da função (fx) = log (x – 1) 4 – x? Nesta função, a = x – 1. Para que f(x) seja função, temos de atender às duas condições: •4–x>0→–x>–4→x<4 •x–1>0→x>1 •x–1≠1→x≠1+1→x≠2 O domínio é a intersecção dos dois conjuntos definidos por três três condições: x é um número • real maior que 1 • mas menor que 4 • e diferente de 2. Em linguagem matemática, D = { x ∈ R | 1 < x < 2 e 2 < x < 4} Repare que, para definir x ≠ 2, isolamos o 2 em duas desigualdades (x menor que 2 e x maior que 2).

x

y = log2 x

Ponto no plano cartesiano

1/2 1 2 4 8

-1 0 1 2 3

(1/2, -1) (1,0) (2,1) (4,2) (8,3)

x

y = log1/2 x

Ponto no plano cartesiano

1/2 1 2 4 8

1 0 -1 -2 -3

(1/2, 1) (1, 0) (2, -1) (4, -2) (8, -3)

4 y = log x 3

y y = log2 x

O logaritmo natural (ln) de um número é o logaritmo que tem como base o número de Euler (e ≈ 2,71828...). Este é um número irracional – um número real que não pode ser obtido da divisão de dois inteiros. O número de Euler (e) é também transcendental – não é raiz de nenhuma equação de coeficiente inteiro. O logaritmo natural obedece às propriedades operatórias dos logaritmos em geral. Assim, para resolver questões que trazem ln, basta usar os mesmos processos adotados para logaritmos de bases mais fáceis de lidar, como 2 ou 10.

90 GE MATEMÁTICA 2017

-2

-1

0

(1, 0) 0

1

-1 -2 -3

base 2

(0,5 , 1)

1

ATENÇÃO

(8, 3)

(4, 2)

2 (2, 1)

x

2 3 (2, -1)

(0,5 , -1)

4

5

6

7

8

9

base 1/2 (4, -2)

y = log1/2 x (8, -3)

Comparando as duas curvas: •Repare que em ambas, para todos os pontos, x > 0 e x & 0. Isso está de acordo com o domínio das funções logarítmicas. •Se x nunca é igual nem menor que zero, então as curvas se aproximam do eixo y sem jamais tocá-lo; •As curvas têm diferentes orientações: para y = log2 x, em que a base é maior que 1, a curva é crescente; já para y = log1/2 xb < 1, em que a base é menor que 1, a curva é decrescente. •As duas curvas se interceptam no ponto (1,0) que está presente nas duas curvas; isso porque o logaritmo de 1, em qualquer base, é zero.


Equações logarítmicas

São equações em que a incógnita está na base de um logaritmo ou em seu logaritmando. Veja nos dois exemplos: Em log 3 (5x + 1) = 4 , x está no logaritmando; Em log x (5x + 1) = 4, x é a base do logaritmo.

NA PRÁTICA EQUAÇÃO LOGARÍTMICA

Resolva a equação: log3(5x +1) = 4

NA PRÁTICA ENVOLVENDO EQUAÇÃO DE 2O GRAU

Uma equação logarítmica pode, ainda, envolver equações de segundo grau. Acompanhe: Resolva: log 2 Q x 2 + 2x - 7 V - log 2 Q x - 1 V = 2 Os logaritmos têm a mesma base (2). Então podemos aplicar a propriedade de log do quociente: log 2 S

2 x 2 + 2x - 7 X = 2 " x + 2x - 7 = 4 x-1 x-1

Multiplicando em cruz: x2 + 2x – 7 = 4x – 4 x2 – 2x – 3 = 0

Primeiro, vamos estabelecer o domínio dessa equação: 1 5x + 1 > 0 → x 2 - 5

Aplicando-se a definição de logaritmo temos que: 34 = 81 = 5x + 1 5x = 80 x = 16 1

Como 16 2 - 5 , a condição de existência está satisfeita, então S = {16}.

NA PRÁTICA COM LOGARITMOS DOS DOIS LADOS

A equação pode ter logaritmos dos dois lados da igualdade. Veja: log5(–x + 3) = log5 (x + 2) Como condição de existência de cada um dos logaritmos, temos: I) -x + 3 > 0 → -x > -3 → x < 3 II) x + 2 > 0 → x > -2 Juntando as duas condições, temos que a equação só terá solução se o valor que obtivermos for maior que -2 e menor que 3: -2 < x < 3 Como os dois logaritmos já estão na mesma base (5), igualamos os logaritmandos: -x + 3 = x + 2 -2x = -1 x=½ O valor obtido atende às duas condições de existência. Então, S = {1/2}.

Resolvendo essa equação de 2º grau, obtemos as raízes x1 = –1 e x2 = 3. Agora vamos verificar se x1 e x2 pertencem ao conjunto domínio, voltando à equação inicial: log 2(x2 + 2x –7) – log 2(x – 1) = 2 Para x1 = –1, temos log2 [(-1)2 + 2(–1) – 7] – log2 [(–1) – 1] = log2 (-8) – log2 (–2) Veja que obtivemos logaritmos de números negativos, que não existem. Portanto, o valor x1 = –1 não entra no conjunto-solução. Para x2 = 3, ficamos com log 2(32 + 2 . 3 – 7) – log2(3 – 1) = log2(8) – log2(2) = 3 – 1 = 2 Os logaritmandos são positivos e encontram-se no conjunto domínio. Então, S = {3}, ou seja, x = 3.

SAIU NA IMPRENSA OS TERREMOTOS DO EQUADOR E DO JAPÃO ESTÃO LIGADOS? A proximidade temporal entre o terremoto que devastou a ilha japonesa de Kyushu, na sexta-feira, e o tremor que causou pelo menos 413 mortos no Equador, um dia depois, levou muitos a associarem os dois eventos (...) A única coisa em comum entre os tremores do Japão e do Equador é que ambos ocorreram em abril e os dois causaram mortes (...) Os dois países estão localizados sobre o chamado Anel de Fogo do Pacífico (ou Cinturão de Fogo do Pacífico), mas o terremoto do Equador foi causado pela subducção da placa tectônica de Nazca, que deslizou para baixo da placa da América do Sul. O tremor no Japão, que deixou pelo menos 42 mortos, teve origem em uma falha superficial localizada em uma placa tectônica completamente diferente, a placa Euroasiática (...) As estatísticas também mostram que a coincidência do último fim de semana sequer foi algo extraordinário. De acordo com a agência geológica dos Estados Unidos (USGS na sigla em inglês), dados registrados desde 1900 até hoje mostram que há, em média, 16 grandes terremotos por ano. BBC, 19/4/2016 GE MATEMÁTICA 2017

91


4

COMO CAI NA PROVA

1.(UFPR 2016) A análise de uma aplicação financeira ao longo do tempo

mostrou que a expressão V(t) = 1000 . 2 0,0625 t fornece uma boa aproximação do valor V (em reais) em função do tempo t (em anos), desde o início da aplicação. Depois de quantos anos o valor inicialmente investido dobrará?

logb a =

1 loga b

Com isso, temos log2 2016 =

a) 8

b) 12

c) 16

d) 24

e) 32

1 log2016 2 1

RESOLUÇÃO

log3 2016 =

Observe que a expressão matemática para o valor de uma aplicação em função do tempo é uma função exponencial. Isso significa que a variável independente, o tempo (t), aparece no expoente.

log7 2016 =

No início da aplicação, t = 0. Como qualquer valor elevado a zero é igual a 1, então sabemos que para o momento em que a aplicação é feita, temos V(0) = 1 000. Este é o valor aplicado.

Com todos os logaritmos na mesma base, fazemos a soma: +1 . 1 +1 . 1 S= 1 . 1 2 log2 2016 5 log3 2016 10 log7 2016

O enunciado pede que se determine o tempo necessário para que esse valor dobre, ou seja, atinja o valor de 2 000. Substituindo esse valor, na função, temos: 2 000 = 1 000 . 2 0,0625 t Dividindo os dois lados da expressão por 1 000, ficamos com 21 = 20,0625t

S = 1 . log2016 2 + 1 . log2016 3 + 1 . log2016 7 2 5 10

Potências iguais com bases iguais têm, obrigatoriamente, expoentes iguais. Assim, 1 = 0,0625 t t = 1 /0,0625 → t = 16 Resposta: C

2. (Fuvest 2016) Use as propriedades do logaritmo para simplificar a expressão S=

1 + 1 5 . log3 2016 2 . log2 2016

+

1 10 . log7 2016

O valor de S é a) 1 2 b) 1 3

e) 1 10

RESOLUÇÃO Os três logaritmos apresentados na expressão do enunciado estão em bases diferentes, o que dificulta bastante qualquer movimento. Entretanto, pela propriedade da mudança de base, podemos mudar todas as bases dos logaritmos para 2016. Acompanhe: Pela propriedade da mudança de base, temos que: logca logcb

Da mesma forma, logab =

Aplicando as propriedades dos logaritmos: 5 2 S = 1 . (log2016 2 + log2016 3 + log2016 7) 10 5 2 S = 1 .log2016 (2 . 3 . 7) 10 Como 2016 = 25 . 3² . 7, então 5 2 S = 1 .log2016 (2 . 3 . 7) = 1 . log2016 2016 = 1 . 1 = 1 10 10 10 10

Resposta: E

3. (UPF 2016) Se 24 b) –1

= 3 n+1 . 16 , então log3 n é igual a: c) 1 d) 1 e) 2 2

n+1

RESOLUÇÃO

d) 1 7

logb a =

1 log2016 7

Colocando 1/10 em evidência, temos S = 1 . (5 . log2016 2 + 2 . log2016 3 + log2016 7) 10

a) –2

c) 1 5

log2016 3

logcb logca

Repare que uma expressão é exatamente a inversa da outra. Assim, concluímos que

92 GE MATEMÁTICA 2017

Repare que 24 = 3 . 8 = 3 . 2³ Então, podemos substituir 24 n + 1 por (3 . 23) n + 1 A igualdade do enunciado pode ser escrita assim: (3 . 23)n+1 = 3n+1 . 16 Aplicando a propriedade que afirma que a potência do produto é igual ao produto das potências, temos 3 n+1 . (23) n+1 = 3 n+1 . 24 Cancelando o termo que aparece dos dois lados da equação (3 n+1), e aplicando a propriedade da potência de potência, chegamos a 2 3n+3 = 24 Potências de bases iguais, para serem iguais, devem ter os mesmos expoentes, então 3n+3=4 → 3n=1 n = 1/3 Mas, atenção, o enunciado pede o logaritmo de n na base 3. Ou seja, o expoente aqui a que a base 3 deve ser elevada para que se obtenha 1/3. Ou seja, o problema pede o expoente a que a base 3 deve ser elevada para que se obtenha 1/3. Assim temos : log3 1/3 = –1 Resposta: B


RESUMO

4. (UFPR 2016) Considere o gráfico da função f(x) = log x e a reta r que passa 2

pelos pontos A e B, como indicado na figura abaixo, sendo k a abscissa do ponto em que a reta r intersecta o eixo Ox. Qual é o valor de k?

NOTAÇÃO CIENTÍFICA n = a . 10x, em que 1 ≤ a < 10

y r A

f(x) = log2x

0

x

0,25 k

Potência e logaritmo

2

POTENCIAÇÃO É a multiplicação de um fator por ele mesmo, várias vezes seguidas: 103 = 10 . 10 . 10 = 1 000 84 = 8 . 8 . 8 . 8 = 4 096 Em ab = c a é a base; b é o expoente; c é a potência, ou seja, o resultado da operação. Os valores a, b e c podem ser quaisquer números reais.

B

PROPRIEDADES DE POTÊNCIA

a) 17/12 b) 14/11 c) 12/7 d) 11/9 e) 7/4

• am : an = am – n

• (am)n = am . n

1 b -b • a =SaX

• a

RESOLUÇÃO Analisando o gráfico, você percebe que k corresponde à raiz da função de 1º grau que define a reta r, que passa pelos pontos A e B. Esses pontos A e B, por sua vez, também pertencem ao gráfico da função y= log2 x. Conhecemos as abscissas dos dois pontos: 0,25 para B e 2 para A. Substituindo esses valores na função logarítmica, determinamos a ordenada de cada ponto (yA e yB ): Ponto A (2; yA ) → yA = log2 2 = 1 Ponto B (0,25; yB ) → yB = log2 0,25 = log2 1/4 = –2 Portanto, r passa pelos pontos A (2; 1) e B (0,25; –2). Para determinar o valor de k temos, ainda, de calcular a declividade da reta r. Retas são definidas por equações do tipo y = ax + b. Encontrando o valor de a: a = ∆y = – 2 – 1 = – 3 = 12 ∆x 0,25 – 2 – 1,75 7 A equação da reta é y = 12 x + b 7 Substituindo as coordenadas do ponto A, por exemplo: 1 = 12 . 2 + b 7 b = 1 – 24 = – 17 7 7 A equação da reta r é y = 12 x – 17 7 7

m n

=

n

am

b b • R m W = mb n n

• (m . n )b = mb . nb • a0 = 1, desde que a ≠ 0

FUNÇÃO EXPONENCIAL f(x) = m . ax, em que: • f(x) é a variável dependente • x é a variável independente • m é um número real qualquer • a é um número real maior que zero e diferente de 1 LOGARITMO é a operação matemática para encontrar o expoente de uma potência. O logaritmo de a na base b é c, se b elevado ao expoente c é igual a a. Em linguagem matemática: logba = c bc = a, sendo que •aeb>0 •bé≠1

)

LOGARITMO DO PRODUTO logb (a . c) = logb a + logb c LOGARITMO DO QUOCIENTE

a log b R c W = log b a - log b c

E sua raiz é: 0 = 12 x – 17 7 7 12 x = 17 7 7 12x = 17 → x = 17 12

• am . an = am + n

1 log b S a X = -log b a

LOGARITMO DE POTÊNCIA • log b (bn) = n • log b (an) = n . log ba MUDANÇA DE BASE DO LOGARITMO

Este é o valor de K

log c a =

log b a log b c

Resposta: A GE MATEMÁTICA 2017

93


5

TRIGONOMETRIA CONTEÚDO DESTE CAPÍTULO

Triângulos e a circunferência trigonométrica ......................................96 Funções trigonométricas ............................................................................100 Como cai na prova + Resumo .....................................................................102

Os danos colaterais do aeromodelismo bélico Os drones, cada vez mais usados em conflitos armados, reduzem as baixas nos exércitos atacantes. Mas fazem também muitas vítimas entre civis inocentes

N

o início de julho de 2016, o presidente dos Estados Unidos (EUA), Barack Obama, apresentou à imprensa um balanço sobre o número de vítimas de ataques de drones em conflitos armados no Oriente Médio, entre 2009 e 2015: afora as ações no Afeganistão e no Iraque, a força militar de veículos aéreos não tripulados norte-americana matou algo entre 2.372 e 2.581 terroristas e militantes de movimentos contra governos. Segundo a Casa Branca, os ataques também levaram a vida de civis. Mas, no mesmo período, foram, no máximo, 116 pessoas atingidas por engano. A opinião pública nos EUA aprova os ataques remotos, que poupam vidas no Exército. Mas diversas organizações não governamentais de direitos humanos questionam a precisão dos ataques e os dados apresentados por Obama. Segundo elas, é impossível aferir os números exatos, mas estima-se que os robozinhos militares tenham matado milhares de inocentes. Os EUA já desenvolviam e testavam drones militares ainda nos anos 1960, durante a Guerra do Vietnã. E seu uso ganhou papel especial depois do ataque terrorista às Torres Gêmeas, em Nova York, em 2001. Originalmente, a tecnologia foi desenvolvida com um primeiro objetivo muito preciso: a busca e eliminação de Osama bin Laden,

94 GE MATEMÁTICA 2017

que assumiu os ataques do Onze de Setembro. Mas seu uso foi estendido para conflitos na África e no Oriente Médio. Além dos EUA, pelo menos outros seis países admitem usar drones em combates: Israel, Reino Unido, Irã, Iraque, Paquistão e Nigéria. Mais de 20 países exportam a tecnologia para cerca de 80 nações (além da venda a grupos como Estado Islâmico e rebeldes sírios). Sem as bombas, as pequenas naves não tripuladas têm aplicações pacíficas, por exemplo, na fotografia para a elaboração de mapas, ou na vistoria de áreas de preservação ambiental. Este é um mercado que cresce ano a ano. Em 2013, o setor movimentou mais de 6 bilhões de dólares – um valor que deve dobrar até a próxima década. A tecnologia inteligente dos drones não seria possível sem outra tecnologia, a do Sistema de Posicionamento Global (GPS), que permite ao apa- POR QUÊ? relho localizar com O grafite num muro precisão um objeto em Sanaa, capital do na superfície da Terra. Iêmen, com a pergunta O GPS trabalha com que denuncia a ação de ângulos e triângulos. drones norte-americanos: Estes são os principais por que mataram temas deste capítulo. minha família?


GE MATEMÁTICA 2017

95

KHALED ABDULLAH/REUTERS


5

TRIGONOMETRIA TRIÂNGULOS E A CIRCUNFERÊNCIA TRIGONOMÉTRICA

ELENABS/iSTOCK

BÚSSOLA DE ÂNGULOS O sistema GPS localiza a posição exata de um objeto na superfície por meio da relação entre triângulos

Régua para triângulos A circunferência trigonométrica facilita os cálculos que envolvem ângulos de triângulos retângulos

O

triângulo e suas medidas são úteis em diversas áreas do conhecimento. Na arquitetura, para definição de dimensões como altura de um edifício; na cartografia, para o desenho, em escala, de mapas; na astronomia, para cálculo de distâncias e posições relativas dos astros; e na engenharia ambiental, para avaliar os estragos de um vazamento de petróleo no mar. As relações entre triângulos são necessárias também para o sistema de posicionamento global (GPS). A localização de um ponto qualquer na superfície da Terra

96 GE MATEMÁTICA 2017

é feita por um método chamado triangulação. Os satélites do GPS em órbita da Terra e os receptores na superfície carregam relógios ultraprecisos. Os satélites emitem sinais e os relógios medem o tempo decorrido entre a emissão e a recepção. Sinais eletromagnéticos viajam à velocidade da luz – 300 mil quilômetros por segundo –, e a distância entre dois pontos é igual à velocidade multiplicada pelo tempo. Assim é medida a distância entre cada satélite e o receptor. Associando a essa medida a distância do receptor a outros satélites, definem-se triângulos cujas relações indicam a posição exata do objeto na superfície.

respondentes são proporcionais. Por consequência, as áreas dos triângulos também são proporcionais. Acompanhe o raciocínio observando a figura abaixo:

Triângulos semelhantes

• Seus ângulos correspondentes são congruentes (têm medida igual). Na figura, o ângulo α é correspondente de α’, β é correspondente de β’ e γ, de γ ’.

Dois triângulos são semelhantes quando seus ângulos e lados correspondentes mantêm uma razão de proporção. Ângulos correspondentes são aqueles que ocupam a mesma posição em relação aos lados de um triângulo. Lados correspondentes são aqueles que ocupam a mesma posição em relação aos ângulos. Então, em triângulos semelhantes os ângulos e lados cor-

D a DÖ aÖ B

_

`

C

`Ö

É facil perceber que os triângulos ABC e A’B’C’ são parecidos. Eles são triângulos semelhantes por duas razões:

• Seus lados correspondentes são proporcionais. Na figura, você observa: • o lado AC é correspondente de A’C’ • o AB é correspondente de A’B’ • o lado BC é correspondente de B’C’.


Razão de semelhança

Ainda observando os dois triângulos da página anterior, repare que o lado AB mede 6 unidades (6 quadradinhos). E seu correspondente A’B’ mede 3 unidades. A razão de semelhança entre esses lados correspondentes, então, é k=

Triângulos semelhantes sempre têm três ângulos congruentes. Acompanhe na figura abaixo o que acontece quando essa semelhança ocorre entre dois triângulos retângulos (aqueles que têm um ângulo de 90o): A

AB 6 = =2 A’B’ 3

A razão se mantém para os demais pares de lados correspondentes: k=

AC 2 5 4 = =2 e k= A’C’ 5 2

2 =2 2

Dizemos, então, que o triângulo AB está para A’B’ na escala de 2 para 1 (em notação matemática, 2 : 1). É esse tipo de escala que permite que se mantenham as proporções em mapas – e que se saiba com bastante precisão o real tamanho da mancha de petróleo no Golfo do México. Triângulos semelhantes também têm áreas semelhantes. Se a razão de semelhança entre os lados vale k, a razão entre as áreas é k2. A razão de semelhança se mantém também para as demais medidas lineares do triângulo. Entre alturas: D a

i _

`

B

B’ B

C A’

Veja que os triângulos ABC e A’B’C têm um ângulo de 90o (em B e B’). Os dois também têm um vértice em comum (C). Então, os ângulos definidos por esse vértice em cada triângulo são congruentes. A soma dos ângulos internos de um triângulo é 180º. Então, se os dois primeiros ângulos são congruentes, o último também será congruente. Conclusão: os triângulos ABC e A’B’C são semelhantes.

A circunferência trigonométrica

A trigonometria é uma ferramenta para calcular medidas e proporções entre triângulos retângulos. A base para isso é a circunferência trigonométrica.

C

iÖ `Ö

Podemos desenhar na circunferência ângulos de 0° a 360° e obter o valor das razões trigonométricas. Acompanhe atentamente na figura: • O segmento OA é a hipotenusa de um triângulo retângulo formado pelos pontos OAP; • O raio da circunferência é uma unidade. Então, o segmento OA (a hipotenusa) vale uma unidade; • A hipotenusa forma com o lado positivo do eixo x um ângulo α; • O ponto A tem coordenadas (x, y). O eixo x é o eixo dos cossenos e o y, eixo dos senos. Então, as coordenadas de um ponto qualquer A são (cos α, sen α). Esta relação vale para qualquer ponto da circunferência.

Os quadrantes

Dependendo do quadrante em que o ponto A se encontra, os valores do seno, cosseno e tangente serão positivos ou negativos. Acompanhe: Quando o ponto A está no quadrante I, o ângulo α terá valor entre 0o e 90o. Veja que, nessa faixa de ângulos, todos os valores de x e y são positivos. Portanto, o seno, o cosseno e a tangente de ângulos no quadrante I são positivos. E os valores do seno, do cosseno e da tangente serão positivos:

aÖ _Ö

• O segmento em azul é a tangente do ângulo α (tg α); • A circunferência é dividida em quatro quadrantes (I, II, III, e IV); • Os graus são lidos a partir da direita, no sentido anti-horário: 0°, 90°, 180°, 270° e 360°.

h 4 = =2 h’ 2 Entre medianas: k=

90º sen

k=

m 17 = =2 m’ 17 2

Observe que: • A circunferência é desenhada sobre um plano cartesiano (eixos x e y); • O eixo x corresponde à medida do cosseno (cos); • O eixo y corresponde à medida do seno (sen); • O centro da circunferência está sobre o ponto O, de coordenadas (0,0); • O raio da circunferência é uma unidade;

1

tg I

R

A tg _

1

sen _ _ O

cos _

0º cos 1 360º P

GE MATEMÁTICA 2017

97


5

TRIGONOMETRIA TRIÂNGULOS E A CIRCUNFERÊNCIA TRIGONOMÉTRICA

Se o ponto A estiver no quadrante II, o ângulo α terá como medida um valor entre 90° e 180°. Veja na figura abaixo: o valor de y (seno) é positivo, mas x (cosseno) é negativo; a tangente, portanto, também é negativa:

Para um ponto A que esteja no quadrante III, o valor de α ficará entre 180º e 270°. O seno e o cosseno serão negativos. A tangente será positiva:

Pitágoras e a trigonometria

Se aplicarmos o Teorema de Pitágoras num triângulo desenhado no 1º quadrante da circunferência trigonométrica, teremos:

• A hipotenusa, que é o raio da circunferência trigonométrica, tem medida 1. Então sen2 α + cos2 α = 1

Ainda no triângulo AOP, podemos calcular a tangente do ângulo α e obter outra relação: tg α =

sen α cos α

Razões trigonométricas em um triângulo retângulo

Com a trigonometria, estabelecemos as razões de semelhança entre triângulos retângulos, associando a medida dos lados à circunferência trigonométrica. As razões trigonométricas são definidas com base nas relações entre as medidas dos lados do triângulo e têm como referência os ângulos. sen α =

cateto oposto a α hipotenusa

cos α =

cateto adjacente a α hipotenusa

tg α =

98 GE MATEMÁTICA 2017

Podemos calcular o valor para um ângulo α de qualquer quadrante trabalhando apenas com ângulos do quadrante I. É que qualquer ponto da circunferência tem três pontos simétricos em relação aos eixos cartesianos nos outros três quadrantes. Veja:

• Os dois catetos do triângulo AOP são os valores de seno e cosseno do ângulo α;

Esta é a chamada relação fundamental da trigonometria.

Finalmente, para um ponto que esteja no quadrante IV, α estará entre 270° e 360°. Nesse caso, o cosseno é positivo e o seno e a tangente, negativos:

Simetria na circunferência trigonométrica

cateto oposto a α cateto adjacente a α

Os pontos A, A’, A’’ e A’’’ são simétricos em relação aos eixos cartesianos. Traduzindo: as coordenadas desses pontos têm os mesmos valores absolutos. A única diferença são os sinais, que variam conforme o quadrante. Portanto, os ângulos definidos por esses pontos têm seno e cosseno iguais, só variando os sinais, conforme o quadrante. Essa simetria é muito útil quando precisamos trabalhar com triângulos que contenham um ângulo maior que 90º.

Ângulos complementares e suplementares

Ângulos complementares são dois ângulos (α e β) que, somados, resultam em 90o. Ângulos complementares apresentam algumas propriedades: Se α + β = 90o, então: • sen α = cos β • cos α = sen β • tg α = 1 tg β Os ângulos α e β são suplementares quando sua soma resultar em 180°. Se α + β = 180°, então: • sen α = sen β • cos α = –cos β • tg α = –tg β


Lei dos senos e cossenos

Para triângulos que não são retângulos (chamados acutângulos ou obtusângulos), duas outras relações são muito importantes. São as leis dos senos e dos cossenos. Observe o triângulo obtusângulo abaixo: D a

NA PRÁTICA

NA PRÁTICA

RELAÇÕES TRIGONOMÉTRICAS

ÂNGULOS MAIORES QUE 360O

Considere o triângulo retângulo ABC, abaixo, de catetos medindo 4 e 6 unidades e o ângulo reto em C. Qual a medida dos ângulos α e β? Dados: Desenhando a situação descrita:

b

c

B

Calcule o seno, o cosseno e a tangente do ângulo de 1 230°. O ângulo de 1 230o é bem maior que os 360o de uma volta completa na circunferência. Para verificar essa diferença no tamanho, dividimos 1 230 por 360: 1 230 : 360 = 3,41...

a

Então, 1 230o correspondem a mais de três voltas na circunferência. Três voltas na circunferência são 360o . 3 = 1080o

C

Quanto falta ainda para chegar aos 1 230o? 1 230 – 1 880 = 150o

` _

`

d

B

c C

A lei dos senos estabelece que: a = b = c sen α sen β sen γ

_ b

A

E a lei dos cossenos, que: a2 = b2 + c2 – 2 . b . c . cos α Repare que, nesta formulação, α é o ângulo oposto ao lado a. Da mesma maneira, podemos estabelecer a lei dos cossenos para os demais ângulos: b2 = a2 + c2 – 2 . a . c . cos β , em que β é o ângulo oposto ao lado b; c2 = a2 + b2 – 2 . a . b . cos γ , em que γ é o ângulo oposto ao lado c. Num triângulo qualquer, a razão entre a medida de um lado e o seno do ângulo oposto a esse lado tem valor igual ao diâmetro da circunferência que circunscreve o triângulo. O diâmetro éd=2.r

Por Pitágoras, encontramos o valor da hipotenusa (c): c2 = a2 + b2 c2 = 42 + 62 = 16 + 36 c = 52 • sen α = a/c = 4 / 2 13 = 2 / 13 = 0,55 • cos α = b/c = 6 / 2 13 = 3 / 13 = 0,83 • tg α = a/b = 4 / 6 = 0,66

Então, o seno, o cosseno e a tangente de 1 230° são iguais ao seno, ao cosseno e à tangente de 150°. sen 1 230° = sen 150° = 1/2

Existe um único ângulo que combina esses valores de seno, cosseno e tangente. Consultando uma tabela de valores trigonométricos, ou utilizando uma calculadora científica, descobrimos que α mede, aproximadamente, 34o.

A tangente é calculada dividindo-se o sen pelo cos:

3

cos 1 230° = cos 150° = – 2

tg 1 230 o =

sen 1 230 o cos 1 230 o 1 2

o

tg 1 230 =

Os três ângulos internos de um triângulo somam 180o. Um desses ângulos mede 90o; outro, 34o. Então, o terceiro ângulo (β) mede: 180o – 90o – 34o β = 56o.

3 - 2

3 tg 1 230 o = - 3

VALORES TRIGONOMÉTRICOS PARA ÂNGULOS NOTÁVEIS C

A

b

a

c

c

a

r

b B

Então, na lei dos senos, temos que a = b = c =2.r sen α sen β sen γ

Vale a pena decorar os valores de seno, cosseno e tangente dos ângulos mais importantes 0º

30º

45º

60º

90º

180º

270º

360º

O rad

r 6

r 4

r 3

r 2

r

3r 2

2r

sen α

0

1 2

2 2

3 2

1

0

–1

0

cos α

1

3 2

2 2

1 2

0

–1

0

1

tg α

0

3 3

1

3

0

0

α

GE MATEMÁTICA 2017

99


TRIGONOMETRIA FUNÇÕES TRIGONOMÉTRICAS

JPL-CALTECH/NASA

5

JATOS CÓSMICOS A radiação de alta energia liberada por buracos negros é estudada com o uso de conceitos da trigonometria

As equações de onda O que representam as curvas no gráfico das funções trigonométricas

N

a astronomia, as funções trigonométricas são instrumento importante para o estudo de características como a temperatura e o tipo de radiação emitido por um corpo celeste. São funções que permitem aos astrônomos avaliar a energia liberada por um astro, seja na forma de luz ou de calor, seja na forma de radiação invisível, como raios X ou raios gama. Os

100 GE MATEMÁTICA 2017

buracos negros, por exemplo, corpos extremamente densos, cuja gravidade atrai tudo em sua volta, emitem de vez em quando jatos de radiação de alta energia, como raios X e gama. Funções trigonométricas são senoides – ondas nas quais os valores de y se repetem periodicamente, ao longo do eixo x, no domínio da função. O conjunto domínio (D) de uma função trigonométrica é o conjunto dos arcos ou ângulos que podem ser expressos por qualquer número real, positivo, negativo ou nulo, seja em graus ou em radianos, não importa. O fato de serem funções periódicas indicam que os valores das funções seno, cosseno e tangente se repetem em intervalos regulares. Esses intervalos são chamados períodos.


Função seno: y = sen(x)

Acompanhe no gráfico abaixo: • O eixo x traz a medida do ângulo, em radianos; • O eixo y traz o valor do seno do ângulo x; • O seno de qualquer ângulo está sempre no intervalo entre -1 e 1. Então, esses são os pontos máximo e mínimo, ao longo da senoide; • A amplitude dessa função – ou seja, os valores que y pode assumir – estão no intervalo real [-1, 1]. Amplitude é a medida de quanto o valor de y varia acima ou abaixo do eixo x. • O período da função seno é 2π. Veja que a partir do ponto (0,0), indo para a direita do eixo, os valores começam a se repetir em 2π ou em -2π. 1

-7π/2 -3π

-5π/2

-2π -3π/2

0

-π/2

SAIBA MAIS ESPECTRO ELETROMAGNÉTICO

A luz visível ocupa apenas uma estreita faixa do chamado espectro eletromagnético – a régua que indica o comprimento das ondas de forças eletromagnéticas. Além da luz visível, o espectro inclui radiação invisível aos olhos humanos, como raios X, ultravioleta e infravermelho.

sen x 0

π/2

Ultravioleta

π

3π/2

2π 5π/2 3π

x (rad) 4π 7π/2

Raios gama

Raios X

Infravermelho Micro-ondas Rádio

-1

Função cosseno: y = cos(x)

Acompanhe, mais uma vez, no gráfico abaixo: • O eixo x traz a medida do ângulo x, em radianos; • O eixo y traz o valor do cosseno de x; • Assim como o seno, o cosseno de qualquer ângulo está sempre no intervalo entre -1 e 1. Então, esses são os pontos máximo e mínimo, ao longo da senoide; • A amplitude da função cosseno também é 1, porque os valores que y pode assumir também variam no intervalo [-1,1] e, portanto, jamais distam mais do que uma unidade do eixo x. • O período da função cosseno também é 2π. Veja que a partir do ponto (0,0), indo para a direita do eixo, os valores começam a se repetir em 2π ou em -2π. 1 cos x

-7π/2

-3π

-5π/2 -2π -3π/2

-π/2

0

0 π/2

π

3π/2 2π

-1

5π/2

x (rad) 7π/2 4π

Luz visível

ATENÇÃO A função tg não é definida para x com valores como -3π/2, -π/2, π/2, 3π/2. Fácil entender porque, lembrando da fórmula da tangente: tg = sen α / cos α. Uma fração jamais pode ter 0 no denominador. Portanto, ângulos de cos = 0 não têm tangente definida.

As curvas parecem iguais. Mas repare que o gráfico da função cosseno está deslocado de π/2 na horizontal em relação ao gráfico da função seno. As funções seno e cosseno são alternadamente crescentes e decrescentes, dependendo do intervalo de x considerado.

Função tangente: y = tg(x) Acompanhe abaixo:

1

-7π/2

-3π

-5π/2

-2π -3π/2

-π/2

0 -1

tg x x (rad) 0

π/2

π

3π/2

5π/2

7π/2

O gráfico da função tangente é sempre crescente, e seu período é π. A amplitude é infinita, pois y pode variar indefinidamente, no sentido positivo ou negativo. GE MATEMÁTICA 2017

101


5

COMO CAI NA PROVA

1. (UEG 2016) Sabendo-se que sen (x) = 1/2 e que x é um ângulo do 1º qua-

drante, o valor da expressão sen (4x) – cos (4x) é a) √3 – 1 b) 1 c) √3 + 1 2 2 2

d) 2

RESOLUÇÃO Segundo o enunciado, tg x = 3 = sen x cos x Portanto, sen x = 3 . cos x Do teorema de Pitágoras aplicado à circunferência trigonométrica, sabemos que cos2 a + sen2 a = 1

RESOLUÇÃO Pelos dados do enunciado determinamos o valor de x. Se x é um ângulo do 1º quadrante, 0° < x < 90°. Se o sen x = 1/2 , então x = 30°. Assim, 4 . x = 4 . 30° = 120°

Se sen x = 3 . cos x, então reescrevemos essa expressão para: cos² x + (3cos x)² = 1 cos² x + 9 cos² x = 1 10 cos² x = 1 cos2 x = 1 � cos x = √10 ou cos x = – √10 10 10 10

Sabemos que: sen 120º = sen 60º = √3 2 cos 120º = – cos 60º = – 1 2 Substituindo esses valores na expressão dada no enunciado, temos: sen (4x) – cos (4x) = sen 120º – cos 120º = √3 – – 1 = √3 + 1 2 2 2 Resposta: C

2. (IFPE 2016) Na cidade de Recife, mesmo que muito discretamente, devido

à pequena latitude em que nos encontramos, percebemos que, no verão, o dia se estende um pouco mais em relação à noite e, no inverno, esse fenômeno se inverte. Já em outros lugares do nosso planeta, devido a grandes latitudes, essa variação se dá de forma muito mais acentuada. É o caso de Ancara, na Turquia, onde a duração de luz solar L, em horas, no dia d do ano, após 21 de março, é dada pela função: L (d) = 12 + 2,8 . sen 2π (d – 80) 365 Determine, em horas, respectivamente, a máxima e a mínima duração de luz solar durante um dia em Ancara. a) 12,8 e 12 b) 14,8 e 9,2 c) 12,8 e 9,2 d) 12 e 12 e) 14,8 e 12

O enunciado diz que x é um dos dois ângulos agudos. Portanto, x está no primeiro quadrante. E todos os ângulos deste quadrante têm cos positivo. Portanto, dos dois valores ficamos com cos x = √10 . 10 Resposta: E

4. (Fuvest 2016) Quando a Lua está em quarto crescente ou quarto minguante,

o triângulo formado pela Terra, pelo Sol e pela Lua é retângulo, com a Lua no vértice do ângulo reto. O astrônomo grego Aristarco, do século III a.C., usou este fato para obter um valor aproximado da razão entre as distâncias da Terra à Lua, dL , e da Terra ao Sol, dS. Crescente t2 a

Terra t1

Sol

ds

a

dL Minguante

Representação simplificada com Terra e Sol fixos.

RESOLUÇÃO A questão parece complexa, mas exige apenas atenção. Analisando a expressão matemática da função que fornece a duração da luz solar em Ancara em relação ao dia do ano: L (d) = 12 + 2,8 . sen 2π (d – 80) 365 Repare que essa função tem um fator que inclui o seno de um ângulo. Sabemos que o valor do seno de qualquer ângulo varia no intervalo [–1, 1]. Assim, o fator sen 2π (d – 80) vale, no máximo, 1 e no mínimo, –1. 365

É possível estimar a medida do ângulo a, relativo ao vértice da Terra, nessas duas fases, a partir da observação de que o tempo t1 , decorrido de uma lua quarto crescente a uma lua quarto minguante, é um pouco maior do que o tempo t2 , decorrido de uma lua quarto minguante a uma lua quarto crescente. Supondo que a Lua descreva em torno da Terra um movimento circular uniforme, tomando t1 = 14,9 dias e t2 = 14,8 dias, conclui-se que a razão dL /dS seria aproximadamente dada por a) cos 77,7° b) cos 80,7° c) cos 83,7° d) cos 86,7° e) cos 89,7°

RESOLUÇÃO Então, a função se resume a: L(d) = 12 + 2,8 . 1 ou L(d) = 12 + 2,8 . (–1) • Para sen = 1, temos: L(d) = 12 + 2,8 = 14,8 horas de luz solar. • Para sen = –1, temos: L(d) = 12 – 2,8 = 9,2 horas de luz solar. Resposta: B

3. (UPE 2015) Num triângulo retângulo, temos que tg x = 3. Se x é um dos

ângulos agudos desse triângulo, qual o valor de cos x? a) 1 b) √5 c) √2 d) 1 2 10 2 4

102 GE MATEMÁTICA 2017

e) √10 10

Vamos calcular a velocidade angular da Lua nos dois trechos assinalados na figura: v1 entre o quarto crescente e o quarto minguante (relativo ao tempo t1 ) e v2 entre o quarto minguante e o quarto crescente (relativo ao tempo t2). Para isso, fazemos a razão entre os respectivos ângulos percorridos e o tempo tomado em cada um desses trechos. A Lua percorre uma circunferência em torno da Terra – ou seja, 360º. Se no trecho percorrido em v2 o ângulo é 2 a, então para o trecho em v1 , o ângulo é 360°– 2 a. Com o valor de t1 e t2 fornecidos no enunciado, temos: v1 = 360º – 2a = 2a = v2 14,9 14,8 14,8 (360° – 2a) = 14,9 . 2 a → 5 328 – 29,6 a = 29,8 a 5 328 = 59,4 a → a = 89,69° Este é o ângulo a assinalado na imagem.


RESUMO

Nos triângulos retângulos formados pela Terra, o Sol e a Lua, a razão dL /dS equivale ao cosseno do ângulo a, ou seja dL /dS = cos a = cos 89,7°. Resposta: E

5. (Unicamp 2016) Considere o triângulo exibido na figura abaixo, com lados de comprimentos a, b e c e ângulos a, b e c. b

TRIÂNGULOS SEMELHANTES têm ângulos correspondentes congruentes (de valor igual) e seus lados correspondentes proporcionais. Nos triângulos semelhantes, as demais medidas lineares (como altura e medianas) são também proporcionais. TEOREMA DE PITÁGORAS Num triângulo retângulo qualquer, c2 = a2 + b2, em que c é a hipotenusa, a e b são os catetos.

a

c

Trigonometria

CIRCUNFERÊNCIA TRIGONOMÉTRICA a

c

b

a) Suponha que a sequência (a, b, c) é uma progressão aritmética (PA). Determine a medida do ângulo b. b) Suponha que a sequência (a, b , c) é uma progressão geométrica (PG) de razão q = √2 . Determine o valor de tg b. Quadrante I: sen > 0, cos > 0 Quadrante II: sen > 0, cos < 0 Quadrante III: sen < 0, cos < 0 Quadrante IV: sen < 0, cos > 0

RESOLUÇÃO a) Lembrando o conceito de PA: uma sequência de valores na qual a diferença entre valores subsequentes é sempre a mesma (razão r). Se o valor de r é sempre o mesmo, então podemos escrever a PA da seguinte maneira: ( b – r, b, b + r). Sabemos que a soma dos ângulos internos de qualquer triângulo é 180°. Portanto, podemos estabelecer a relação b – r + b + b + r = 180° 3b = 180° → b = 60°. Resposta: b = 60° b) Lembrando o conceito de PG: uma sequência de valores na qual o quociente entre um valor e o valor subsequente é sempre o mesmo (razão q). Se os lados a, b, c estão em PG, então b/a = c/b = q. O enunciado informa que a razão q da PG vale q = √2. Podemos, então, escrever a PG da seguinte maneira: (a, √2a , 2a).

ÂNGULOS COMPLEMENTARES (α + β = 90o): • sen α = cos β • cos α = sen β • tg α = 1 tg β ÂNGULOS SUPLEMENTARES (α + β = 180o): • sen α = sen β • cos α = –cos β • tg α = –tg β RAZÕES TRIGONOMÉTRICAS NO TRIÂNGULO RETÂNGULO

Veja na figura que o ângulo b é oposto ao lado b, e que b = √2a, aplicamos a lei dos cossenos: b² = a² + c² – 2 . a . c . cos b (√2a)2 = a2 + (2a)2 – 2 . a . 2a . cos b 2a2 = 5a2 – 4a2 . cos b – 3 a2 = – 4 a2 . cos b cos b = 3/4 Substituindo esse valor na relação fundamental da trigonometria, temos: sen2 b + cos2 B = 1 2 sen2 b = 1 – 3 = 1 – 9 = 7 4 16 16 Portanto, como b é um ângulo agudo e seu seno obrigatoriamente é positivo, sen b = √7 4 √7 Por consequência: tg b = sen b = /4 = √7 3/4 3 cos b Resposta: tg b = √7 3

sen α =

cateto oposto a α hipotenusa

cos α =

cateto adjacente a α hipotenusa

tg α =

cateto oposto a α cateto adjacente a α

LEI DOS SENOS a b c = = = 2r sen α sen β sen γ LEI DOS COSSENOS a2 = b2 + c2 – 2 . b . c . cos α , em que α é o ângulo oposto ao lado a.

(Para saber mais sobre PA e PG, veja o capítulo 6) GE MATEMÁTICA 2017

103


6

PROGRESSÕES CONTEÚDO DESTE CAPÍTULO

Progressão aritmética ..................................................................................106 Progressão geométrica.................................................................................108 Como cai na prova + Resumo .....................................................................110

Disparidade econômica evidente na demografia A explosão demográfica na África e a contínua redução da população na Europa coloca em risco tanto países pobres quanto ricos

S

egundo a Organização das Nações Unidas (ONU), até 2050 a população mundial crescerá dos atuais 7,3 bilhões de pessoas para 9,7 bilhões, e até 2100 para 11,2 bilhões. Isso representa um crescimento de mais de 30% nos próximos 35 anos, e outros 15% nos 50 anos seguintes. A primeira preocupação com o crescimento populacional, que afeta a todos no planeta, refere-se à sustentabilidade: é cada vez mais concreta a ameaça de que os recursos naturais não sejam suficientes para garantir a sobrevivência de todos. Dispor de água potável, terras para a produção de alimentos e recursos para geração de energia são três dos desafios de qualquer nação. No entanto, a evolução no número de habitantes não se dá da mesma maneira, nem no mesmo ritmo, em todas as regiões do mundo – o que representa diferentes impactos sobre países ricos e pobres. Enquanto a Europa, com grande número de países desenvolvidos, perderá 92 milhões de habitantes ate 2100, a África mais que triplicará sua população no mesmo período. A queda na taxa de fertilidade nos países ricos resulta na desproporção entre o número de jovens e o de idosos, que dependem dos mais novos para sobreviver. As nações europeias estão entre aquelas em que há cada vez menos jovens para sustentar os que não podem mais trabalhar. Na África o

104 GE MATEMÁTICA 2017

problema tem outro matiz. De todas as regiões do mundo, o continente africano é aquele na qual mais cresce o Produto Interno Bruto (PIB, a soma de todas as riquezas geradas por um país) – algo em torno de 6% ao ano. Mas é, também, a região com as maiores taxas de fertilidade (4,7 filhos por mulher). Apesar da ainda alta incidência de doenças infecciosas, como aids e febre do Ebola, a taxa de mortalidade vem caindo. E o crescimento populacional na África é o mais alto de todas as regiões do globo. Estima-se que, no fim deste século, o número de habitantes no continente suba dos atuais 1,2 bilhão para 4,4 bilhões – a maioria deles vivendo nos 27 países mais pobres do planeta. O desafio para essas nações é reduzir a pobreza, melhorar a saúde e a educação da população. O crescimento populacional e a oferta de recursos para garantir a sobrevivência de todos não seguem linhas paralelas. Neste capítulo você revê os conceitos POBREZA E INFÂNCIA e os cálculos que en- Países africanos pobres volvem progressões têm alta taxa de fertilidade. aritméticas e geo- No Malaui, a população métricas, que estão atual de 16 milhões deve no centro das teorias triplicar até 2050. do matemático inglês O desafio é suprir as Thomas Malthus. necessidades de todos


GE MATEMÁTICA 2017

105

ARIS MESSINIS/AFP PHOTO


6

PROGRESSÕES PROGRESSÃO ARITMÉTICA

Tipos de PA

AVTG/iSTOCK

Uma PA pode ser crescente, decrescente ou constante, dependendo do valor da razão: • Se r > 0, a PA é crescente. A sequência (-6, -1, 4, 9, ...) é uma PA crescente, porque r = 5; • Se r < 0, a PA é decrescente. A sequência (23, 20, 17, 14, ...) é uma PA decrescente porque r = -3; • Se r = 0, então todos os termos da PA serão iguais. A PA é constante. A sequência (5, 5, 5, ...) é uma PA constante porque r = 0.

Termo geral de uma PA FALTA DE COMIDA Para Thomas Malthus, a produção de alimentos não acompanharia o crescimento populacional

As séries que evoluem pela soma Numa sequência em progressão aritmética, a diferença entre dois termos é sempre a mesma

A

questão da sustentabilidade é preocupação antiga. No século XIX, o demógrafo e economista inglês Thomas Malthus publicou seu mais célebre trabalho, Ensaio sobre o Princípio da População, no qual afirmava que “o poder de crescimento da população é indefinidamente maior que o poder que a Terra tem de fornecer recursos para a subsistência do homem”. Malthus comparava o ritmo de crescimento populacional com o da natureza de fornecer alimentos, por exemplo. Analisando a população da Inglaterra, à época, de 7 milhões de pessoas, e a produção das lavouras, ele concluiu que rapidamente o campo não teria mais condições de alimentar todos. A defasagem entre o crescimento da população e o aumento da produção de alimentos, pela ideia de Malthus, ocorreria porque o primeiro, para ele, seguia uma progressão geométrica (PG), e o segundo, uma progressão aritmética (PA). Aqui você conhece as PAs. As PGs são tratadas a partir da pág. 108. Progressão aritmética (PA) é uma sequência de valores (ou termos) em que a diferença (ou razão, r) entre

106 GE MATEMÁTICA 2017

um termo qualquer e seu antecedente é sempre a mesma, ou seja, é constante. Em linguagem matemática: (a1, a2, a3, a4, ..., an, ...) é PA desde que a2 – a1 = a3 – a2 = a4 – a3 = ... =an – an-1 = r, quando n * 2 É o que ocorre com o ritmo de aumento na produção de alimentos no exemplo proposto por Malthus. A cada 25 anos, a quantidade de alimentos aumenta, mas ainda é suficiente para alimentar mais 7 milhões de pessoas, apenas: 7 , 14 , 21 , 28 , 35 (em milhões) +7

+7 +7 +7

(em milhões)

Dizemos que a sequência (7, 14, 21, 28, 35) é uma PA em que n = 5 e r = 7. Ou seja, é uma sequência de cinco valores que variam seguindo a razão 7, pois: a2 – a1 = 14 – 7 = 7 a3 – a2 = 21 – 14 = 7 a4 – a3 = 28 – 21 = 7 etc.

Podemos escrever todos os termos de uma PA conhecendo apenas: • o primeiro termo e a razão da PA; ou • dois termos e suas posições na sequência. Considere a PA abaixo, em que r = 5: ( 2 , 7 , 12 , 17 , 22 , 27 , 32 , 37 , ... , an ) +5 +5 +5 +5 +5 +5 +5 +5 +5 É possível deduzir o valor de um termo qualquer (an) descobrindo a lei matemática que define cada termo da PA: a2 = a1 + r a3 = a2 + r ou a3 = a1 + r + r

a3 = a1 + 2 . r

Da mesma maneira: a4 = a3 + r ou a4 = a1 + r + r + r a4 = a1 + 3 . r a5 = a4 + r ou a5 = a1 + r + r + r + r a 5 = a1 + 4 . r a6 = a5 + r ou a5 = a1 + r + r + r + r + r a 6 = a1 + 5 . r Repare que o fator que multiplica r vale sempre n – 1: a 2 = a1 + 1 . r a3 = a1 + 2 . r a4 = a1 + 3 . r

1=2–1 2=3–1 3=4–1

Então, podemos concluir que a fórmula para calcular o valor de qualquer termo da PA é: an = a1 + (n - 1) . r , para n * 2


Soma de termos de uma PA

Em toda PA finita, a soma de quaisquer dois termos equidistantes dos extremos é igual à soma dos extremos. Observe: )!3-!!!8-!!!23-!!!28-!!!33-!!!38-!!!43-!!!48*!!

O RITMO DE AUMENTO DE UMA POPULAÇÃO Ano 0

POPULAÇÃO 7 milhões

25 anos

14 milhões

50 anos

28 milhões

multiplicados por 2

multiplicados por 2

multiplicados por 2

75 anos multiplicados por 2

56 milhões 100 anos

tpnb!>!4:

112 milhões

tpnb!>!4: tpnb!>!4: tpnb!>!4:

O RITMO DE AUMENTO DA PRODUÇÃO DE ALIMENTOS ALIMENTOS

Com a propriedade acima, podemos calcular a soma de todos os termos de uma PA finita. Na PA (2, 7, 12, 17, 22, 27, 32, 37), já calculamos a soma de dois termos equidistantes dos extremos: 39. Para a soma de todos os oito termos (S8 ), temos, então: a1 + a8 = 39 a2 + a7 = 39 a3 + a6 = 39 a4 + a5 = 39 Então, S8 = 39 + 39 + 39 + 39 = 4 . 39 = 156

Ano 0

Repare que, como os termos foram somados aos pares, para uma PA de oito termos, temos quatro pares. Então, podemos definir a regra para a soma total dos termos de uma PA:

O RACIOCÍNIO DE MALTHUS A primeira pirâmide mostra o crescimento da população da Inglaterra, à época dele, de 7 milhões de habitantes; a segunda, o aumento na produção de alimentos. A população dobra de tamanho a cada período de 25 anos. Já a produção de alimentos cresce sempre em PA – pela soma de 7 milhões a cada período de 25 anos: 7 + 7 + 7... A história acabou desmentindo Malthus. O desenvolvimento tecnológico aumentou a produtividade das fazendas. E a industrialização permite estocar alimentos por longos períodos, aumentando a oferta para a população.

Sn =

n . (a1 + an) 2

Com a mesma expressão, calcula-se a soma dos n primeiros termos de uma PA. Veja: Se na PA (2, 7, 12, 17, 22, 27, 32, 37) quisermos calcular a soma dos quatro primeiros termos (S4 ), recorreremos de novo à soma dos termos equidistantes dos extremos: 2 + 17 = 19 7 + 12 = 19 Então, S4 = 2 .19 = 38 Confira com a fórmula: Sn =

n . (a1 + an ) 2

S4 =

4 . (a1 + a4 ) 2

S4 =

4 . (2 + 17 ) 2

S4 =

4 . 19 = 38 2

7 milhões 25 anos 14 milhões 50 anos 21 milhões 75 anos 28 milhões 100 anos

+ 7 milhões

+ 7 milhões

+ 7 milhões

+ 7 milhões

35 milhões

NA PRÁTICA TERMO GERAL E SOMA DOS TERMOS DE UMA PA

Quantos múltiplos de 3 existem entre 100 e 299? A sequência de múltiplos é uma PA. Neste caso, uma PA de r = 3 (a diferença entre 3, 6, 9,...). O primeiro e o último termo têm de ser divisíveis por 3. Então, a1 = 102 e an = 297. an = 297 é o último termo da sequência. Pela fórmula para o termo geral de PA, temos: an = a1 + (n – 1) . r → 297 = 102 + (n – 1) . 3 → 195 = 3n – 3 → 198 = 3n → n = 66 Existem, portanto, 66 múltiplos de 3 no intervalo de 100 a 299. Qual o valor de x que torna verdadeira a equação (x – 3) + (x – 6) + (x – 9) + ... + ( x – 42) = 371? O lado esquerdo da equação traz uma PA em que a1 = (x – 3), a2 = (x – 6) ... an = (x – 42) A razão dessa PA é r = a2 – a1 = (x – 6) – (x – 3) = x – x – 6 + 3 → r = –3 Então, a1 = x – 3 e an = x – 42 O valor de n para o último termo, pela fórmula do termo geral é: an = a1 + (n – 1) . r → x – 42 = (x – 3) + (n – 1) . (–3) = x – 3 – 3n + 3 → 3n = 42 → n = 14 Encontramos o valor de x, usamos a fórmula para a soma de termos: Sn = n . (a1 + an) / 2 S14 = 14 . [(x – 3) + (x – 42)] / 2 → S14 = 14 . (2x – 45) / 2 → S14 = 14 . x – 315 Voltando à equação original, temos então 14x – 315 = 371 → x = 371 + 315 / 14 → x = 49 GE MATEMÁTICA 2017

107


6

PROGRESSÕES PROGRESSÃO GEOMÉTRICA

FILIPEFRAZAO/iSTOCK

PRESSÃO HUMANA Malthus achava que a população crescia tão rápido que logo ficaria sem alimentos. Não foi bem assim, mas o número de pessoas no mundo aumentou tanto que ameaça os recursos naturais

Sequências que crescem ou decrescem em ritmo muito acelerado As progressões geométricas podem explicar, pelo menos em parte, os riscos que a humanidade impõe ao meio ambiente

A

história desmentiu a previsão de Malthus, de que faltariam alimentos para a população, porque ele não poderia imaginar o desenvolvimento da tecnologia. Os fertilizantes, por exemplo, aumentaram a produtividade nas lavouras. E os alimentos enlatados permitem que a população tenha o que comer mesmo em períodos de entressafra. Mas há quem considere que as ideias do demógrafo inglês são válidas para as questões de sustentabilidade. A exploração cada vez maior de recursos naturais, como água e solo, pode levar a um colapso ecológico e econômico. Progressão geométrica (PG) é uma sequência de números reais não nulos em que o quociente entre um termo qualquer e o termo antecedente é sempre o mesmo. Esse quociente constante é chamado razão da PG e é indicado por q. Em linguagem matemática:

108 GE MATEMÁTICA 2017

(a1, a2, a3, a4, ..., an, ...) é PG desde que a2 : a1 = a3 : a2 = a4 : a3 = ... = an : an-1 = q, para n * 2 No exemplo proposto por Malthus, a população, que se duplica a cada 25 anos, cresce em PG – ou seja, a cada 25 anos a população multiplicase por 2: 7 , 14 , 28 , 56 , 112 (em milhões) .2 .2 .2 .2 A sequência (7, 14, 28, 56, 112) é uma PG de cinco termos (n = 5) e razão q = 2, pois: a2 : a1 = 2 a3 : a2 = 2 a4 : a3 = 2 a5 : a4 = 2

NA PRÁTICA RAZÃO

Numa PA crescente, o primeiro, o quarto e o décimo terceiro termos formam uma PG. Sabendo que o primeiro termo da PA é 3, escreva a PG. A PA é (a1, a2, a3, a4, ... a13) Conhecemos a1 = 3 E, pela fórmula do termo geral de PA, sabemos que: a4 = 3 + 3 . r e a13 = 3 + 12 . r, em que r é a razão da PA (ou seja, a diferença entre dois termos subsequentes). O enunciado diz que esses três termos formam uma PG. Temos, portanto: (3, 3 + 3r, 3 + 12r) A razão entre esses termos, na PG, é q – o quociente da divisão entre dois termos subsequentes: q = an / a (n-1) A razão é sempre a mesma. Então, podemos igualar o quociente entre os termos a1 , a2 e a3 da PG: q=

3 + 3r 3 + 12r = 3 3 + 3r

Multiplicando em cruz: 9 + 18r + 9r2 = 9 + 36r 9r2 – 18r = 0 → r . (9r – 18) = 0 r1 = 0 (não convém) / r2 = 2 (convém) Portanto, a PG é (3, 9, 27).


CRESCIMENTO EM DESCOMPASSO 112 milhões

Segundo Malthus, depois de 100 anos, a população teria chegado a 112 milhões de pessoas

Mas haveria alimento suficiente para apenas 35 milhões.

56 milhões

35 milhões 28 milhões 21 milhões 14 milhões 7 milhões

População Alimentos Ano zero

25 anos depois

50 anos depois

75 anos depois

100 anos depois

GENTE DEMAIS, COMIDA DE MENOS Acima, novamente a teoria de Malthus, agora num gráfico de linhas. Repare que, nos primeiros 25 anos, a produção de alimentos acompanharia o crescimento de uma população de 7 milhões de pessoas (ambos subiriam 7 milhões). Mas, a partir de então, a cada 25 anos, a população dobraria de tamanho e a produção de alimentos continuaria crescendo apenas 7 milhões. Ao cabo de um século, a quantidade de alimento disponível seria suficiente para apenas um terço da população.

Tipos de PG

Uma PG pode ser crescente, decrescente ou oscilante, conforme o sinal da razão (q) e do primeiro termo (a1 ): • Quando a1 > 0 e q > 1 ou a1 < 0 e 0 < q < 1, a PG é crescente. A sequência (2, 6, 18, 54, 162) é uma PG crescente com a1 = 2 e q = 3. A sequência (–5, –2,5, –1,25, –0,625) também é uma PG crescente, em que a1 = –5 e q = 0,5. • Quando a1 › 0 e 0 ‹ q ‹ 1 ou a1 ‹ 0 e q › 1, a PG é decrescente. A sequência (40, 20, 10, 5, ...) é uma PG decrescente porque a1 = 40 e q = 0,5. A sequência (–4, –8, –16, ...) também é uma PG decrescente, porque a1= –4 e q = 2. • Quando q < 0, a PG é alternada ou oscilante – ou seja, ora o termo é negativo, ora positivo. A sequência (4, –4, 4, –4, ...) é uma PG oscilante, em que q = –1. • Quando q = 1, a PG é constante, ou seja, todos seus termos são iguais. Exemplos de PG constantes: (5, 5, 5, 5, 5, ...) (1/8, 1/8, 1/8, ...)

• Quando q = 0 e a1 & 0, a sequência será chamada estacionária. A PG estaciona porque qualquer número multiplicado por zero resulta em zero. A PG em que a1 = 5 e q = 0 é estacionária: (5, 0, 0, 0, ...)

Termo geral de uma PG

Conhecendo o primeiro termo (a1) e a razão (q) de uma PG, escrevemos todos os seus demais termos. Considere a PG ( 2, 6, 18, 54, ..., an ). Nela, a1 = 2; q = 3 Acompanhe o raciocínio: a2 = a1 . q a3 = a1 . q2 a3 = a2 . q ou a3 = a1 . q . q a4 = a3 . q ou a4 = a1 . q.q.q a4 = a1 . q3 a5 = a4 . q ou a5 = a1 . q . q . q . q a5 = a1 . q4 Repare como se altera o valor do expoente de q: a razão é sempre elevada a um número que corresponde a n – 1: 1=2–1 a2 = a1 . q1 2 a3 = a1 . q 2=3–1 3 3=4–1 a 4 = a1 . q 4 a 5 = a1 . q 4=3–1 Então podemos concluir que a fórmula para o valor de qualquer termo de uma PG é dada pela expressão: an = a1 . q(n - 1), em que n * 2

NA PRÁTICA PG CRESCENTE

Determine o valor de x para que a sequência abaixo seja uma PG crescente. (x – 2, x + 2, 2x + 4) Primeiro, vamos encontrar a razão q da PG: q=

x + 2 2x + 4 = x–2 x+2

Multiplicando em cruz, temos: x2 + 4x + 4 = 2x2 + 4x – 4x – 8 –x2 + 4x + 12 = 0 Resolvendo a equação do 2º grau, obtemos dois valores para x: x1 = –2 e x2 = 6. Substituindo esses valores na sequência confirmamos qual deles cria uma PG crescente: Para x1 = –2, a sequência é (–4, 0, 0) → PG estacionária Para x2 = 6, a sequência é (4, 8, 16) → PG crescente Então, para que a PG seja crescente, x = 6

NA PRÁTICA TERMO GERAL

Numa PG, o terceiro termo é 50 e o sexto termo é 6 250. Qual é o primeiro termo da sequência? Conhecemos os valores de a3 (50) e a6 (6 250). A cada termo da PG, a razão q é multiplicada uma vez. Os dois termos dados distam três posições (6 – 3 = 3). Então, de um para o outro, os valores foram multiplicados pela razão q três vezes: a6 = a3.q3 → 6 250 = 50 . q3 q3 = 125 → q = 5 Conhecendo q e o valor de a3, encontramos a1. Pelo mesmo raciocínio, entre a3 e a1, os valores foram multiplicados por q duas vezes: a3 = a1 . q2 → 50 = a1 . 52 a1 = 50 / 52 → a1 = 2 GE MATEMÁTICA 2017

109


6

COMO CAI NA PROVA

1. (PUCRJ 2015)

RESOLUÇÃO

a) Quantos múltiplos de 13 há entre 100 e 200? b) Quantos múltiplos de 17 há entre 1000 e 2000?

RESOLUÇÃO a) Você encontra rapidamente o primeiro múltiplo de 13 maior que 100: 104 (104 = 13 . 8). Encontra também com facilidade o maior múltiplo de 13 abaixo de 200: 195 (195 = 13 . 15). Assim, podemos escrever a sequência dos múltiplos de 13 entre 100 e 200: (104, 117, 130, ... , 195). Trata-se de uma PA de razão r = 13 e a1 = 104. Vamos escrever o termo geral dessa PA e, em seguida, determinar a posição do último termo, an = 195. Para o termo geral: an = 104 + (n – 1) . 13 Substituindo o último termo, que já conhecemos: 195 = 104 + (n – 1) . 13 91 = (n – 1) . 13 n–1=7 n=8 Portanto, 195 é o 8º termo da sequência, que tem 8 termos. Resposta: existem 8 múltiplos de 13 entre 100 e 200. b) O procedimento é o mesmo do item a. O menor múltiplo de 17 maior que 1000 é 1003 (1003 = 17 . 59). E o maior múltiplo de 17 abaixo de 2 000 é 1989 (1989 = 17 . 117). Então, podemos escrever a sequência de múltiplos de 17 entre 1000 e 2000: (1003, 1020, 1037, ... , 1989). Esta é uma PA de razão r = 17 e a1 = 1003. O termo geral dessa sequência é an = 1003 + (n – 1) . 17 Substituindo na fórmula do termo geral o valor do último termo da PA, temos: 1989 = 1003 + (n – 1) . 17 986 = (n – 1) . 17 n – 1 = 58 n = 59 Resposta: existem 59 múltiplos de 17 entre 1000 e 2000.

Nas duas sequências, vamos escrever seus termos gerais e calcular os valores referentes a seus respectivos décimos termos, ou seja, seus a10 . PA: an = 1 + (n – 1) . 1 → a10 = 1 + 9 . 1 = 10 PG: an = 2 . 2n – 1 → a10 = 2 . 29 = 210 Como a quantidade de alimentos é dada em toneladas, ou seja, milhares de quilogramas, a razão pedida é 10 000 = 10 4 = 2 4 . 5 4 = 5 4 210 210 210 26 Resposta: B

3. (PUCSP 2016) Seja o triângulo equilátero T cujo lado mede x cm. Unindo1

se os pontos médios dos lados de T1 , obtém-se um novo triângulo equilátero T2 ; unindo-se os pontos médios dos lados do triângulo T2 , obtém-se um novo triângulo equilátero T3 ; e, assim, sucessivamente. Nessas condições, se a área do triângulo T9 é igual a 25√3 cm2 então x é igual a: 64 a) 640 b) 520 c) 440 d) 320

RESOLUÇÃO Acompanhe na figura abaixo: cada vez que você constrói um triângulo com o vértice definido no ponto médio dos lados de um triângulo original, os lados de cada triângulo subsequente medem a metade do lado do triângulo anterior. Ou seja, esses lados variam numa PG de razão q = 1/2.

x

x

x

2. (UEL 2016) Leia o texto a seguir.

Segundo teorias demográficas, a população mundial cresceria em ritmo rápido, comparado a uma PG = (2, 4, 8, 16, 32, 64, ... at , ...), e a produção mundial de alimentos cresceria em um ritmo lento, comparado a uma PA = (1, 2, 3, 4, ... bt , ...). Adaptado de: <http://educação.uol.com.br/disciplinas/geografia/teorias-demograficasmalthusianos-neomalthusianos-e-reformistas.htm>. Acesso em: 15 jun. 2015.

Suponha que PA seja a sequência que representa a quantidade de alimentos, em toneladas, produzidos no tempo t > 0, e que PG seja a sequência que representa o número de habitantes de uma determinada região, nesse mesmo tempo t. A partir dessas informações, assinale a alternativa que apresenta, corretamente, a razão entre a quantidade de alimentos, em kg, e o número de habitantes, para t = 10 anos. a) 53/26 b) 54/26 c) 55/26 d) 53/25 e) 54/25

110 GE MATEMÁTICA 2017

x 2

x 2

x 2

Apenas olhando os triângulos acima, você percebe que a área de triângulos subsequentes, um menor que o outro, vai se reduzindo em uma razão de ¼. Esta é a razão q de uma nova PG, da área dos triângulos. (Esse raciocínio vale, inclusive, para a sequência de qualquer tipo de polígono cujos lados medem a metade do polígono anterior.) Pela fórmula da área de um triângulo equilátero, temos que a área de T1 em função de x é A1 = x2√3 4


RESUMO

A área de T9 é o nono termo da PG (A1 , A2 , A3 , ..., A9 ) de razão q = ¼. Pela fórmula do termo geral de uma PG, definimos An = x2√3 . 1 n – 1 4 4 Para A9 temos: 25√3 = x2√3 . 1 8 64 4 4 2 25 = x 64 4 . 48 25 = x2 26 218 x2 = 25 . 218 = 25 . 212 26 Tirando a raiz quadrada em ambos os lados da igualdade, temos: x = 5 . 26 = 5 . 64 = 320 Resposta: D

4. (FGV 2016) Mauro iniciou um programa de perda de peso quando estava

pesando 90 kg. A programação previa a perda de 1,6 kg na primeira semana, 1,5 kg na segunda, 1,4 kg na terceira, 1,3 kg na quarta, e assim sucessivamente até que a perda semanal de peso se estabilizasse em 0 kg, ocasião em que ele iniciaria o controle de manutenção do peso atingido. Sabe-se que o programa realizado por Mauro foi plenamente cumprido.

Progressões PA é uma sequência de termos em que a diferença (ou razão, r) entre um termo qualquer e seu antecedente é constante: a2 – a1 = a3 – a2 = a4 – a3 = ... =an – an–1 = r, quando n ≥ 2 TERMO GERAL DE UMA PA an = a1 + (n – 1) . r, para n ≥ 2 SOMA DOS TERMOS DE UMA PA Sn =

n . (a1 + an) 2

PG é uma sequência de números reais em que o quociente (razão, q) entre um termo qualquer e seu antecedente é constante: a2 a3 a4 a = = = ... = n = q, com n ≥ 2 an–1 a1 a2 a3 TERMO GERAL DE UMA PG Encontra-se qualquer termo (an) de uma PG, calculando-se: an = a1 . qn – 1, n ≥ 2 SOMA DOS TERMOS DE UMA PG FINITA

a) Considere o período que vai do início do regime até o final da última semana em que Mauro perdeu algum peso e calcule a média mensal de perda de peso desse período. Para isso, admita meses com 4 semanas. b) Sendo P o peso de Mauro em quilogramas e n o número de semanas completas decorridas a partir do instante em que Mauro iniciou o programa de perda de peso, determine P em função de n, com n inteiro positivo.

RESOLUÇÃO a) A sequência de valores de perda de peso semanais é (1,6; 1,5; 1,4; 1,3; ... 0,1). Atenção aqui: o enunciado pede que seja considerado o período “até o final da última semana em que Mauro perdeu algum peso”. Portanto, a semana em que tem início o período de manutenção não entra na sequência. Essa sequência é uma PA de razão r = – 0,1 e termo geral dado pela fórmula an = 1,6 + (n – 1) . (–0,1) Substituindo nessa expressão o valor do último termo da sequência (0,1), determinamos sua posição: 0,1 = 1,6 + (n – 1) . (– 0,1) – 1,5 = (n – 1) . (– 0,1) n – 1 = 15 n = 16 Portanto, a sequência tem 16 termos. Assim, o período considerado durou 16 semanas, ou quatro meses.

Sn =

a1 . (qn – 1) para q ≠ 1 q–1

SOMA DOS TERMOS DE UMA PG INFINITA a lim Sn = – 1 q–1 n→∞

b) O peso P de Mauro após n semanas de tratamento é igual à diferença entre o peso de Mauro no início do tratamento (90 kg) e a soma do peso perdido a cada semana. Essa soma corresponde à soma de uma PA de razão r = (–0,1) e primeiro termo a1 = 1,6. Pela fórmula da soma dos termos de uma PA, temos Sn = (1,6 + an) . n (I) 2 O último termo dessa sequência é an = 1,6 + (n – 1) . (– 0,1) (II). Substituindo a equação (II) em (I), temos: Sn = [1,6 + 1,6 + (n – 1) . (–0,1)] . n 2 Sn = [3,2 – 0,1n + 0,1] . n 2 Sn = 3,3n – 0,1n2 2

Somando os valores da sequência, temos o total de peso perdido no período: S16 = (1,6 + 0,1) . 16 = 1,7 . 8 = 13,6 kg 2

Portanto, o peso P de Mauro, em kg, após n semanas de tratamento será: P(n) = 90 – Sn = 90 – 3,3n – 0,1n2 2

Portanto, a média mensal de perda de peso, M, no período de 4 meses é M = 13,6 = 3,4 kg/mês 4

P(n) = 90 – 1,65n + 0,05n²

Resposta: o peso perdido a cada mês, em média, é de 3,4 kg/mês.

Resposta: a função que define o peso de Mauro em relação ao número de semanas de tratamento é P(n) = 0,05 n² – 1,65n + 90 GE MATEMÁTICA 2017

111


7

COMBINATÓRIA E PROBABILIDADE CONTEÚDO DESTE CAPÍTULO

Combinatória ..................................................................................................114 Probabilidade ..................................................................................................117 Como cai na prova + Resumo .....................................................................120

Contra toda a probabilidade A decisão dos britânicos de deixar a União Europeia gera perplexidade e levanta preocupações sobre os rumos da economia mundial

A

sensação que ficou foi a de um tiro dado no pé. Depois de 17,5 milhões de cidadãos britânicos terem aprovado a saída do Reino Unido da União Europeia (UE), a imprensa mostrou um país perplexo diante da ousadia política e econômica. De imediato, a deserção alterou o cenário político na própria Inglaterra, com a renúncia do primeiro-ministro, David Cameron. As consequências econômicas são ainda imprevisíveis, mas prometem ser mais profundas e extensas. Como vivemos uma economia globalizada, todas as nações do planeta devem ser afetadas. Assim que o resultado do plebiscito foi anunciado, o clima entre os próprios britânicos tinha o tom de arrependimento. Alguns movimentos chegaram a pedir novo plebiscito. O Reino Unido é a primeira nação que decide abandonar a UE. O maior bloco econômico do mundo foi criado em 1992, pelo tratado de Maastrich, com o objetivo de promover a integração das nações europeias. Uma dessas medidas é o livre-comércio, com a eliminação de tarifas alfandegárias; outra é a livre circulação de pessoas. Hoje, a UE reúne 28 países-membros, mas nem todos aderiram a todas as medidas. O euro, a moeda única criada em 2001, por exemplo, foi adotada por apenas 19 países, que passaram a integrar a chamada Zona do Euro.

112 GE MATEMÁTICA 2017

O Brexit (contração das palavras “Britain” e “exit”, ou seja, saída britânica) teve como pano de fundo o descontentamento das parcelas mais conservadoras da população com alguns dos mecanismos de integração do bloco. Uma das regras questionadas é a imposição de uma política monetária centralizada no Banco Central Europeu. Os insatisfeitos reclamavam, também, dos repasses financeiros feitos à UE, usados para socorrer as economias mais fracas, como a Grécia. Na verdade, o Reino Unido jamais aderiu completamente às políticas da UE – nunca adotou o euro, nem liberou suas fronteiras à migração. O mal-estar entre UE e Reino Unido cresceu com a crise econômica mundial deflagrada em 2008. O Brexit pegou no contrapé as casas de aposta britânicas. Até algumas semanas antes do plebiscito, elas calculavam que a probabilidade de saída não passava de 30%, e pagavam 11 libras a cada 5 a quem apostasse na permanência. Mas, às vésperas do ESTAMOS FORA plebiscito, essa proba- Manchete nos jornais bilidade havia subido sobre a opção vitoriosa para 70%. no plebiscito: o Reino Possibilidades e Unido quer deixar de probabilidades são os ser país-membro da temas deste capítulo. União Europeia


LEON NEAL/AFP PHOTO

GE MATEMÁTICA 2017

113


COMBINATÓRIA E PROBABILIDADE COMBINATÓRIA

RENATO PIZZUTTO

72

IDENTIDADE ÚNICA Cada placa de automóvel no Brasil é criada pelo arranjo de três letras e quatro algarismos

A arte de contar e combinar O conjunto formado pelo arranjo de elementos de outros dois ou mais conjuntos depende do modo como esses elementos são combinados

114 GE MATEMÁTICA 2017

A

análise combinatória lida com o cálculo das diversas possibilidades de combinar elementos em diferentes conjuntos. Por exemplo, combinar 9 dígitos no número dos celulares, ou três letras e quatro algarismos nas placas dos automóveis. A combinatória é uma ferramenta essencial para o cálculo das probabilidades (veja a partir da pág. 117). O número de combinações possíveis entre elementos de um ou mais conjuntos depende das condições em que essa combinação é feita. Em alguns casos, os elementos de um conjunto podem se repetir, em outros, não. Há casos, ainda, em que a ordem dos elementos do grupo é importante. Nos números de telefone, por exemplo, os algarismos

podem se repetir, e a troca de posição de um único dígito já cria um número diferente, para outro aparelho.

Árvore das possibilidades

Uma das maneiras de calcular as possibilidades para um evento é montar a árvore de possibilidades – um diagrama para calcular o número de resultados possíveis de um evento e a probabilidade de obter cada resultado. Veja a árvore de possibilidades para a sequência de três lançamentos de uma moeda. Repare que, a cada vez em que a moeda é lançada, há apenas duas possibilidades de resultado: ou dá cara, ou dá coroa. Assim, a cada novo lançamento, o número de possibilidades é multiplicado por 2. Veja na figura na página ao lado.


1º lançamento

2º lançamento

3º lançamento

Sequências

NA PRÁTICA ÁRVORE DAS POSSIBILIDADES

Ca r

a

Quantos números ímpares formados por três algarismos distintos existem? Temos dez algarismos (de 0 a 9) para distribuir por três casas: unidade, dezena e centena. Vamos casa a casa: • Casa das centenas: pode ser ocupada por qualquer algarismo, menos o zero (ou não teríamos um número com três algarismos). São 9 possibilidades.

Co r

oa

• Casa das dezenas: pode ser ocupada por qualquer algarismo, par ou ímpar, inclusive o zero. Mas esse algarismo não pode ter sido usado na casa das centenas. Ficamos então, de novo, com 9 possibilidades. • Casa das unidades: o enunciado diz que o número tem de ser ímpar. Portanto, essa casa só pode ser ocupada pelos algarismos 1, 3, 5, 7 ou 9. São 5 possibilidades.

Repare que o número total de possibilidades aumenta a cada novo lançamento: às duas possibilidades para o primeiro lançamento somamse outras duas para o segundo, e mais duas para o terceiro lançamento. Ao final dos três lançamentos, há um conjunto de oito possibilidades de resultado. Assim, o número total de possibilidades é o produto das possibilidades de cada um dos lançamentos. Este é o princípio multiplicativo. Confira abaixo. Lançamento

Número de possibilidades

2

2.2=4

2.2.2=8

Permutação simples

São situações em que as posições dos elementos no conjunto são importantes, ou seja, a cada troca de posição, um novo conjunto é formado. Por exemplo, a ordem de pessoas em uma fila, ou de livros em uma estante. Em todos esses casos, os elementos com que se trabalha são obrigatoriamente distintos. Em uma permutação simples de n elementos, o total de possibilidades é o fatorial de n, ou seja, n! P(n) = n!

ATENÇÃO Em matemática, o ponto de exclamação (!) indica fatorial – a multiplicação de números naturais consecutivos, em ordem decrescente: n! = n . (n – 1) . (n – 2) . (n – 3)... 3 . 2. 1, com n ≥ 2 Exemplo: 6! = 6 . 5 . 4 . 3 . 2 . 1 = 720

PARA IR ALÉM O filme O Jogo da Imitação, de Morten Tyldum, aborda o tema combinatória. Ele conta a história do pioneiro da computação Alan Turing, que quebrou o código secreto usado pelos nazistas em suas mensagens, durante a Segunda Guerra Mundial.

O número total de número ímpares de três algarismos é a multiplicação de cada uma dessas possibilidades: 9 . 9 . 5 = 405. Existem, portanto 405 números ímpares de três algarismos distintos.

NA PRÁTICA PERMUTAÇÃO SIMPLES

Quantos anagramas são possíveis da palavra SIMULADO? Anagrama é a recombinação de todas as letras de uma palavra, apenas alterando sua posição. SIMULADO é formada por 8 letras distintas. Então, pela fórmula,

P 8s = 8! = 40 320 anagramas. E quantos anagramas de SIMULADO podemos montar, mantendo as letras D e O lado a lado? As letras D e O se comportam como uma única letra. Consideramos, então, que nossa palavra funciona como se tivesse apenas sete letras. Mas as letras D e O podem trocar de lugar entre si: DO ou OD. O total de possibilidades é

2 .p 7s = 2 . 7! = 10 080 GE MATEMÁTICA 2017

115


72

COMBINATÓRIA E PROBABILIDADE COMBINATÓRIA

Permutação com repetição

Ocorre quando o conjunto onde haverá permutação apresenta elementos repetidos. Nesse caso, descontamos os casos de repetição. No princípio multiplicativo, descontar significa dividir. A fórmula geral para permutação com repetição é: P na,b,c...

n! = a!b!c!...

• n é o número de elementos • a, b, c... são os elementos repetidos. Por exemplo, quantos são os anagramas possíveis da palavra COURO? Consideramos as cinco letras, como se não houvesse repetição da letra O. Temos, então uma permutação simples: P 5s = 5! = 120

Mas a permutação de um O com o outro não cria um novo anagrama. Então, descontando essa permutação, pela divisão: 120 : 2 = 60 anagramas.

Arranjos

Arranjo simples é aquele que, de um grupo de n elementos distintos, queremos formar um subconjunto ordenado e único de alguns desses elementos (p): A n,p =

n! (n – p) !

• n é o total de elementos; • p é o número de elementos de cada subconjunto a ser formado. Arranjo com repetição é aquele que admite repetição. A fórmula geral para o cálculo de arranjos com repetição é A rn,p = n p

• n é o total de elementos; • p é o número de elementos de cada subconjunto a ser formado.

Combinação simples

Nela, a troca de posição entre os elementos não cria um novo grupo. Ou seja, são arranjos em que não importa a ordem dos elementos. Veja o exemplo a seguir.

NA PRÁTICA

NA PRÁTICA

COMBINAÇÃO SIMPLES

ARRANJO COM REPETIÇÃO

Em uma urna, há cinco bolas pretas e três bolas brancas. De quantas maneiras é possível, ao se sortearem três bolas, que saiam duas bolas pretas e uma branca? As bolas pretas e brancas são idênticas entre si, então a ordem em que elas são sorteadas não faz a menor diferença. Temos a combinação de dois eventos: sortear duas bolas pretas e sortear uma bola branca. • Para o sorteio das pretas: 5 bolas pretas, duas a duas:

5! C 5, 2 = 3! 2! = 10

• Para sorteio das brancas: 3 bolas brancas, uma a uma:

A n,p n! p! = (n – p) !p!

Eliminando a placa 0000, ficamos com 9 999 possibilidades. O total de possibilidades é o produto dos valores obtidos acima: 17 576 . 9 999 = 175 742 424 placas possíveis.

NA PRÁTICA

Pelo princípio multiplicativo, o número de possibilidades para o evento total é o produto dos dois eventos tomados separadamente: 10 . 3 = 30 possibilidades.

ARRANJO SIMPLES

A senha de um cadeado é composta de quatro algarismos distintos. Qual o número máximo de tentativas que uma pessoa que não conhece a senha deve fazer para abrir o cadeado? A senha faz o arranjo de 10 algarismos distintos, quatro a quatro.

A 10, 4 =

116 GE MATEMÁTICA 2017

C n,p =

Quantas placas de automóvel é possível confeccionar com as regras em uso no Brasil, de três letras e quatro algarismos? (Detalhe: a legislação brasileira não permite placas com quatro zeros.) O alfabeto tem 26 letras e existem 10 algarismos (0 a 9). Fazendo os cálculos para cada um desses conjuntos, temos: • Arranjo de 26 letras, três a três: 26 . 26 . 26 = 263 = 17 576 possibilidades; • Arranjo de 10 algarismos, quatro a quatro: 10 . 10 . 10 . 10 = 104 = 10 000 possibilidades.

3! C 3, 1 = 2!1! = 3

Este é o número de possibilidades de, ao sortearmos três bolas, tirarmos duas pretas e uma branca.

De quantas formas é possível formar uma comissão de três membros, a partir de um conjunto de cinco pessoas (A, B, C, D e E)? Começamos tratando a situação como se fosse um caso de arranjo simples. O número de possibilidades seria 5 . 4 . 3 = 60. Mas, nas combinações simples, a ordem não é importante (como é nos arranjos). Portanto, incluídas nas 60 possibilidades, temos seis possibilidades de permutação para cada grupo de três membros: {A, B, C}, {A, C, B}, {C, A, B}, {C, B, A}, {B, A, C} e {B, C, A}. Para descontar essa repetição, devemos fazer a divisão do total de possibilidades pelas possibilidades repetidas – neste caso, por 6. Assim, o total de comissões possíveis de se montarem nessas condições é 60 : 6 = 10. O número possível de combinações simples, de n elementos combinados p a p é dado por:

10! 10! 10 . 9 . 8 .7 . 6! = 6! = = 10 . 9 . 8 . 7 = 5 040 6! (10 – 4) !

Para abrir o cadeado, a não ser que o arrombador tenha sorte, ele precisará fazer 5 040 tentativas.


7

COMBINATÓRIA E PROBABILIDADE PROBABILIDADE

FEZINHA DIFÍCIL São mínimas as probabilidades de ganhar num jogo como a Mega-Sena, que permite milhões de combinações

Questão de sorte (ou azar) Ganhar na loteria ou ter um filho homem são eventos que dependem do acaso. Mas sempre podemos estimar as probabilidades

A

s possibilidades, já vimos, são todos os possíveis resultados de um evento. Probabilidade é outra coisa: a chance de que determinado resultado ocorra. Toda probabilidade é uma proporção, apresentada como porcentagem, ou como uma chance a cada x vezes. Ou seja, a probabilidade de ocorrer um evento é uma fração de todos os possíveis resultados.

Conceitos

As probabilidades são calculadas para experimentos aleatórios – aqueles que, se repetidos, têm resultados incertos, mas com a mesma chance de ocorrer. Por exemplo, não importa quantas vezes um dado é lançado, só existem seis possibilidades de resultado: 1, 2, 3, 4, 5 ou 6. E que resultado dará é um evento

impossível de prever com certeza. Todo cálculo de probabilidade envolve dois tipos de conjuntos. O espaço amostral (S) é o conjunto de possibilidades, possíveis resultados. O evento (E) é o conjunto dos resultados desejados. Quando temos um experimento aleatório, a probabilidade de E ocorrer é dada pela fração P (E) =

n (E) n (S)

em que n é o número de eventos. • Se n(E) = n(S), o evento tem probabilidade 1 = 100%. Este é chamado um evento certo; • Se n(E) = 0, então P(E) = 0 = 0 %; este é um evento impossível. GE MATEMÁTICA 2017

117


72

COMBINATÓRIA E PROBABILIDADE PROBABILIDADE

Tipos de eventos

• Eventos independentes são aqueles em que a ocorrência de um não interfere na possibilidade de ocorrer o outro. Pelo princípio multiplicativo, o número de possibilidades de um evento A ocorrer e de outro evento B também ocorrer é igual ao produto das possibilidades de cada um deles ocorrer separadamente: P (A + B) = P(A) . P(B)

• Dois eventos A e B são complementares se a intersecção dos conjuntos de possibilidades for vazio e se a união dos dois for igual ao espaço amostral S. Veja: Em eventos complementares, a soma das probabilidades dos dois eventos (A e B) é sempre igual a 100%: P(A) + P(B) = 1 • Eventos mutuamente exclusivos são aqueles nos quais a ocorrência de um elimina qualquer probabilidade de ocorrer o outro. Por exemplo: qual a probabilidade de você sortear um número que seja par, primo e maior que 5? Nenhuma, porque o único número primo par é 2, que é menor que 5. Acima de 2, todos os primos são ímpares.

União de dois eventos

Possibilidades e probabilidades envolvem conjuntos. E a teoria dos conjuntos ajuda, muitas vezes, a resolver problemas nessa área. A união de dois eventos A e B é dada pela expressão: P (A , B) = P (A) + P (B) - P (A + B)

Médias, mediana e moda

A combinatória e a probabilidade estão na base das estatísticas – a coleta e organização de dados que fazem um retrato de determinada situação. A estatística estuda uma amostra, que é parte de um universo de elementos. Por exemplo, para estudar o desempenho de todos os 240 alunos de dez classes do Ensino Médio, os diretores de uma escola podem selecionar oito alunos de cada sala. O universo é o total de alunos da escola: 240. E a amostra é a soma dos alunos selecionados: 80. Numa amostra significativa, que represente o universo, é possível estabelecer relações que podem ser extrapoladas para o universo. Conheça alguns conceitos básicos usados na estatística: • Médias Existem dois tipos de médias. A média aritmética é a soma de todos os valores dos elementos de um conjunto dividida pelo número total de elementos de um conjunto. Na média ponderada, levamos em consideração o peso de cada elemento (por exemplo, provas com peso 2 ou 1). Neste caso, o cálculo é multiplicar cada elemento por seu peso, somar todos eles e dividir o resultado pela soma de todos os pesos. • Mediana é a medida central de uma lista de medidas colocadas em ordem crescente, ou decrescente. Quando temos um número ímpar de valores, a mediana é exatamente o valor central. Quando o número de valores é par, a mediana é a média aritmética dos dois valores centrais. • Moda é o valor que mais aparece em uma série de dados.

NA PRÁTICA UNIÃO DE EVENTOS

Em uma reunião de 50 homens, 20 estão de gravata, 25 estão de colete e 15 usam gravata e colete. Ao sortear ao acaso um desses homens, qual é a probabilidade de ele vestir colete ou gravata? Vamos representar os conjuntos por um diagrama de Venn.

A

B

A B

S

• O espaço amostral (ou universo) é S = 50; • Desse total, 20 estão de gravata (vamos chamar de conjunto A) e 25, de colete (conjunto B); • Entre esses dois conjuntos, está A ∩ B, de homens que estão de gravata e, também, de colete (15); • E outros 20 não estão nem de gravata, nem de colete. Para encontrar o número de homens com gravata ou colete (união de A e B), somamos os elementos de A e B: 20 + 25 = 45. Mas temos de eliminar aqueles que estão, ao mesmo tempo de gravata e de colete (o conjunto A ∩ B). Então 45 – 15 = 30. A probabilidade de sortear um desses 30 homens é dada pela fração

30 P (E) = 50 = 60%

NA PRÁTICA MÉDIA, MEDIANA E MODA

Veja o diagrama de Venn com os valores: A

As notas dos alunos de uma classe são 4, 2, 9, 5, 6, 2, 7, 2, 6 . • A média aritmética das notas é a soma das notas dividida pelo total de notas: 4 + 2 + 9 + 5 + 6 + 2 + 7 + 2 + 6 / 9 = 43/9 = 4,7

B 5

15

10

20

• Para encontrar a mediana, devemos colocar as notas em ordem crescente: 2, 2, 2, 4, 5, 6, 6, 7, 9 A mediana é a nota no centro da lista: Med = 5 • A moda é mais fácil ainda de ser definida: é a nota que aparece mais vezes na lista: 2

118 GE MATEMÁTICA 2017

Repare que fora dos conjuntos A e B estão 20 homens que não usam nem gravata, nem colete. O espaço amostral é a soma de todos os valores: 20 + 5 + 15 + 10 = 50.


SAIBA MAIS PRATICAMENTE IMPOSSÍVEL

Jogos de loteria são eventos aleatórios que podem ser considerados jogos de azar porque as chances de ser contemplado são muito pequenas. A Mega-Sena, por exemplo: para vencer, o apostador tem de acertar a combinação de seis números num espaço amostral de 60 números. A probabilidade de acertar com uma aposta simples, de 6 números, é calculada assim: Qual a chance de fazer a Mega-Sena apostando em apenas seis números? O primeiro passo é verificar qual o número total de possibilidades de aposta, ou seja, quantas combinações de seis números o apostador pode fazer, entre 1 e 60. Lembre que:

C n, k =

n! (n – k) !k!

• n = 60 (total de elementos do conjunto) • k = 6 (número de elementos que devem ser combinados a cada vez)

C 60, 6 =

60! (60 – 6) ! . 6!

60! C 60, 6 = 54! . 6! O fatorial de 60 indica a multiplicação de todos os números naturais, em ordem decrescente, do 60 até 1. Então: 60! = 60 . 59 . 58 . 57 . 56 . 55 .(...) . 3 . 2 . 1 Uma vez que podemos indicar o fatorial em qualquer ponto da operação, simplificamos a divisão: 60! = 60 . 59 . 58 . 57 . 56 . 55 . 54!

60 . 59 . 58 . 57 . 56 . 55 . 54! Assim, C 60, 6 = 54! . 6! C 60, 6 =

60 . 59 . 58 . 57 . 56 . 55 . 54! 54! . 6 . 5 . 4 . 3 . 2 .1

Fazendo as simplificações:

C 60, 6 = C 60, 6 =

60 . 59 . 58 . 57 . 56 . 55 . 54! 54! . 6 . 5 . 4 . 3 . 2 .1 600 766 320 = 50 063 860 12

Portanto, na Mega-Sena é possível obter 50 063 860 combinações de seis números escolhidos entre 1 e 60. Essas são todas as possibilidades. Calculemos, agora, a probabilidade de serem sorteados exatamente os seis números em que se apostou. Ou seja, do total de 50 063 860 possibilidades, só um resultado é válido. Então, a probabilidade de acertar a sena é de 1 em 50 063 860: 1 1 Probabilidade = C 60, 6 = 50 063 860 , 0,000002%

A chance de acertar a sena é, aproximadamente, de 1 em 50 milhões de apostas. Qual a chance de fazer a quina (acertar cinco números) apostando apenas em seis números? Calculamos, primeiro, as possíveis combinações dos seis números sorteados, cinco a cinco:

6! 6 . 5! C 6, 5 = = (6 – 5) ! . 5! 1! . 5! Existem seis possíveis combinações dos seis números sorteados, cinco a cinco. Só que cada uma dessas seis possíveis combinações deve “casar” com um sexto número que não será sorteado. Se a cada concurso são sorteados seis números dentre 60, o total de números não sorteados é 54. Existem, então, 54 possibilidades de apostar num número não sorteado. Em linguagem matemática:

C 54, 1 =

54! 54 . 53! = 53! = 54 (54 – 1) ! .1!

Finalmente, calculemos o total de possíveis combinações de cinco números que devem ser sorteados e um que não deve ser sorteado:

C 6, 5 . C 54, 1 = 6 . 54 = 324 Isso significa que, em cada concurso da MegaSena, o apostador tem 324 possíveis combinações de cinco números entre os seis sorteados e um que não será sorteado. Agora, vamos calcular a probabilidade de acertar cinco números. Para fazer a quina, é preciso que os seis números sorteados coincidam exatamente com uma dessas 324 possibilidades. Ou seja, é preciso que uma – e apenas uma – dessas 324 possibilidades seja

sorteada, do total de 50 063 860 possíveis resultados do sorteio:

Probabilidade =

C 6, 5 . C 54,1 324 C 60, 6 = 50 063 860

Então, a chance de alguém acertar a quina é de aproximadamente 1 a cada 150 000 apostas – ou seja, de 0,00065%. Por fim, qual a chance de alguém fazer a quadra (acertar quatro números) apostando em seis números? Seguindo o mesmo raciocínio desenvolvido para a quina, a aposta deve ter exatamente 4 dos 6 números sorteados (C6,4 ) e 2 entre os 54 não sorteados (C54,2 ). Assim, o número de combinações possíveis para uma quadra é dado pela expressão:

C 6, 4 . C 54, 2 C 6, 4 .C 54, 2 =

6! 54! . (6 – 4) ! . 4! (54 – 2) ! . 2!

6 . 5 . 4! 54 . 53 . 52! C 6, 4 .C 54, 2 = 2! . 4! . 52! . 2! 6 . 5 . 4! 54 . 53 . 52! C 6, 4 . C 54, 2 = 2 .1 . 4! . 52! . 2 .1 Fazendo as simplificações:

6 . 5 . 4! 54 . 53 . 52! C 6, 4 . C 54, 2 = 2 .1 . 4! . 52! . 2 .1 6 . 5 54 . 53 C 6, 4 . C 54, 2 = 2 . 2 C 6, 4 . C 54, 2 = 15 .1 431 = 21 465 Portanto, o apostador tem 21 465 combinações possíveis para fazer a quadra. A aposta só será vencedora se for sorteada exatamente uma dessas 21 465 combinações, entre todas as 50 063 860 possibilidades do sorteio:

Probabilidade =

C 6, 4 . C 54, 2 C 60, 6 =

1 21 465 = 50 063 860 = 2 332 Então, a chance de um apostador fazer a quadra num concurso da Mega-Sena é de 1 em 2 332 vezes, ou seja, cerca de 0,043%. GE MATEMÁTICA 2017

119


72

COMO CAI NA PROVA

1. (IFSP 2016) O gráfico abaixo apresenta informações sobre os números de

livros lidos no mês passado pelos alunos de uma determinada turma. Sabendo-se que a informação de todos os alunos consta nesse gráfico, e que não há aluno que leu mais de 3 livros, utilize-o para responder as questões abaixo. Nenhum

incluindo repetições, e os 10 algarismos, também incluindo repetições. Admita ainda que, no novo sistema, cada carro do Mercosul tenha uma sequência diferente de letras e algarismos em qualquer ordem. Veja alguns exemplos das novas placas.

3

1 livro

10

2 livros 3 livros

15 12

I) A média do número de livros lidos no mês passado por essa turma é exatamente: a) 2,6 b) 1,5 c) 1,9 d) 2,05 e) 1,73

No novo sistema descrito, calcule o total de placas possíveis com o formato “Letra-Letra-Algarismo-Algarismo-Algarismo-Letra-Letra”, nessa ordem. Em seguida, calcule o total geral de possibilidades de placas com 4 letras (incluindo repetição) e 3 algarismos (incluindo repetição) em qualquer ordem na placa. Deixe suas respostas finais em notação de produto ou de fatorial.

RESOLUÇÃO II) Escolhido aleatoriamente um aluno dessa turma, a probabilidade de o aluno escolhido não ter lido livro no mês passado é: a) 3,5% b) 2,75% c) 2,5% d) 1,75% e) 7,5%

RESOLUÇÃO I)Para calcular a média de livros lidos pela turma precisamos dividir o total de livros lidos pela quantidade de alunos na turma. No gráfico, descobrimos o número de alunos: 3 + 10 + 15 + 12 = 40 alunos. A quantidade total de livros encontramos calculando o produto entre a quantidade de leitores e a quantidade de livros e, depois, somando tudo: 3 . 0 + 10 . 1 + 15 . 2 + 12 . 3 = 76 livros. A média de livros lidos por aluno nesse mês é M = 76 = 1,9 40 Resposta: C

Para a primeira parte da questão: • para cada posição com letra, temos 26 possibilidades (o alfabeto completo). Como temos quatro posições com letras, pelo princípio multiplicativo ficamos inicialmente com 26 . 26 . 26 . 26 = 26 4; • para cada posição com número, temos 10 possibilidades (0 a 9). Há três posições com números. Novamente pelo princípio multiplicativo, temos, então: 10 . 10 . 10 = 10 3. Resposta: No total, temos 264 . 10³ placas no formato“Letra-Letra-AlgarismoAlgarismo-Algarismo-Letra-Letra”. A segunda parte da questão: Se as posições Letra/Algarismo não forem fixas, podemos ter: • uma combinação de 4 letras em 7 possíveis posições, ou seja, combinações de 7 posições, tomadas quatro a quatro, ou • uma combinação de 3 algarismos em 7 possíveis posições, ou seja, combinações de 7 posições, tomadas três a três. Essas duas possibilidades geram o mesmo número de combinações, 35. C7,4 = C7,3 = 7! = 7 . 6 . 5 . 4! = 35 4!3! 4!3!

II) A probabilidade de ocorrer um evento (E) é dada pela expressão P (E) = n (E) , em que n é o número de eventos e S, o espaço amostral. n (S) S é o total de alunos da classe: 40. O evento (E) “não leu nenhum livro” vale 3 (três alunos apenas não leram nenhum livro). Então temos: P (E) = 3 / 40 = 0,075. As alternativas são dadas em porcentagem. Transformando 0,075 em porcentagem, ficamos com 7,5% Resposta: E

Segundo o princípio multiplicativo, nesse caso, teremos 35 vezes mais possibilidades de placas que o resultado obtido anteriormente, ou seja, teremos um total de 35 . 26 4 . 10³ placas. Resposta: Para 4 letras (incluindo repetição) e 3 algarismos (incluindo repetição) em qualquer ordem, temos 35 . 26 4 . 10³ placas possíveis.

2. (Unesp 2016) Está previsto que, a partir de 1º de janeiro de 2017, entrará em

RESOLUÇÃO

vigor um sistema único de emplacamento de veículos para todo o Mercosul, o que inclui o Brasil. As novas placas serão compostas por 4 letras e 3 algarismos. Admita que no novo sistema possam ser usadas todas as 26 letras do alfabeto,

120 GE MATEMÁTICA 2017

3. (UCS 2016) Um supermercado está selecionando, entre 15 candidatos que se apresentaram, 3 funcionários para desempenhar a função de “caixa”. De quantas maneiras diferentes pode ser feita essa escolha? a) 5 b) 45 c) 215 d) 360 e) 455

A questão é resolvida pela árvore das possibilidades. Há 15 candidatos para apenas três vagas. Para a primeira vaga, temos 15 possibilidades; para a segunda, 14 possibilidades e para a terceira, 13 possibilidades. Temos, então, de 15 . 14 . 13 = 2 730 possíveis combinações.


RESUMO

Como as vagas se destinam ao mesmo cargo (caixa de supermercado), a ordem não importa e precisamos descontar isso ao final, dividindo o total obtido acima por 3 . 2 . 1 = 6. O número total de diferentes maneiras de ocupar as vagas será de 2 730 : 6 = 455. Resposta: E

4. (Fatec 2016) No boxe, um dos esportes olímpicos, um pugilista tem à

sua disposição quatro golpes básicos: o jab, o direto, o cruzado e o gancho. Suponha que um pugilista, preparando-se para os Jogos Olímpicos do Rio, em 2016, queira criar uma sequência com 6 golpes, empregando necessariamente dois jabs, dois diretos, um cruzado e um gancho. Assim, o número máximo de sequências que ele poderá criar será de a) 180 b) 160 c) 140 d) 120 e) 100 Lembre-se de que: Para permutação com repetição n! Pnk1 ,k2 ,k3 , ... = k1!k2!k3!...

RESOLUÇÃO O enunciado fornece a fórmula para a permutação com repetição, em que n é o número total de golpes na sequência a ser criada, ou seja, n = 6, e k1 , k2 , k3 e k4 são, respectivamente, as quantidades de repetições de jabs, diretos, cruzados e ganchos. Pelo enunciado, k1 = 2 jabs, k2 = 2 diretos, k3 = 1 cruzado e k4 = 1 gancho. Substituindo os valores na fórmula, temos: P62,2,1,1 = 6! = 6 . 5. 4 . 3 . 2! = 180 2!2!1!1! 2!2! Resposta: A

5. (USF2016) Em um grande hospital, há 500 leitos e todos estão ocupados.

Uma das alas desse hospital é destinada a pessoas com HIV positivo. 40% dos internados são mulheres e sabe-se que, entre elas, 10% são HIV positivo. Entre os homens internados nesse hospital, 15% são HIV positivo. Escolhido um paciente ao acaso, qual a probabilidade de ele ser HIV positivo?

RESOLUÇÃO A questão exige a leitura atenta do enunciado para a coleta das informações necessárias à resolução. • O número de pacientes internados é 500; • As mulheres representam 40% dos leitos ocupados. Portanto: 0,4 . 500 = 200 pacientes são mulheres; • Dessas 200 mulheres internadas, 10% são portadoras do HIV. Calculando 10% sobre 200, temos que 20 mulheres são soropositivas no hospital. • Se as mulheres ocupam 40% dos leitos, os retantes 60% são de homens 0,6 . 500 = 300 pacientes; • Desses, 15% são portadores do vírus HIV, o que resulta em um total de 0,15 . 300 = 45 homens soropositivos. Desse modo, o total de pacientes, homens ou mulheres, portadores do vírus HIV é de 20 + 45 = 65 pacientes. A probabilidade de se sortear, entre os 500 pacientes, um desses 65 é P = 65 . 100% = 13% 500

Lorem ipsondolor Combinatória e probabilidade S

n! PERMUTAÇÃO GIAMCORE MAGNA SIMPLES: accum Pam, core feum auguerit, si n = vullam, blam, quat. Lor sequat lorerci tem accum il ulput nummy nit n! b, c, ... P a, = PERMUTAÇÃO COM REPETIÇÃO: nullam adit ea ad tetumsan hent lor ninit adionsequip a!b!c!... exeros do dolor sum zzrit amcorer sustrud dui et autpatin eugue ven! lenim vulluptate ARRANJO SIMPLES: consectem A n, p =zzrit wismod el ulputatum incing (n – p) ! et lutdiamcom molumsandip.

ARRANJO COMDOLOBOR EAFACIDUNT REPETIÇÃO: sustrud A rn, p magna = n p feugiam veniam zzrilit luptatem iriusto consequi eraesto eugait luptat do ese tat dolut venis amconsed mincillandre commodi onullan ver A n, p n! C n, p =corp!iliquat. = COMBINAÇÃO SIMPLES: sustrud modigniam ipsuscillam, (n – p) !p! Num volobor eraestionum ing eniatummy nulputem vent amet iusto odignim quisisA adiam PROBABILIDADE: probabilidade aliquat vel deesequip um evento ocorrer é uma fração de todas as possibilidades de ocorrer o evento: IS NULLAnFEUGAIT aut venim nostrud min ut wissecte magni(E) P (E) = bh et nimnincillandre do commy non hendip eu feugait lobore (S) magnim am, quisciduis nulluptatum venit in velendi gnissenit, sequat. Equat. EVENTOS MUTUAMENTE Ut iliscidunt la commy EXCLUSIVOS nostion hendiam são aqueles commod em dit velendrero diat,develum ingelimina ex elit at pratin esectet nonullan que a ocorrência completamente a possiheniam doloreet amcore do facil utpat. Osto odiamet, velent bilidade de ocorrência deeuqualquer outro. Entre eventos pratet nosto desse tipo, aconsequisl soma dasullandrem probabilidades quat amédolorem sempreveliquatue igual a 1, minseja, ou velesequam 100%. nonse facipisim zzriure. RCILIQUATET VULLAN ute commy nullaorem ip ero consectet EVENTOS INDEPENDENTES são aqueles em que o resultado lumum de velnão ulput influi veliquis no resultado exerosting dosendreros demais. aut A probabilidade ilis at. Lesto dode lorperci independentes eventos tio dolutpat ullaore acontecerem riurerit insimultaneamente henim iusci bla at. é igual Gait ao produto das probabilidades de cada deles acontecer atummolore tie te er ipisim dit wisl ipsumum dunt velis aliquat. isoladamente: P (A + B) = P (A) .P (B) NONUMMO LOBORERO etumsandrem dolorperatem do duis acidunt velCOMPLEMENTARES EVENTOS ullamet nosto coreetsão alisaqueles aliquipit nos vent quais adignisim a interipsuscipit Del ut lutatéaute mincill andipsustis do exeraestrud secção dosinconjuntos um conjunto vazio, e a união coincide eum o nissed essequat nonulput volore er ip elenit ing com espaço amostral: P(A) + P(B) = 1tem (ou adit 100%) et irilit iureet laorem veraess equisi. Ecte vulla commy nullam, sis nulluptat, sumEVENTOS: venibh elesto facilit nit lorem UNIÃO DE DOIS É aconum soma nonulla das probabilidades de delesto cada ea feui blandre eui tetexcluídas lam ocorrer um dos eventos, as probabilidades de os dois eventos ocorrerem ao mesmo tempo: IS NULLA aut>venim min ut wissecte magniP (A , B) =FEUGAIT P (A) + P (B) P (A + nostrud B) bh et nim incillandre do commy non hendip eu feugait lobore magnim am, FATORIAL É aquisciduis multiplicação nulluptatum de números venitnaturais, in velendi emgnissenit, sequênsequat. cia decrescente. Equat. Ut n! iliscidunt = n . (n –la1) commy . (n – 2)nostion . (...) . 3hendiam . 2 . 1, para commod n ≥ 2. dit velendrero diat, vel ing ex elit at pratin esectet nonullan heniam doloreet do eu facil utpat. MÉDIA, MEDIANAamcore E MODA Média aritmética é a soma de todos os valores de um conjunto RCILIQUATET dividida pelo número VULLANtotal ute commy de elementos nullaorem de um ip ero conjunto. consectet lum vel ulput veliquis exerosting endreros aut ilis at. Lesto dolorperciponderada tio dolutpatconsidera ullaore riurerit bla at. Gait Média o pesoindehenim cada iusci elemento. atummolore tie te er ipisim dit wisl ipsum dunt velis aliquat. Mediana é a medida central dos valores listados em ordem NONUMMO crescente. EmLOBORERO uma lista com etumsandrem número pardolorperatem de valores, a mediana do duis éacidunt a médiavel aritmética ullamet nosto dos dois coreet valores aliscentrais. aliquipit vent adignisim ipsuscipit in Del ut lutat aute mincill andipsustis do exeraestrud eum nissed essequat nonulput volore temsérie adit er elenit. Moda é o valor que mais aparece em uma deipdados.

Resposta: A probabilidade de se escolher ao acaso um paciente portador do HIV é de 13%. GE MATEMÁTICA 2017

121


8

MATRIZES CONTEÚDO DESTE CAPÍTULO

Conceitos e propriedades............................................................................124 Determinantes ................................................................................................126 Como cai na prova + Resumo .....................................................................128

Sai a TV analógica, entra a digital A partir de 2018, para assistir à televisão o brasileiro terá de adquirir um aparelho novo, ou adaptar o antigo. Em troca, receberá imagem e sons de melhor qualidade

O

governo federal promete que até o final de 2018 serão cancelados os sinais analógicos das transmissões de TV. A partir de então, todo o território nacional receberá apenas sinais digitais. A mudança exige investimento tanto das emissoras quanto dos telespectadores. As primeiras precisam adquirir equipamentos para a emissão; os segundos, aparelhos de TV que captam os sinais digitais, ou conversores de sinal. A vantagem para a população é o acesso a imagem e som de melhor qualidade. A troca de um sistema por outro vai permitir, ainda, liberar a faixa de frequência de 700 MHz, hoje ocupada pelos sinais analógicos da TV, para outros usos – a telefonia 4G –, melhorando a qualidade do sistema de comunicação do país, como um todo. A diferença entre analógico e digital está na forma como os sinais eletromagnéticos são emitidos e recebidos. O analógico é transmitido de modo contínuo. Esses sinais poderiam ser representados como ondas, como as sonoras, ou as de um gráfico das funções seno ou cosseno (veja o capítulo 5). Se a amplitude varia, digamos, entre 0 e 10, as ondas passam por todos os valores intermediários (0,001, ... 0,24, ... 9,877...). Como o número de valores entre qualquer número inteiro é infinito, os sinais analógicos variam

122 GE MATEMÁTICA 2017

muito ao longo do tempo. E, por isso, estão sujeitos a oscilações e interferências – o que, num aparelho de TV, aparece como chuviscos, faixas e chiados. Já os sinais digitais são transmitidos de maneira descontínua, como um bip-bip, no qual cada bip assume um valor discreto. Por exemplo, entre 0 e 10, um sinal digital só pode ter valor 0, 1, 2, 3, ... ou 10. Se fossem representados em um gráfico, esses sinais apareceriam como barras (veja o capítulo 2). Com essa limitação nos valores possíveis, os sinais digitais são menos vulneráveis a interferências – daí a maior nitidez de imagem e qualidade do som. Na tela de uma TV, os sinais digitais podem ser representados como uma matriz de cores, semelhante às matrizes matemáticas. Nessa matriz, cada ponto tem um valor, que depende de sua posição – ou seja, em que linha e em que coluna ele se encontra. As cores e as tonalidades das SUCATA imagens são criadas Até 2018 todos os lares assim, ponto a ponto. brasileiros deverão ter Neste capítulo você aposentado os televisores revê os conceitos e as de tubo, ou adaptado os propriedades de ma- aparelhos para receber trizes e determinantes. sinais digitais


UNDERWORLD111/iSTOCK

GE MATEMÁTICA 2017

123


MATRIZES CONCEITOS E PROPRIEDADES

Prateleiras matemáticas Matrizes são como tabelas, em que as células são organizadas em linhas e colunas

P

artindo direto de um exemplo você entende facilmente o conceito de matriz. As duas tabelas abaixo foram montadas pelo proprietário de uma loja para controle do estoque. A loja comercializa camisetas com quatro estampas diferentes, em três tamanhos e em modelagem masculina e feminina.

1 1 1 1 1 1 1 1 1 1 1 1 1 1 1 1 1 1 1 1 1 1 1 1 1 1 1 1 1 1 1 1 1 1 1 1

1 1 1 1 1 1 1 1 1 1 1 1 1 1 1 1 1 1 1 1 1 1 1 1 1 1 1 1 1 1 1 1 1 1 1 1

1 1 1 1 1 1 1 1 1 1 1 1 1 1 1 1 1 1 1 1 1 1 1 1 1 1 1 1 1 1 1 1 1 1 1 1

1 1 1 1 1 1 1 1 1 1 1 1 1 1 1 1 1 1 1 1 1 1 1 1 1 1 1 1 1 1 1 1 1 1 1 1

1 1 1 1 1 1 1 1 1 1 1 1 1 1 1 1 1 1 1 1 1 1 1 1 1 1 1 1 1 1 1 1 1 1 1 1

1 1 1 1 1 1 1 1 1 1 1 1 1 1 1 1 1 1 1 1 0 0 1 1 1 1 1 1 1 1 1 1 1 1 1 1

1 1 1 1 1 1 1 1 1 1 1 1 1 1 1 1 1 1 1 0 0 0 0 1 1 1 1 1 1 1 1 1 1 1 1 1

1 1 1 1 1 1 1 1 1 1 1 1 1 1 1 1 1 1 0 0 0 1 0 1 1 1 1 1 1 1 1 1 1 1 1 1

1 1 1 1 1 1 1 1 1 1 1 1 1 1 1 1 1 0 0 0 1 1 0 1 1 1 1 1 1 1 1 1 1 1 1 1

1 1 1 1 1 1 1 1 1 1 1 1 1 1 1 1 0 0 0 0 1 1 0 1 1 1 1 1 1 1 1 1 1 1 1 1

1 1 1 1 1 1 1 1 1 1 1 1 1 1 1 0 0 0 0 0 1 1 0 1 1 1 1 1 1 1 1 1 1 1 1 1

1 1 1 1 1 1 1 1 1 1 1 1 1 1 0 0 0 1 0 1 1 1 0 1 1 1 1 1 1 1 1 1 1 1 1 1

1 1 1 1 1 1 1 1 1 1 1 1 1 0 0 0 1 1 0 1 1 1 0 1 1 1 1 1 1 1 1 1 1 1 1 1

1 1 1 1 1 1 1 1 1 1 1 1 0 0 0 1 1 1 0 1 1 1 0 1 1 1 1 1 1 1 1 1 1 1 1 1

1 1 1 1 1 1 1 1 1 1 1 1 0 0 0 1 1 1 0 1 1 1 0 1 1 1 1 1 1 1 1 1 1 1 1 1

1 1 1 1 1 1 1 1 1 1 1 1 0 0 0 1 1 0 0 0 1 1 0 1 1 1 1 1 1 1 1 1 1 1 1 1

1 1 1 1 1 1 1 1 1 1 1 0 0 0 0 1 0 0 0 0 1 1 0 1 1 1 1 1 1 1 1 1 1 1 1 1

1 1 1 1 1 1 1 1 1 1 1 0 0 0 0 0 0 0 0 0 1 1 0 1 1 1 1 1 1 1 1 1 1 1 1 1

1 1 1 1 1 1 1 1 1 1 0 0 0 0 0 0 0 0 0 0 1 1 0 1 1 1 1 1 1 1 1 1 1 1 1 1

1 1 1 1 1 1 1 1 1 1 0 0 0 0 0 0 0 0 0 0 1 1 0 1 1 1 1 1 1 1 1 1 1 1 1 1

1 1 1 1 1 1 1 1 1 0 0 0 0 0 0 0 0 0 0 0 0 1 0 1 1 1 1 1 1 1 1 1 1 1 1 1

1 1 1 1 1 1 1 1 1 0 0 0 0 0 0 0 0 0 0 0 0 0 0 1 1 1 1 1 1 1 1 1 1 1 1 1

1 1 1 1 1 1 1 1 1 0 0 0 0 0 0 0 0 0 0 0 0 0 0 0 1 1 1 1 1 1 1 1 1 1 1 1

1 1 1 1 1 1 1 1 1 0 0 0 0 0 0 0 0 0 0 0 0 0 0 0 1 1 1 1 1 1 1 1 1 1 1 1

1 1 1 1 1 1 1 1 1 0 0 0 0 0 0 0 0 0 0 0 0 0 0 0 0 1 1 1 1 1 1 1 1 1 1 1

1 1 1 1 1 1 1 1 1 0 0 0 0 0 0 0 0 0 0 0 0 0 0 0 0 1 1 1 1 1 1 1 1 1 1 1

1 1 1 1 1 1 1 1 1 0 0 0 0 0 0 0 0 0 0 0 0 0 0 0 0 0 1 1 1 1 1 1 1 1 1 1

1 1 1 1 1 1 1 1 0 0 0 0 0 0 0 0 0 0 0 0 0 0 0 0 0 0 1 1 1 1 1 1 1 1 1 1

1 1 1 1 1 1 1 0 0 0 0 0 0 0 0 0 0 0 0 0 1 0 0 0 0 0 1 1 1 1 1 1 1 1 1 1

1 1 1 1 1 1 0 0 0 0 0 0 0 0 0 0 0 0 0 0 1 1 1 0 0 0 1 1 1 1 1 1 1 1 1 1

1 1 1 1 1 0 0 0 0 0 0 0 0 0 0 0 0 0 0 0 1 1 1 1 1 1 1 1 1 1 1 1 1 1 1 1

1 1 1 1 0 0 0 0 0 0 0 0 0 0 0 0 0 0 0 0 1 1 1 1 1 1 1 1 1 1 1 1 1 1 1 1

1 1 1 0 0 0 0 0 0 0 0 0 0 0 0 0 0 0 0 0 1 1 1 1 1 1 1 1 1 1 1 1 1 1 1 1

1 1 0 0 0 0 0 0 0 0 0 0 0 0 0 0 0 0 0 0 1 1 1 1 1 1 1 1 1 1 1 1 1 1 1 1

1 0 0 0 0 0 0 0 0 0 0 0 0 0 0 0 0 0 0 0 1 1 1 1 1 1 1 1 1 1 1 1 1 1 1 1

1 1 1 1 1 1 1 0 0 0 0 0 0 0 0 0 0 0 1 0 1 1 1 1 1 1 1 1 1 1 1 1 1 1 1 1

1 1 1 1 1 1 1 1 0 0 0 0 0 0 0 0 0 1 1 0 1 1 1 1 1 1 1 1 1 1 1 1 1 1 1 1

1 1 1 1 1 1 1 1 1 0 0 0 0 0 0 0 1 1 1 0 1 1 1 1 1 1 1 1 1 1 1 1 1 1 1 1

1 1 1 1 1 1 1 1 1 0 0 0 0 0 0 0 1 1 1 0 1 1 1 1 1 1 1 1 1 1 1 1 1 1 1 1

1 1 1 1 1 1 1 1 1 0 0 0 0 0 0 1 1 1 1 0 1 1 1 1 1 1 1 1 1 1 1 1 1 1 1 1

1 1 1 1 1 1 1 1 1 0 0 0 0 0 0 1 1 1 0 0 1 1 1 1 1 1 1 1 1 1 1 1 1 1 1 1

1 1 1 1 1 1 1 1 1 0 0 0 0 0 0 1 1 1 0 0 1 1 1 1 1 1 1 1 1 1 1 1 1 1 1 1

1 1 1 1 1 1 1 1 1 1 0 0 0 0 0 1 1 1 0 0 1 1 1 1 1 1 1 1 1 1 1 1 1 1 1 1

1 1 1 1 1 1 1 1 1 1 0 0 0 0 0 1 1 1 0 0 1 1 1 1 1 1 1 1 1 1 1 1 1 1 1 1

1 1 1 1 1 1 1 1 1 1 1 0 0 0 0 0 1 1 0 0 1 1 1 1 1 1 1 1 1 1 1 1 1 1 1 1

1 1 1 1 1 1 1 1 1 1 1 1 0 0 0 0 1 1 0 0 1 1 1 1 1 1 1 1 1 1 1 1 1 1 1 1

1 1 1 1 1 1 1 1 1 1 1 1 1 0 0 0 0 1 0 0 1 1 1 1 1 1 1 1 1 1 1 1 1 1 1 1

1 1 1 1 1 1 1 1 1 1 1 1 1 0 0 0 0 0 0 0 1 1 1 1 1 1 1 1 1 1 1 1 1 1 1 1

1 1 1 1 1 1 1 1 1 1 1 1 1 1 0 0 0 0 0 0 0 1 1 1 1 1 1 1 1 1 1 1 1 1 1 1

1 1 1 1 1 1 1 1 1 1 1 1 1 1 1 0 0 0 0 0 0 1 1 1 1 1 1 1 1 1 1 1 1 1 1 1

1 1 1 1 1 1 1 1 1 1 1 1 1 1 1 1 0 0 0 0 0 0 1 1 1 1 1 1 1 1 1 1 1 1 1 1

1 1 1 1 1 1 1 1 1 1 1 1 1 1 1 1 1 0 0 0 0 0 0 1 1 1 1 1 1 0 0 0 0 0 1 1

1 1 1 1 1 1 1 1 1 1 1 1 1 1 1 1 1 1 0 0 0 0 0 0 1 1 1 1 0 0 1 1 1 0 0 1

1 1 1 1 1 1 1 1 1 1 1 1 1 1 1 1 1 1 1 0 0 0 0 0 1 1 1 0 0 1 1 1 0 0 1 1

1 1 1 1 1 1 1 1 1 1 1 1 1 1 1 1 1 1 1 1 0 0 0 0 0 0 0 0 1 1 1 0 0 1 1 1

1 1 1 1 1 1 1 1 1 1 1 1 1 1 1 1 1 1 1 1 1 0 0 0 0 0 1 1 1 1 0 0 1 1 1 1

1 1 1 1 1 1 1 1 1 1 1 1 1 1 1 1 1 1 1 1 1 1 0 0 0 0 0 1 1 1 0 0 1 1 1 1

1 1 1 1 1 1 1 1 1 1 1 1 1 1 1 1 1 1 1 1 1 1 1 0 0 0 0 0 1 1 1 0 0 1 1 1

1 1 1 1 1 1 1 1 1 1 1 1 1 1 1 1 1 1 1 1 1 1 1 1 1 1 1 0 0 1 1 1 0 0 1 1

1 1 1 1 1 1 1 1 1 1 1 1 1 1 1 1 1 1 1 1 1 1 1 1 1 1 1 1 0 0 1 1 1 0 0 1

1 1 1 1 1 1 1 1 1 1 1 1 1 1 1 1 1 1 1 1 1 1 1 1 1 1 1 1 1 0 0 0 0 0 1 1

1 1 1 1 1 1 1 1 1 1 1 1 1 1 1 1 1 1 1 1 1 1 1 1 1 1 1 1 1 1 1 1 1 1 1 1

1 1 1 1 1 1 1 1 1 1 1 1 1 1 1 1 1 1 1 1 1 1 1 1 1 1 1 1 1 1 1 1 1 1 1 1

1 1 1 1 1 1 1 1 1 1 1 1 1 1 1 1 1 1 1 1 1 1 1 1 1 1 1 1 1 1 1 1 1 1 1 1

1 1 1 1 1 1 1 1 1 1 1 1 1 1 1 1 1 1 1 1 1 1 1 1 1 1 1 1 1 1 1 1 1 1 1 1

Modelagem feminina Tamanho / Estampa lisa

flores dragão pirata

P

10

5

0

16

M

15

12

10

14

G

12

7

16

11

Modelagem masculina Tamanho / Estampa

lisa

flores dragão pirata

P

8

4

8

12

M

7

13

10

20

G

15

8

16

2

Cada uma dessas tabelas corresponde a uma matriz: Para as camisetas femininas, a matriz A1: 10 5 0 16 15 12 10 14 A1= 12 7 16 11 Para as camisetas masculinas, a matriz A2: 8 4 8 12 7 13 10 20 A2= 15 8 16 2 Analisando as duas matrizes: As matrizes trazem apenas células com valores numéricos. Não há cabeçalho ou título, como numa tabela. Isso porque quem criou a

124 GE MATEMÁTICA 2017

matriz sabe exatamente o que significa cada linha e cada coluna. E as linhas e colunas trazem valores ordenados e posicionados segundo a lógica do criador da tabela; Uma matriz pode ser escrita dentro de parênteses ou de colchetes, não faz diferença; A1 e A2 têm três linhas e quatro colunas. Então, são matrizes de ordem 3 x 4. Na notação de uma matriz genérica, o número total de linhas é chamando m; o número de colunas, n; Numa matriz de ordem 3 x 4, o número total de elementos é 12; Cada valor da matriz ocupa uma casa única, no cruzamento de determinada linha com determinada coluna. Então, qualquer valor está no cruzamento m x n; Para uma matriz genérica A, dizemos então que o elemento aij está no cruzamento da linha i com a coluna j. Por exemplo, na matriz A1, do nosso exemplo, o elemento de valor 5 ocupa a primeira linha e a segunda coluna. Dizemos então que, em A1, a12 = 5 (lê-se “a um-dois é igual a 5”). A notação A = (aij)m x n indica que a matriz A é formada pelos elementos aij. Se A tem m linhas e n colunas, então 1 ) i ) m e 1 ) j ) n, sendo que m e n ∈ N.

IMAGEM EM NÚMEROS A figura desenhada pelo código binário, utilizado em informática, é uma matriz. Cada posição tem um único valor: ou 1, ou 0

1 1 1 1 1 1 1 1 1 1 1 1 1 1 1 1 1 1 1 1 1 1 1 1 1 1 1 1 1 1 1 1 1 1 1 1

VITOR INOUE

8


Ou seja, qualquer elemento a da matriz tem de estar numa posição i x j que seja menor ou igual ao número total de linhas (m) e colunas (n).

em reais. O custo de cada peça varia segundo a estampa – o tamanho não importa, segundo a tabela:

Soma de matrizes

Só é possível somar ou subtrair matrizes com o mesmo número de linhas e colunas. Seguindo com o exemplo da loja de camisetas: O estoque de camisetas por estampa e tamanho é dado pela soma das matrizes S = A1 + A2:

ATENÇÃO Repare que a célula ocupada por um valor é definida da mesma forma como definimos um ponto no plano cartesiano, por um par ordenado (veja no capítulo Geometria)

liso

20

flores

25

dragão

30

pirata

40

A matriz correspondente a essa tabela é:

S=

8 4 8 12 7 13 10 20 15 8 16 2

20 P = 25 30 40

S=

10 + 8 15 + 7 12 + 15

5+4 12 + 13 7+8

0+8 10 + 10 16 + 16

S=

18 9 22 25 27 15

8 20 32

A matriz S, soma de A1 e A2: 18 9 8 28 S= 22 25 20 34 27 15 32 13

16 + 12 14 + 20 11 + 2

28 34 13

Multiplicação de matriz por um número k

Seguindo com nosso exemplo: quantas camisetas de cada tipo existiriam em estoque se o comerciante triplicasse a quantidade de cada uma delas? Basta multiplicar por 3 cada elemento da matriz S. Veja: 18 9 8 28 3 . S = 3 . 22 25 20 34 27 15 32 13 3 . 18 3.9 3.8 3 . S = 3 . 22 3 . 25 3 . 20 3 . 27 3 . 15 3 . 32 54 27 24 84 3 . S = 66 75 60 102 81 45 96 39

3 . 28 3 . 34 3 . 13

Multiplicação de matrizes Diferentemente do que ocorre com a multiplicação de números reais, com matrizes, em muitos casos, a ordem dos fatores pode alterar o produto.

Preço (R$)

10 5 0 16 15 12 10 14 12 7 16 11

A matriz S traz a soma de camisetas conforme o tamanho e a estampa. Por exemplo, existem 20 camisetas com estampa de dragão, no tamanho M (a23).

ATENÇÃO

Modelo

A multiplicação só é possível se o número de colunas de uma matriz for igual ao número de linhas da outra matriz; Assim, a matriz produto P da multiplicação de A1 por A2 tem o número de colunas de uma das matrizes e o número de linhas da outra; Os elementos de P são obtidos pela multiplicação dos elementos de cada linha de A1 pelos elementos correspondentes de cada coluna de A2. Depois, os resultados são somados.

Ainda no exemplo das camisetas: O comerciante quer saber o valor do estoque,

Repare: cada linha de S traz camisetas de um mesmo tamanho, mas com estampas diferentes. Se o valor em reais varia conforme a estampa, então cada valor das linhas de S deve ser multiplicado pelo valor correspondente de P. 20 25 30 40 18 . 20 + 9 . 25 + 8 . 30 + 28 . 40 S . P = 22 . 20 + 25 . 25 + 20 . 30 + 34 . 40 27 . 20 + 15 . 25 + 32 . 30 + 13 . 40 S.P=

18 22 27

1 945 S . P = 3 025 2 395

9 25 15

8 20 32

28 34 13

.

Valor total das camisetas P Valor total das camisetas M Valor total das camisetas G

Repare: A matriz P traz o valor, em reais, do estoque, conforme o tamanho das camisetas. O número de colunas da primeira matriz é igual ao número de linhas da segunda; S . P tem o número de linhas da primeira matriz e o número de colunas da segunda.

Matriz identidade

É uma matriz quadrada – ou seja, com número de linhas (i) igual ao número de colunas (j) – cujos elementos são 1 ou zero. 1 0 0 I= 0 1 0 0 0 1 Repare que o valor 1 aparece apenas numa diagonal entre as posições em que i = j (a11, a22, a33 etc.). Esta é a diagonal principal. Para as demais posições, o valor é zero. A multiplicação de uma matriz por sua matriz identidade é sempre igual à matriz original. GE MATEMÁTICA 2017

125


MATRIZES DETERMINANTES

REDHOUANE/iSTOCK

8

AS FACES DE UMA PIRÂMIDE Pirâmides, como as do Egito, têm quatro lados em forma de triângulo. O cálculo da área desse triângulo pode ser feito usando-se matrizes

Propriedades das matrizes quadradas Algumas operações com matrizes que têm o mesmo número de linhas e de colunas facilitam cálculos, inclusive em geometria

M

atrizes quadradas têm sempre associado a elas um número real, chamado determinante. Determinantes têm diversas aplicações em matemática – por exemplo, na definição da área de um triângulo (veja o quadro Na prática, na pág. ao lado). Dependendo da ordem da matriz quadrada, existe uma regra específica para encontrar o determinante.

Determinante de matrizes 2 x 2 Considere a matriz A= 1 2 3 4 O determinante é calculado em três passos: primeiro, multiplicamos os valores da diagonal principal; segundo, multiplicamos os valores da diagonal secundária; e, terceiro, subtraímos o produto da diagonal secundária do produto da diagonal principal. Visualizando a explicação: diagonal diagonal principal secundária det A = 1 2 = 1 . 4 – 2 . 3 = 4 – 6 = –2 3 4 Portanto, o determinante da matriz A é –2.

126 GE MATEMÁTICA 2017

Determinantes de matrizes 3 x 3

Para matrizes quadradas de ordem 3, o caminho para encontrar o determinante é um pouco diferente. Nesse caso, aplicamos a regra de Sarrus. Acompanhe o raciocínio, no exemplo abaixo, como se aplica essa regra, passo a passo. Considere a matriz B=

1 –2 0

2 3 4 1 5 2

Repetimos as duas primeiras colunas ao lado direito da matriz original. 1 2 3 1 2 det B = –2 4 1 –2 4 0 5 2 0 5 Multiplicamos os valores da diagonal principal e das diagonais paralelas a ela: 1 2 3 1 det B = –2 4 1 –2 0 5 2 0 8

2 4 5 0 –30

Somamos os produtos dessa multiplicação: 8 + 0 – 30 = –22


Fazemos o mesmo com a diagonal secundária: 1 det B = –2 0 0 5

2 3 1 4 1 –2 5 2 0 –8

NA PRÁTICA

2 4 5

ÁREA DE UM TRIÂNGULO

Somando os valores encontrados, temos: 0+5–8=–3

Qual a área do triângulo ABC da figura com vértices de coordenadas A (1, 1), B (4, 1) e C (1, 3)?

4

2 1

Propriedades dos determinantes

0 3 –5 12 2 0 4 =0 det B = 0 0 –8 1 3 0 4 1 –2 2. Se os elementos de duas linhas ou colunas de uma matriz A forem iguais ou proporcionais, então det A = 0. Ex: 1 det A = 1 –4

2 2 –2

3 3 –2

2 det B = 1 3

–4 7 –2 5 = 0 –6 1

=0

3. Se trocarmos a posição de duas linhas ou colunas, o valor do determinante tem o sinal invertido. Por exemplo: 4 3 = 4 . 5 – (–2 . 3) = 26 det A = –2 5 det B =

–2 5 = –2 . 3 – 4 . 5 = –26 4 3

C

3

Por fim, subtraímos os dois valores obtidos das duas diagonais: det B = –22 – (–3) = –22 + 3 = –19 Este é o determinante da matriz. Conhecer algumas das propriedades dos determinantes facilita muito os cálculos. 1. Se todos os elementos de uma linha ou coluna de uma matriz A forem nulos, det A = 0. Ex: det A = 1 2 3 0 0 0 =0 3 4 5

y

-2

-1

0

A 0

B 1

2

3

4

x 5

A área de um triângulo é dada pela fórmula A = base . altura 2 Portanto, para o triângulo medindo 2u e 3u, a área é 2.3=3 2 Agora veja a relação da área do triângulo com o determinante de uma matriz. • Montamos a matriz referente aos pontos que definem o triângulo no plano cartesiano: A (1, 1), B (4, 1) e C (1, 3). 1 1 1 C= 4 1 1 1 3 1 Repare que a primeira coluna traz os valores de x para A, B e C. A segunda coluna, os valores de y. A terceira coluna é completada apenas com 1. • Calculando o determinante de C: 1 1 1 11 det C = 4 1 1 4 1 = (1 + 1 + 12) – (1 + 3 + 4) = 14 – 8 = 6 1 31 13 Veja: Repetimos as duas primeiras colunas ao lado da matriz original. Fazendo a multiplicação das duas diagonais e subtraindo um valor pelo outro, temos: det C = (1 + 1 + 12) – (1 + 3 + 4) = 14 – 8 = 6 O determinante da matriz C é exatamente o dobro da área do triângulo. Detalhe: A área de uma figura é sempre um valor positivo. Por isso, o valor do determinante deve ser apresentado como módulo (valor absoluto)

ATENÇÃO

ATENÇÃO

Por padrão, o determinante de uma matriz é indicado por um par de barras paralelas ao redor da matriz original.

Quando o determinante se referir à área de um triângulo, o resultado será sempre dado em módulo do valor – ou seja, em valor absoluto. Isso porque área é uma grandeza física que nunca pode ter valor negativo. GE MATEMÁTICA 2017

127


82

COMO CAI NA PROVA

1. (IFPE 2016) Rodrigo, Otavio e Ronaldo gostam muito de comida japonesa e saíram para comer temaki, também conhecido como sushi enrolado à mão, cujo formato lembra o de um cone. Foram, então, visitando vários restaurantes, tanto no sábado quanto no domingo. As matrizes a seguir resumem quantos temakis cada um consumiu e como a despesa foi dividida: 3 2 0 2 3 0 S= 1 1 2 e D= 0 2 1 0 3 2 1 0 2

S refere-se às quantidades de temakis de sábado e D, às de domingo. Cada elemento aij nos dá o número de cones que a pessoa i pagou para a pessoa j, sendo Rodrigo o número 1, Otávio, o número 2 e Ronaldo, o número 3 (aij representa o elemento da linha i e da coluna j de cada matriz). Assim, por exemplo, no sábado, Rodrigo pagou 3 temakis que ele próprio consumiu (a11), 2 temakis consumidos por Otávio (a12) e nenhum por Ronaldo (a13), que corresponde à primeira linha da matriz S. Quantos temakis Otávio ficou devendo para Rodrigo neste fim de semana? a) nenhum b) 1 c) 2 d) 3 e) 4

RESOLUÇÃO Primeiro, vamos entender bem o enunciado: • você sabe que qualquer posição na matriz é definida como aij ; • o enunciado diz que cada amigo é representado por um número: 1 para Rodrigo, 2 para Otávio e 3 para Ronaldo. Esse número refere-se aos índices i e j, de cada posição (aij); • assim, se Otávio é representado pelo número 2, ele aparece nas posições a12, a21, a22, a23 e a32 Vamos encontrar o total de temakis consumidos no final de semana, somando as matrizes S e D: 3 2 0 S+D= 1 1 2 0 3 2

2 3 0 5 5 0 0 2 1 = 1 3 3 1 0 2 1 3 4

Como para Rodrigo i = 1 e para Otávio i = 2, então, no final de semana todo, Rodrigo pagou para Otávio 5 temakis (a12 = 5) e Otávio pagou para Rodrigo 1 temaki (a21 = 1). O saldo foi de 5 – 1 = 4. Portanto, Otávio deve 4 temakis para Rodrigo. Resposta: E

2. (Uerj 2016) Considere uma matriz A com 3 linhas e 1 coluna, na qual foram

escritos os valores 1, 2 e 13, nesta ordem, de cima para baixo. Considere, também, uma matriz B, com 1 linha e 3 colunas, na qual foram escritos os valores 1, 2 e 13, nesta ordem, da esquerda para a direita. Calcule o determinante da matriz obtida pelo produto de A x B.

RESOLUÇÃO Vamos começar escrevendo as duas matrizes segundo as regras do enunciado. A matriz A é uma matriz coluna e a matriz B é uma matriz linha. São elas: 1 A = 2 e B = ( 1 2 13 ) 13

128 GE MATEMÁTICA 2017

A multiplicação é possível, já que a quantidade de colunas de A é igual à quantidade de linhas de B; a matriz C, produto de A por B, será quadrada de ordem 3. A3x1 x B1x3 = C3x3 Procedendo ao produto: 1 1 2 13 2 . (1 2 13) = 2 4 26 13 13 26 169 Veja que a matriz tem duas linhas proporcionais à primeira. Uma das propriedades das matrizes quadradas afirma que basta termos duas linhas ou colunas proporcionais para que o determinante da matriz seja nulo. Portanto, det (A x B) = 0 Resposta: O determinante da multiplicação A x B vale zero

3. (Enem 2012) Um aluno registrou as notas bimestrais de algumas de suas

disciplinas numa tabela. Ele observou que as entradas numéricas da tabela formavam uma matriz 4x4, e que poderia calcular as médias anuais dessas disciplinas usando produto de matrizes. Todas as provas possuíam o mesmo peso, e a tabela que ele conseguiu é mostrada a seguir. 1º bimestre

2º bimestre

3º bimestre

4º bimestre

Matemática

5,9

6,2

4,5

5,5

Português

6,6

7,1

6,5

8,4

Geografia

8,6

6,8

7,8

9,0

História

6,2

5,6

5,9

7,7

Para obter essas médias, ele multiplicou a matriz obtida a partir da tabela por a) 1 1 1 1 b) 1 1 1 1 c) 1 2 2 2 2 4 4 4 4 1 1 1 d) 1 e) 1 2 4 1 2

1 4

1 2

1 4

1 2

1 4

RESOLUÇÃO Primeiro, entenda bem: a tabela apresentada no enunciado traz as notas das provas de cada bimestre. E a partir delas o aluno quer encontrar a média anual obtida em cada disciplina. Transformando a tabela com as notas em matriz, temos 5,9 6,2 4,5 5,5 6,6 7,1 6,5 8,4 8,6 6,8 7,8 9,0 6,2 5,6 5,9 7,7 O enunciado diz que a matriz das notas (vamos chamar de matriz A) foi multiplicada por outra (matriz B) para dar como resultado a matriz das médias (C). Sabemos que A é uma matriz com 4 linhas e 4 colunas (m = 4 e n = 4). E que a matriz C terá uma única coluna e 4 linhas, cada uma delas correspondendo a uma disciplina. Portanto, para C, m = 4 e n = 1. A multiplicação é dada pela expressão A4 x 4 x Bm x n = C4 x 1


RESUMO

Pela regra da multiplicação de duas matrizes, o número de linhas de uma (matriz A) deve ser igual ao número de colunas de outra (matriz B). Então já sabemos que para B, m = 4 e n = 1

Lorem ipsondolor Matrizes

Se todas as provas têm o mesmo peso, então o cálculo da média anual é a soma das notas dividida pelo número de notas (4). Dividir por 4 é o mesmo que multiplicar por 1 . 4

MATRIZES São GIAMCORE MAGNA comoaccum tabelas, am, nas vullam, quais core cadafeum elemento auguerit, ocupa si blam,posição uma quat. Lor determinada: sequat lorerci linha temiaccum e coluna il ulput j. O número nummy nit de linhas dead colunas (n) hent define ordem da matriz. Por nullam(m) aditeea tetumsan lorainit adionsequip exeros exemplo, A23 ézzrit umaamcorer matriz com 2 linhas e 3autpatin colunas.eugue Matrizes do dolor sum sustrud dui et velenim vulluptate consectem elnúmero ulputatum quadradas são aquelas quezzrit têm wismod o mesmo de incing linhas et lutdiamcom molumsandip. e de colunas. Qualquer matriz tem duas diagonais:

Então, na multiplicação das matrizes A e B, a matriz B correta é composta de uma única coluna com 4 linhas, todas com o valor 1 . 4 Resposta: E

4. (Uema 2015) Uma matriz A(m x n) é uma tabela retangular formada por

m x n números reais (aij), dispostos em m linhas e n colunas. O produto de duas matrizes A = (aij)m x n e B = (bij)n x p é uma matriz C = (cij)m x p em que o elemento cij é obtido da multiplicação ordenada dos elementos da linha i, da matriz A, pelos elementos da coluna j, da matriz B, e somando os elementos resultantes das multiplicações. A soma de matrizes é comutativa, ou seja, A + B = B + A. Faça a multiplicação das matrizes A e B, e verifique se esse produto é comutativo, ou seja: A x B = B x A. 1 2 3 0 1 –2 A = 0 1 2 e B = 1 –2 3 0 0 1 0 1 0

RESOLUÇÃO Como as matrizes são quadradas e de mesma ordem, 3, é possível fazer as multiplicações A x B e B x A. A questão pede que verifiquemos se os resultados das duas multiplicações são os mesmos – o que significaria que a multiplicação delas é comutativa (tanto faz a ordem em que a multiplicação for feita). Vamos aos produtos: 1 2 3 0 1 –2 A . B = 0 1 2 . 1 –2 3 0 0 1 0 1 0 1 . 0 + 2 . 1 + 3 . 0 1 . 1 + 2 . (–2) + 3 . 1 1 . (–2) + 2 . 3 + 3 . 0 A . B = 0 . 0 + 1 . 1 + 2 . 0 0 . 1 + 1 . (–2) + 2 . 1 0 . (–2) + 1 . 3 + 2 . 0 0 . 0 + 0 . 1 + 1 . 0 0 . 1 + 0 . (–2) + 1 . 1 0 . (–2) + 0 . 3 + 1 . 0 2 0 4 A. B = 1 0 3 0 1 0 0 1 –2 1 2 B . A = 1 –2 3 . 0 1 0 1 0 0 0 0 . 1 + 1 . 0 + (–2) . 0 B . A = 1 . 1 + (–2) . 0 + 3 . 0 0.1+1.0+0.0 0 1 0 B. A = 1 0 2 0 1 2

3 2 1 0 . 2 + 1 . 1 + (–2) . 0 1 . 2 + (–2) . 1 + 3 . 0 0.2+1.1+0.0

0 . 3 + 1 . 2 + (–2) . 1 1 . 3 + (–2) . 2 + 3 . 1 0.3+1.2+0.1

diagonal diagonal EAFACIDUNT sustrud magna feugiam veniam principal DOLOBOR secundária zzrilit luptatem iriusto consequi eraesto eugait luptat do ese = Avenis A21amconsed mincillandre commodi onullan ver 11 tatAdolut A22 12 sustrud A modigniam ipsuscillam, cor iliquat. Num volobor eraestionum ing eniatummy nulputem vent amet iusto odignim MATRIZ IDENTIDADE quisis adiam É aquela aliquat na vel qualesequip a diagonal principal contém apenas o valor 1, e os demais elementos são iguas a 0: IS NULLA FEUGAIT aut venim nostrud min ut wissecte magni1 0 0 bh et nim incillandre do commy non hendip eu feugait lobore I= 0 1 0 magnim am, quisciduis nulluptatum venit in velendi gnissenit, 0 0 1 sequat. Equat. Ut iliscidunt la commy nostion hendiam commod ditmultiplicação velendrero diat, vel ing ex elit at pratin nonullan A de uma matriz qualquer poresectet sua matriz idenheniamdá doloreet amcore do eu faciloriginal. utpat. Osto odiamet, velent tidade como produto a matriz pratet nosto consequisl ullandrem quat am dolorem veliquatue min velesequam OPERAÇÕES COMnonse MATRIZES facipisim Na soma zzriure. ou subtração de duas matrizes, fazemos a operação entre elementos corresponRCILIQUATET dentes. Por exemplo VULLAN naute soma commy de A22 nullaorem + B22, fazemos: ip ero consectet a11 + b11, lum a12 +vel b12ulput , a21 +veliquis b21 etc.exerosting Na multiplicação endrerosde autuma ilis at. matriz Lestopor dolorperci um número, tio dolutpat multiplicamos ullaore oriurerit número, in henim separadamente, iusci bla at.pelos Gait elementos. se uma matriz for multiplicada por 2, atummolorePor tieexemplo, te er ipisim dit wisl ipsum dunt velis aliquat. fazemos a11 . 2, a12 . 2, a21 . 2, a22 . 2 etc. A multiplicação entre NONUMMO duas matrizes LOBORERO só é possível etumsandrem quando o número dolorperatem de linhas do de uma duis acidunté vel matriz igual ullamet ao número nostodecoreet colunas alisdealiquipit outra. Os vent elementos adignisim da ipsuscipit matriz P produto in Del utde lutat A1 .aute A2 são mincill obtidos andipsustis pela multiplicação do exeraestrud dos eum nissedde elementos essequat cada linha nonulput de A1 pelos volore elementos tem aditcorrespondener ip elenit ing et irilit tes de cada iureetcoluna laoremdeveraess A2. Depois, equisi. osEcte resultados vulla commy são somados. nullam, sis nulluptat, sum venibh elesto conum nonulla facilit nit lorem delesto ea feui blandre DETERMINANTE É um número eui tetreal lam(k) que pode ser associado a determinada matriz quadrada. O determinante de matriz 2 x 2 ISencontrado é NULLA FEUGAIT pela aut soma venim da nostrud multiplicação min ut wissecte dos valores magnida bh et nim principal diagonal incillandrecom do commy o produto nondos hendip valores eu feugait da diagonal lobore magnim am,Para secundária. quisciduis matrizes nulluptatum 3 x 3, seguimos venit inavelendi regra de gnissenit, Sarrus: sequat. Equat. copiamos as duas Ut iliscidunt primeiraslacolunas commyànostion direita hendiam da matrizcommod original; dit velendrero diat, multiplicamos os valores vel ingda exdiagonal elit at pratin principal esectet e, também, nonullan heniam doloreet amcoresomamos do eu facil utpat. das diagonais paralelas; esses produtos. Fazemos o mesmo com os valores da diagonal secundária e suas paralelas. RCILIQUATET Por fim, subtraímos VULLAN os valores. ute commy nullaorem ip ero consectet lum vel ulput veliquis exerosting endreros aut ilis at. Lesto dolorperci tio dolutpat PROPRIEDADES DOSullaore DETERMINANTES riurerit in henim Se todos iusci osbla elementos at. Gait atummolore de uma linhatie oute uma er ipisim colunaditdewisl uma ipsum matriz dunt A forem velis aliquat. iguais a zero, então o determinante da matriz é zero (det A = 0) . Se duas NONUMMO linhas ou duas LOBORERO colunas tiverem etumsandrem elementos dolorperatem iguais ou propordo duis acidunt vel cionais, o determinante ullamet nostodacoreet matrizalis é igual aliquipit a zero vent também. adignisim Se ipsuscipit inde Del ut lutatduas aute linhas mincillou andipsustis do exeraestrud trocarmos posição duas colunas, o sinal do eum nissed essequat nonulput volore tem adit er ip elenit. determinante se inverte.

Resposta: O resultado de A x B é diferente do resultado de B x A. Então, essa multiplicação não é comutativa. GE MATEMÁTICA 2017

129


RAIO-X DECIFRE OS ENUNCIADOS E VEJA AS CARACTERÍSTICAS TÍPICAS DAS QUESTÕES QUE CAEM NAS PROVAS

PUC-RIO 2015 A medida da área, em cm2, de um quadrado [1] que pode ser inscrito [2] em um círculo de raio igual a 5 cm é: c) 25 d) 50 2 e) 50 a) 20 b) 25 2

DICAS PARA A RESOLUÇÃO

Desenhe a figura que representa a situação descrita no enunciado: um quadrado dentro de um círculo. Assim: r

1 Desenhar a situação proposta no enunciado é o melhor caminho para começar a resolver problemas de geometria. Incluindo na figura as medidas conhecidas e as variáveis, fica mais fácil identificar a relação entre todos os valores – por exemplo, área e lado do quadrado com o raio do círculo. 2 Não se confunda: um quadrado inscrito em um círculo está dentro do círculo. O círculo, por sua vez, circunscreve o quadrado. As relações entre as dimensões de duas figuras inscritas ou circunscritas são completamente diferentes em cada uma das situações.

r Só pelo desenho você já identifica o tamanho da diagonal do quadrado (2r) e que essa diagonal forma com dois lados do quadrado um triângulo retângulo. A alternativa correta é a E. Veja a resolução na pág. 132.

ENEM 2014 Na alimentação de gado de corte, o processo de cortar a forragem, colocá-la no solo, compactá-la e protegê-la com uma vedação denomina-se silagem. Os silos mais comuns são os horizontais, cuja forma é a de um prisma [1] reto trapezoidal, conforme mostrado na figura.

h

B c b

Legenda [2]

b - largura do fundo B - largura do topo C - comprimento do silo h - altura do silo

Considere um silo de 2 m de altura, 6 m de largura de topo e 20 m de comprimento. Para cada metro de altura do silo, a largura do topo tem 0,5 m a mais do que a largura do fundo. Após a silagem, 1 tonelada de forragem ocupa 2 m3 desse tipo de silo. EMBRAPA, Gado de corte. Disponível em: www.cnpgc.embrapa.br Acesso em: 1 ago. 2012 (adaptado)

Após a silagem, a quantidade [3] máxima de forragem que cabe no silo, em toneladas, é a) 110 b) 125 c) 130 d) 220 e) 260

130 GE MATEMÁTICA 2017

1 Prisma é um dos sólidos geométricos que mais caem nas questões de vestibular. Para questões como esta, você deve saber de cor as fórmulas gerais de área e volume.

DICAS PARA A RESOLUÇÃO

Normalmente, quando trabalhamos com prismas, costumamos desenhá-los na vertical, como a figura abaixo. Se para você ficar mais confortável, nada o impede de reproduzir a figura dessa maneira e fazer os cálculos sobre ela. Isso não faz a menor diferença para esta questão.

2 Não se confunda com os nomes que o enunciado atribui a cada dimensão da figura. Dependendo do caso, a altura pode ser considerada comprimento, e vice-versa. 3 Questão de interpretação básica de texto: quando o enunciado fala em quantidade, refere-se a volume.

b h

c

B

A resposta correta é a alternativa A. A resolução completa está na pág. 133.


ENEM 2013 Um carpinteiro fabrica portas retangulares maciças, feitas de um mesmo material. Por ter recebido de seus clientes pedidos de portas mais altas, aumentou sua altura em 1/8, preservando suas espessuras [1]. A fim de manter o custo com o material de cada porta, precisou reduzir a largura [2]. A razão[3] entre a largura da nova porta e a largura da porta anterior [4] é a) 1/8 b) 7/8 c) 8/7 d) 8/9 e) 9/8

DICAS PARA A RESOLUÇÃO

A questão é fácil, mas a resposta não é automática. Você tem de fazer contas. Se não prestar atenção, é provável que assinale a alternativa E. Mas a correta é a D. Você deve aplicar conhecimentos básicos de geometria (área de retângulo, no caso) e tomar cuidado ao realizar operações com frações. Veja a resolução completa na pág. 132.

1 Fique atento a tudo o que o enunciado informa. Neste caso, se a espessura foi mantida nas duas portas, você deve trabalhar com uma figura de duas dimensões – retângulos, em que só se medem a altura e a largura. 2 Os enunciados de matemática podem, muitas vezes, parecer um labirinto de palavras. Leia calmamente o texto e desenhe a situação descrita. Se a altura aumentou e a área tem de ser mantida, então a largura deve diminuir, é claro. 3 As questões do Enem não costumam exigir que você conheça fórmulas, apenas as mais básicas, como a área de um retângulo. Mas, como em qualquer vestibular, você tem de dominar conceitos e o linguajar para interpretar corretamente os enunciados. Razão, você sabe, é o mesmo que proporção. E toda proporção envolve uma divisão. 4 Matemática é lógica. Cuidado ao definir os nomes para as incógnitas e variáveis que o problema traz. Neste caso, por exemplo, você pode chamar as portas normais de N e as mais altas de A. A área de uma será AN e a outra será AA.

UEMA 2015 Um fabricante de brinquedos utiliza material reciclado: garrafas, latinhas e outros. Um dos brinquedos despertou a atenção de um estudante de geometria, por ser confeccionado da seguinte forma [1]: amarra-se um barbante em um bico de garrafa pet cortada e, na extremidade, cola-se uma bola de plástico que, ao girar em torno do bico, forma uma circunferência. O estudante representou-a no sistema por coordenadas cartesianas, conforme a figura a seguir: y

1 Muitas vezes, nas provas de vestibular e do Enem, os enunciados trazem uma longa história e descrevem situações difíceis de serem visualizadas. Não se afobe na leitura. Vá até o fim e, depois, volte ao início, grifando o que realmente importa. Você verá que não importa a descrição do brinquedo, nem o envolvimento do estudante. A situação descrita é claramente mostrada na figura. 2 As unidades de medidas são fundamentais em qualquer problema de matemática. Seja qual for a unidade adotada (metro, centímetro, quilômetro ou u.c., como nesta questão), comprimentos e larguras são sempre dados em unidades simples. Já áreas têm a unidade elevada ao quadrado (u.c.2). E volume, ao cubo (u.c.3).

C (3,4)

x

Considerando o tamanho do barbante igual a 6 unidades de comprimento (u.c.) e o bico centrado no ponto (3, 4), a equação que representa a circunferência é igual a a) x2 + y2 – 6x – 8y – 11 = 0 [3] b) x2 + y2 + 6x + 8y – 11 = 0 c) x2 + y2 + 6x + 8y + 11 = 0 d) x2 + y2 – 6x – 8y + 11 = 0 e) x2 + y2 – 8x – 6y – 11 = 0 [2]

3 Veja que as alternativas são todas muito semelhantes. Os examinadores apenas trocaram sinais na equação, em cada uma delas. Então, confira seus cálculos e atenção na hora de assinalar a alternativa correta.

DICAS PARA A RESOLUÇÃO

Não se assuste com o enunciado longo. A pergunta é direta – qual a equação de uma circunferência – e a questão é fácil. Basta você ter de cabeça a forma geral da equação da circunferência e substituir valores. A alternativa correta é a A. Veja a resolução completa na pág. 132. GE MATEMÁTICA 2017

131


SIMULADO QUESTÕES SELECIONADAS ENTRE OS MAIORES VESTIBULARES DO PAÍS COM RESPOSTAS COMENTADAS CAPÍTULO 1

1. (Enem 2013)

Uma ponte precisa ser dimensionada de forma que possa ter três pontos de sustentação. Sabe-se que a carga máxima suportada pela ponte será de 12 t. O ponto de sustentação central receberá 60% da carga da ponte, e o restante da carga será distribuído igualmente entre os outros dois pontos de sustentação. No caso de carga máxima, as cargas recebidas pelos três pontos de sustentação serão, respectivamente, a) 1,8 t; 8,4 t; 1,8 t b) 3,0 t; 6,0 t; 3,0 t c) 2,4 t; 7,2 t; 2,4 t d) 3,6 t; 4,8 t; 3,6 t e) 4,2 t; 3,6 t; 4,2 t

2. (Fuvest 2014)

5. (UEG 2015)

O celular de Fabiano está com 50% de carga na bateria. Quando está completamente carregado, ele demora exatamente 20 horas para descarregar toda a bateria em modo stand by, supondo-se que essa bateria se descarregue de forma linear. Ao utilizar o aparelho para brincar com um aplicativo, a bateria passará a consumir 1% da carga a cada 3 minutos. Quantos minutos Fabiano poderá brincar, antes que a bateria se descarregue completamente? a) Três horas b) Duas horas e meia c) Duas horas d) Uma hora e meia

CAPÍTULO 2

6. (CFTMG 2015) Na figura a seguir, as retas r, s, t e w são paralelas e a, b e

c representam medidas dos segmentos tais que a + b + c = 100.

Um apostador ganhou um prêmio de R$ 1 000 000,00 na loteria e decidiu investir parte do valor em caderneta de poupança, que rende 6% ao ano, e o restante em um fundo de investimentos, que rende 7,5% ao ano. Apesar do rendimento mais baixo, a caderneta de poupança oferece algumas vantagens e ele precisa decidir como irá dividir o seu dinheiro entre as duas aplicações. Para garantir, após um ano, um rendimento total de pelo menos R$ 72 000,00 a parte da quantia a ser aplicada na poupança deve ser de, no máximo, a) R$ 200 000,00 b) R$ 175 000,00 c) R$ 150 000,00 d) R$ 125 000,00 e) R$ 100 000,00

r s

t

18

a b

24 33

c

w

3. (Enem 2014)

Para se construir um contrapiso, é comum, na constituição do concreto, se utilizar cimento, areia e brita, na seguinte proporção: 1 parte de cimento, 4 partes de areia e 2 partes de brita. Para construir o contrapiso de uma garagem, uma construtora encomendou um caminhão betoneira com 14 m3 de concreto. Qual é o volume de cimento, em m3, na carga de concreto trazido pela betoneira? a) 1,75 b) 2,00 c) 2,33 d) 4,00 e) 8,00

Conforme esses dados, os valores de a, b e c são, respectivamente, iguais a a) 24, 32 e 44 b) 24, 36 e 40 c) 26, 30 e 44 d) 26, 34 e 40

4. (Enem 2013)

a) 20

Um carpinteiro fabrica portas retangulares maciças, feitas de um mesmo material. Por ter recebido de seus clientes pedidos de portas mais altas, aumentou sua altura em 1/8, preservando suas espessuras. A fim de manter o custo com o material de cada porta, precisou reduzir a largura. A razão entre a largura da nova porta e a largura da porta anterior é a) 1/8 b) 7/8 c) 8/7 d) 8/9 e) 9/8

132 GE MATEMÁTICA 2017

7. (PUC-RIO 2015)

A medida da área, em cm², de um quadrado que pode ser inscrito em um círculo de raio igual a 5 cm é?

8.

b) 25 2

c) 25

d) 50 2

e) 50

(Uema 2015) Um fabricante de brinquedos utiliza material reciclado: garrafas, latinhas e outros. Um dos brinquedos despertou a atenção de um estudante de geometria, por ser confeccionado da seguinte forma: amarra-se um barbante em um bico de garrafa pet cortada e, na extremidade, cola-se uma bola de plástico que, ao


girar em torno do bico, forma uma circunferência. O estudante representou-a no sistema por coordenadas cartesianas, conforme a figura a seguir:

y

11. (Enem 2014)

Na alimentação de gado de corte, o processo de cortar a forragem, colocá-la no solo, compactá-la e protegê-la com uma vedação denomina-se silagem. Os silos mais comuns são os horizontais, cuja forma é a de um prisma reto trapezoidal, conforme mostrado na figura.

C (3,4) •

h

B

Legenda b - largura do fundo B - largura do topo C - comprimento do silo h - altura do silo

c

x b Considerando o tamanho do barbante igual a 6 unidades de comprimento (u.c.) e o bico centrado no ponto (3, 4), a equação que representa a circunferência é igual a a) x² + y² – 6x – 8y – 11 = 0 b) x² + y² + 6x + 8y – 11 = 0 c) x² + y² + 6x + 8y + 11 = 0 d) x² + y² – 6x – 8y + 11 = 0 e) x² + y² – 8x – 6y – 11 = 0

9. (Unesp 2014)

A figura mostra um plano cartesiano no qual foi traçada uma elipse com eixos paralelos aos eixos coordenados.

y 6

Considere um silo de 2 m de altura, 6 m de largura de topo e 20 m de comprimento. Para cada metro de altura do silo, a largura do topo tem 0,5 m a mais do que a largura do fundo. Após a silagem, 1 tonelada de forragem ocupa 2 m3 desse tipo de silo. EMBRAPA, Gado de corte. Disponível em: www.cnpgc.embrapa.br Acesso em: 1 ago. 2012 (adaptado)

Após a silagem, a quantidade máxima de forragem que cabe no silo, em toneladas é a) 110 b) 125 c) 130 d) 220 e) 260

12. (ESPM 2014)

5 4

A figura abaixo mostra a trajetória de um móvel a partir de um ponto A com BC = CD, DE = EF, FG = GH, HI = IJ, e assim por diante.

3 2

B

1

16 m

-1 0 -1

1

2

3

4

Considere o triângulo ABC no plano cartesiano com vértices A = (0, 0), B = (3, 4) e C = (8, 0). O retângulo MNPQ tem os vértices M e N sobre o eixo das abscissas, o vértice Q sobre o lado AB e o vértice P sobre o lado BC. Dentre todos os retângulos construídos desse modo, o que tem área máxima é aquele em que o ponto P é 16 a) S 4, 5 X

b) S 4 , 3 X 17

12 c) S 5, 5 X

d) S 11 , 2 X 2 e) S 6, 85 X

D

F

H

J

x

Valendo-se das informações contidas nesta representação, determine a equação reduzida da elipse.

10. (Fuvest 2014)

12 m

A

C

E

G

I

P

Considerando infinita a quantidade desses segmentos, a distância horizontal AP alcançada por esse móvel será de a) 65 m b) 72 m c) 80 m d) 96 m e) 100 m

13. (Fuvest 2015)

A equação x² + 2x + y² + my = n, em que m e n são constantes, representa uma circunferência no plano cartesiano. Sabe-se que a reta y = – x + 1 contém o centro da circunferência e a intersecta no ponto (–3, 4). Os valores de m e n são, respectivamente, a) –4 e 3 b) 4 e 5 c) –4 e 2 d) –2 e 4 e) 2 e 3 GE MATEMÁTICA 2017

133


SIMULADO CAPÍTULO 3

14. (Unifesp 2015)

19. (PUC-RIO 2015, adaptada)

Considere a hipérbole de equação y = 1/x mostrada na figura abaixo:

A concentração C, em partes por milhão (ppm), de certo medicamento na corrente sanguínea após t horas da sua ingestão é dada pela função polinomial C(t) = – 0,05t² + 2t + 25. Nessa função, considera-se t = 0 o instante em que o paciente ingere a primeira dose do medicamento. Álvaro é um paciente que está sendo tratado com esse medicamento e tomou a primeira dose às 11 horas da manhã de uma segunda-feira. a) A que horas a concentração do medicamento na corrente sanguínea de Álvaro atingirá 40 ppm pela primeira vez? b) Se o médico deseja prescrever a segunda dose quando a concentração do medicamento na corrente sanguínea de Álvaro atingir seu máximo valor, para que dia da semana e horário ele deverá prescrever a segunda dose?

15. (PUC–PR 2015)

y

(–2,2) x

Seja uma função afim f(x) cuja forma é f(x) = ax + b, com a e b números reais. Se f(–3) = 3 e f(3) = –1, os valores de a e b são respectivamente: a) 2 e 9 b) 1 e –4 c) 1/3 e 3/5 d) 2 e –7 e) –2/3 e 1

Determine os pontos de interseção entre a hipérbole e a reta de equação y – 2 = x + 2.

16. (Uece 2014)

a) 4

Em uma corrida de táxi, é cobrado um valor inicial fixo, chamado de bandeirada, mais uma quantia proporcional aos quilômetros percorridos. Se por uma corrida de 8 km pagam-se R$ 28,50 e por uma corrida de 5 km pagam-se R$ 19,50, então o valor da bandeirada é a) R$ 7,50 b) R$ 6,50 c) R$ 5,50 d) R$ 4,50

17. (CFTMG 2015)

20. (UPE 2015)

Se escrevermos a função quadrática f(x) = 2x² – x + 3 na forma f(x) = a . (x – m)² + n, o valor de a + m + n é igual a 19

b) 27 4

41

c) 8

33

d) 8

e) 25 8

21. (PUC-RIO 2015)

A figura abaixo mostra uma reta e uma parábola de eixo vertical.

Um economista observa os lucros das empresas A e B do primeiro ao quarto mês de atividades e chega à conclusão que, para este período, as equações que relacionam o lucro, em reais, e o tempo, em meses, são LA(t) = 3t – 1 e LB (t) = 2t + 9. Considerando-se que essas equações também são válidas para o período do quinto ao vigésimo quarto mês de atividades, o mês em que as empresas terão o mesmo lucro será o a) vigésimo b) décimo sétimo c) décimo terceiro d) décimo

18. (Uece 2015)

Se a função real de variável real definida por f(x) = ax² + bx + c é tal que f(1) = 2, f(2) = 5 e f(3) = 4, então o valor de f(4) é a) 2 b) –1 c) 1 d) –2

134 GE MATEMÁTICA 2017

y

(0,8)

(0,2) x

(-2,0) (2,0)

(4,0)

a) Sabendo que a reta corta os eixos nos pontos (–2, 0) e (0, 2), encontre a equação da reta. b) Sabendo que a parábola corta os eixos nos pontos (0, 8), (2, 0) e (4, 0), encontre a equação da parábola. c) Encontre os pontos de interseção entre a reta e a parábola.


22. (PUC-RIO 2015)

27. (Fuvest 2014)

23. (UFRGS 2015)

a) ela não possui raízes reais b) sua única raiz real é –3 c) duas de suas raízes reais são 3 e –3 d) suas únicas raízes reais são –3, 0 e 1 e) ela possui cinco raízes reais distintas

2 Sobre a equação (x + 3). 2x – 9 . log |x² + x – 1| = 0, é correto afirmar que

Seja f(x) = 4 – 6 . 2 + 8. a) Calcule f(0). b) Encontre todos os valores reais de x para os quais f(x) = 168. c) Encontre todos os valores reais de x para os quais f(x) < 0. x

x

Dadas as funções f e g, definidas respectivamente por f(x) = x² – 4x + 3 e g(x) = –x² –4x – 3 e representadas no mesmo sistema de coordenadas cartesianas, a distância entre seus vértices é a) 4

b) 5

c)

5

d)

10

e) 2 5

24. (Fuvest 2013)

interbits ®

log10N

O número N de átomos de um isótopo radioativo existente em uma amostra diminui com o tempo t, de acordo com a expressão N(t) = N0 e–λt sendo N0 o número de átomos deste isótopo em t = 0 e λ a constante de decaimento. Abaixo, está apresentado o gráfico do log10N em função de t, obtido em um estudo experimental do radiofármaco Tecnécio 99 metaestável (99mTc), muito utilizado em diagnósticos do coração.

5,8 5,7

NOTE E ADOTE: A meia-vida (T1/2 ) de um isótopo radioativo é o intervalo de tempo em que o número de átomos desse isótopo existente em uma amostra cai para a metade; log 2 = 0,3 e log 5 = 0,7.

5,6 5,5 5,4

0

4

2

6

8

10

t (horas)

A partir do gráfico, determine a) o valor de log10N0; b) o número N0 de átomos radioativos de 99mTc ; c) a meia-vida (T1/2) do 99mTc.

25. (UPE 2015)

Os biólogos observaram que, em condições ideais, o número de bactérias Q(t) em uma cultura cresce exponencialmente com o tempo t, de acordo com a lei Q(t) = Q0 . ekT sendo k > 0 uma constante que depende da natureza das bactérias; o número é irracional e vale aproximadamente 2,718 e Q0 é a quantidade inicial de bactérias. Se uma cultura tem inicialmente 6 000 bactérias e, 20 minutos depois, aumentou para 12 000, quantas bactérias estarão presentes depois de 1 hora? a) 1,8 x 104

b) 2,4 x 104

c) 3,0 x 104

d) 3,6 x 104

e) 4,8 x 104

A função f definida por f(x) = 4–x – 2 intercepta o eixo das abscissas em b) –1

onde M é a magnitude do terremoto, E é a energia liberada (em joules) e E0 = 104,5 joules é a energia liberada por um pequeno terremoto usado como referência. Qual foi a ordem de grandeza da energia liberada pelo terremoto do Japão de 11 de março de 2011, que atingiu magnitude 9 na escala Richter? a) 1014 joules b) 1016 joules c) 1017 joules d) 1018 joules e) 1019 joules

29. (UFSM 2015)

Quando um elemento radioativo, como o Césio 137, entra em contato com o meio ambiente, pode afetar o solo, os rios, as plantas e as pessoas. A radiação não torna o solo infértil, porém tudo que nele crescer estará contaminado. A expressão Q(t) = Q0e–0,023t representa a quantidade, em gramas, de átomos radioativos de Césio 137 presentes no instante t, em dias, onde Q0 é a quantidade inicial. O tempo, em dias, para que a quantidade de Césio 137 seja a metade da quantidade inicial é igual a (Use ln 2 = 0,69) a) 60

b) 30

c) 15

d) 5

c) –1/2

d) 0

e) 1/2

e) 3

30. (Espcex 2012)

Na figura abaixo, dois vértices do trapézio sombreado estão no eixo x e os outros dois vértices estão sobre o gráfico da função real f(x) = logkx com k > 0 e k ≠ 1. Sabe-se que o trapézio sombreado tem 30 unidades de área; assim, o valor de k + p – q é y f(x)=logk x 2 1

26. (UFRGS 2014) a) –2

Terremotos são eventos naturais que não têm relação com eventos climáticos extremos, mas podem ter consequências ambientais devastadoras, especialmente quando seu epicentro ocorre no mar, provocando tsunamis. Uma das expressões para se calcular a violência de um terremoto na escala Richter é

2 E M = 3 . log 10 S E X 0

CAPÍTULO 4

6,0 5,9

28. (UPE 2012)

p

q

x

a) –20 b) –15 c) 10 d) 15 e) 20

gráfico fora de escala GE MATEMÁTICA 2017

135


SIMULADO

36. (PUC-PR 2015)

CAPÍTULO 5

Um consumidor, ao adquirir um automóvel, assumiu um empréstimo no valor total de R$ 42 000,00 (já somados juros e encargos). Esse valor foi pago em 20 parcelas, formando uma progressão aritmética decrescente. Dado que na segunda prestação foi pago o valor de R$ 3 800,00, a razão desta progressão aritmética é:

31. (PUC-RIO 2014)

Assinale a alternativa correta a) sen (1 000°) < 0 b) sen (1 000°) > 0 c) sen (1 000°) = cos (1 000°) d) sen (1 000°) = –sen (1 000°) e) sen (1 000°) = –cos (1 000°)

a) –300

b) –200

c) –150

d) –100

e) –350

37. (PUC-RIO 2015)

A soma dos números inteiros compreendidos entre 100 e 400, que possuem o algarismo das unidades igual a 4, é:

32.

(Uece 2015) Sejam x, y e z as medidas dos lados do triângulo XYZ e R a medida do raio da circunferência circunscrita ao triângulo. Se o produto dos senos dos ângulos internos do triângulo é k . x . y . z , então o valor de k é R3

a) 0,500

b) 0,250

c) 0,125

d) 1,000

33. (Espcex / Aman 2015)

a) 1 200

b) 2 560

b) –1

c) 0

d) 1

d) 6 420

e) 7 470

38. (Enem 2013)

As projeções para a produção de arroz no período de 2012–2021, em uma determinada região produtora, apontam para uma perspectiva de crescimento constante da produção anual. O quadro apresenta a quantidade de arroz, em toneladas, que será produzida nos primeiros anos desse período, de acordo com essa projeção. Ano 2012

O valor de (cos 165° + sen 155° + cos 145° – sen 25° + cos 35° + cos 15°) é a) 2

c) 4 980

e) 1/2

34. (CFTMG 2015) O esboço do gráfico da função f(x) = a + b.cos(x) é

Projeção da produção (t) 50,25

2013

51,50

2014

52,75

2015

54,00

mostrado na figura seguinte.

A quantidade total de arroz, em toneladas, que deverá ser produzida no período de 2012 a 2021 será de interbits ®

y 6 5

a) 497,25 b) 500,85 c) 502,87 d) 558,75 e) 563,25

4 3 2 1 -π/2

0 -1

π/2

π

3π/2

39. (Fuvest 2011)

x

Seja x > 0 tal que a sequência a1 = log2x, a2 = log4(4x), a3 = log8(8x) forme, nessa ordem, uma progressão aritmética. Então, a1 + a2 + a3 é igual a

-2

a) 13/2

b) 15/2

Nessa situação, o valor de a . b é a) 2

b) 3

c) 17/2

d) 19/2

e) 21/2

CAPÍTULO 7 c) 5

d) 6

40. (Enem 2013)

(PUC-RIO 2015) Os números a1= 5x – 5, a2 = x + 14 e a3 = 6x – 3 estão em PA. A soma dos 3 números é igual a:

Numa escola com 1 200 alunos foi realizada uma pesquisa sobre o conhecimento desses em duas línguas estrangeiras, inglês e espanhol. Nessa pesquisa constatou-se que 600 alunos falam inglês, 500 falam espanhol e 300 não falam qualquer um desses idiomas. Escolhendo-se um aluno dessa escola ao acaso e sabendo-se que ele não fala inglês, qual a probabilidade de que esse aluno fale espanhol?

a) 48

a) 1/2

CAPÍTULO 6

35.

b) 54

136 GE MATEMÁTICA 2017

c) 72

d) 125

e) 130

b) 5/8

c) 1/4

d) 5/6

e) 5/14


41. (Enem 2014)

CAPÍTULO 8

Ao final de uma competição de ciências em uma escola, restaram apenas três candidatos. De acordo com as regras, o vencedor será o candidato que obtiver a maior média ponderada entre as notas das provas finais nas disciplinas química e física, considerando, respectivamente, os pesos 4 e 6 para elas. As notas são sempre números inteiros. Por questões médicas, o candidato II ainda não fez a prova final de química. No dia em que sua avaliação for aplicada, as notas dos outros dois candidatos, em ambas as disciplinas, já terão sido divulgadas. O quadro apresenta as notas obtidas pelos finalistas nas provas finais Candidato

Química

Física

I

20

23

II

X

25

III

21

18

44. (Uerj 2016)

Na região conhecida como Triângulo das Bermudas, localizada no Oceano Atlântico, é possível formar um triângulo com um vértice sobre a cidade porto-riquenha de San Juan, outro sobre a cidade estadunidense de Miami e o terceiro sobre as Ilhas Bermudas. A figura abaixo mostra um sistema de coordenadas cartesianas ortogonais, com os vértices do triângulo devidamente representados. A escala utilizada é 1: 17.000.000, e cada unidade nos eixos cartesianos equivale ao comprimento de 1 cm.

A menor nota que o candidato II deverá obter na prova final de química para vencer a competição é a) 18

b) 19

c) 22

d) 25

e) 26

42. (Enem 2013)

Um banco solicitou aos seus clientes a criação de uma senha pessoal de seis dígitos, formada somente por algarismos de 0 a 9, para acesso à conta corrente pela internet. Entretanto, um especialista em sistemas de segurança eletrônica recomendou à direção do banco recadastrar seus usuários, solicitando, para cada um deles, a criação de uma nova senha com seis dígitos, permitindo agora o uso das 26 letras do alfabeto, além dos algarismos de 0 a 9. Nesse novo sistema, cada letra maiúscula era considerada distinta de sua versão minúscula. Além disso, era proibido o uso de outros tipos de caracteres. Uma forma de avaliar uma alteração no sistema de senhas é a verificação do coeficiente de melhor, que é a razão do novo número de possibilidades de senhas em relação ao antigo. O coeficiente de melhor da alteração recomendada é 6 a) 62 6

10

b) 62! 10!

c) 62! 4!

10! 56!

d) 62! – 10!

Um recipiente hermeticamente fechado e opaco contém bolas azuis e bolas brancas. As bolas de mesma cor são idênticas entre si e há pelo menos uma de cada cor no recipiente. Na tentativa de descobrir quantas bolas de cada cor estão no recipiente, usou-se uma balança de dois pratos. Verificou-se que o recipiente com as bolas pode ser equilibrado por: I) 16 bolas brancas idênticas às que estão no recipiente ou II) 10 bolas brancas e 5 azuis igualmente idênticas às que estão no recipiente ou III) 4 recipientes vazios também idênticos ao que contém as bolas. Sendo PA , PB e PR , respectivamente, os pesos de uma bola azul, de uma bola branca e do recipiente na mesma unidade de medida, determine PB

PB

45. (UERJ 2015)

Observe a matriz A, quadrada e de ordem três. 0,3 0,47 0,6 A = 0,47 0,6 x 0,6 x 0,77

e) 626 – 106

43. (Fuvest 2014, adaptada)

os quocientes PA e PR

Calcule, em km², a área do Triângulo das Bermudas, conforme a representação plana da figura.

Considere que cada elemento aij dessa matriz é o valor do logaritmo decimal de (i + j). O valor de x é igual a: a) 0,50 b) 0,70 c) 0,77 d) 0,87

46. (Mackenzie 2015)

1 1 0 1 0 0 0 0 0 Se A = 0 1 0 , B = 0 1 0 , C = 0 0 0 e os inteiros x e y são tais 0 0 1 0 0 1 0 0 0 que A² + x . A + y . B = C, então a) x = 0 b) x = 1 c) x = – 2 d) x = – 1 e) x = 2 GE MATEMÁTICA 2017

137


SIMULADO

RESPOSTAS

5. Ao brincar no aplicativo, a bateria se descarrega de modo constante: 1% a

CAPÍTULO 1

cada 3 minutos. Como ainda restam 50% de carga na bateria, isso dá a Fabiano 50 . 3 = 150 minutos para brincar. Como cada uma hora corresponde a 60 minutos, isso corresponde a um intervalo de 2 horas e meia. Resposta: B

1. Calculando os 60% de carga (12 toneladas) sobre o ponto central: 10060 . 12 = 7,2 t

CAPÍTULO 2

Esta é a carga máxima para o ponto central. O restante da carga (40%), a ser distribuído entre dois outros pontos, correspondem a 40%, que é a diferença entre 100% e 60%: 12 – 7,2 = 4,8 t Esta é a carga para os outros dois pontos. Então, cada um deles receberá metade dela: 2,4 t. Resposta: C

2. Vamos chamar de x o valor a ser aplicado na caderneta de poupança. Então,

o valor destinado ao fundo de investimentos é (1 000 000 – x). O rendimento total de pelo menos R$ 72 000 (⩾ 72 000) é a soma dos rendimentos nas duas aplicações, acrescida das taxas de rendimento em cada uma: • 6% para a caderneta de poupança; • 7,5% para o fundo. Então, 6% . x + 7,5% . (1 000 000 – x) ⩾ 72 000 0,06 . x + 0,075 . (1 000 000 – x) ⩾ 72 000 0,06 . x – 0,075 . x + 75 000 ⩾ 72 000 –0,015 . x ⩾ –3 000 0,015 . x ⩽ 3 000 3 000 3 000 . 1 000 3 000 x # 0, 015 = 15 = = 200 000 15

6. Pelo teorema de Tales, os segmentos a, b e c são proporcionais a 18, 24 e 33, respectivamente. Chamando a constante de proporcionalidade de k, temos: a = 18k, b = 24k e c = 33k. O enunciado informa que a + b + c = 100 . Portanto, 18k + 24k + 33k = 100 75k = 100 → k = 100 = 4 75 3 Agora, basta aplicar o valor de k à medida de cada segmento: 4 a = 18 . 3 = 24

3. Pelo enunciado, a proporção de cimento/areia/brita é de 1 : 4 : 2.

Somando os fatores, temos: 1 + 4 + 2 = 7. Assim, do total, 1/7 é cimento, 2/7 correspondem a areia e 4/7, a brita. O volume encomendado é de 14 m3. Então, o volume de cimento é: 1 . 14 = 2 m3 7

4 c = 33 . 3 = 44

Resposta: A

7. Observe a imagem do quadrado de lados L, inscrito em um círculo: L r=5 r=5

1000

Portanto, o valor a ser aplicado na caderneta de poupança deve ser, no máximo, de R$ 200 000. Resposta: A

4 b = 24 . 3 = 32

Se r = 5 cm, então as diagonais do quadrado medem 10 cm. Repare que essa diagonal forma com dois lados L um triângulo retângulo: L

r=5

Resposta: B

4. Vamos chamar as portas de altura normal de N e as portas mais altas de

A. Se a espessura se mantém, então, trabalhamos com retângulos, cuja área é dada pela expressão A = b . h Então, AN = bN . hN E AA = bA . hA Para manter a quantidade de material, então, todas as portas devem ter a mesma área: AA = AN Para AA: h teve um aumento de 1/8 sobre hN→ hN + 1 hN / 8 = 9 hN / 8. Portanto, AA = bA . 9hN / 8 Para as portas mais altas, AA = bA . 9 hN /8 Igualando as áreas: AA = AN → bA . 9 hN /8 = bN . hN Cortando hN dos dois lados da igualdade, ficamos com 9bA / 8 = bN → bA = 8 bN /9 A proporção entre a largura das duas portas, portanto, é de 8/9 Resposta: D

138 GE MATEMÁTICA 2017

Por Pitágoras, temos: d 2 = L2 + L2 2L2 100 10 2 = 2L2 " 2L2 = 100 " 2 = 2 " L2 = 50 " L = 5 2 2

A área do quadrado é lado vezes lado: L2. Então, A = (5 2) = 25 . 2 = 50 cm 2 Resposta: E

8. A equação reduzida da circunferência é: ( x – k ) + ( y – y ) – r 2

c

2

c

2

O centro tem coordenadas (xC , yC ) e o raio é r. Substituindo os valores dados das coordenadas do centro e do raio, temos: (x – 3)² + (y – 4)² = 6² x² – 6x + 9 + y² – 8y + 16 – 36 = 0 x² – 6x + y² – 8y – 11 = 0 Rearranjando: x² + y² – 6x – 8y – 11 = 0 Resposta: A


9. Na figura, percebemos que o eixo maior da elipse está na vertical e o menor, na horizontal. Então, a equação reduzida da elipse é

Q x – m V2 b2

+

Q y – n V2 a2

=1

Portanto o comprimento do lado b do retângulo é 2 e a ordenada dos pontos P e Q é 2. O ponto Q, de coordenadas (xQ , 2), está localizado sobre a reta r que passa pelos pontos A e B, dos quais conhecemos suas coordenadas. Podemos então obter a equação da reta que passa por A e B, no formato y = ax + b:

em que (m,n) são as coordenadas do centro da elipse. No gráfico do enunciado encontramos: • O centro está no ponto C (2, 3); • O eixo maior vale 2a = 6 → a = 3 • O eixo menor vale 2b = 4 → b = 2 Substituindo esses valores na equação da elipse, temos:

Q x – 2 V2 22

Q x – 2 V2 4

+

Q y – 3 V2 32

=1

+

Q y – 3 V2 9

=1

4 y= 3x

cartesiano os pontos e as figuras descritas no enunciado. Veja: • na retinha pontilhada vertical (dentro do retângulo), identificar: b • do lado de fora de todas as figuras, depois do ponto C, fazer outra retinha tracejada, vertical. Identificar: h y B 4 4-b

A

b M

3 a

Substituindo a ordenada do ponto Q na equação recém-obtida da reta que passa por a e b, podemos obter o valor da abscissa de Q:

2 =

4 3x

6 3 xQ = 4 = 2

Portanto, as coordenadas de Q são S 2 , 2 X Como o ponto Q está a meia altura de AB, na abscissa y = 2, a razão de semelhança entre os dois triângulos é 1:2. Como a base AC do triângulo ABC mede 8 unidades, então, a base QP do triângulo QBP mede 4 unidades. Portanto, a abscissa de P será igual a 3 + 4 = 3 + 8 = 11 . Portanto, as coordenadas de P são S 11 , 2 X . 2 2 2 2 2 Resposta: D 3

P h

b

4–0 4 a= 3 0 = 3 –

Como a reta r passa pela origem do sistema cartesiano, então seu coeficiente linear é zero. Com isso, a equação da reta fica:

10. A primeira coisa a fazer em problemas desse tipo é localizar no plano

Q

voltada para baixo. Portanto, o valor máximo da função está nas coordenadas do vértice da parábola. Esse ponto corresponde à área máxima do retângulo. O valor de b em que isso ocorre é b vértice = –8 –4 = 2

b N

x 8C

Repare: • PQ // MN. Então, os triângulos ABC e QBP são semelhantes; • o lado QP = MN = a; • o lado QM = PN = b; • a altura do triângulo ABC vale 4. Então, a altura do triângulo QBP vale 4 – b. (Lembre-se que as relações de semelhança são válidas também para a altura dos triângulos.) Sabemos que o lado AC vale 8. Então podemos escrever a razão entre os dois triângulos semelhantes: hABC / hQBP = 8 / a

11. A questão pede conhecimentos em volume de sólidos. O silo é um prisma,

cujo volume é dado por: A = Abase . h O prisma pode ser considerado na vertical ou na horizontal. Vamos considerar a figura na vertical:

b h

c

B

4 8 = a 4– b

Multiplicando em cruz e simplificando obtemos: 4a = 32 – 8b → a = 8 – 2b (I) A área do retângulo MNPQ é dada por AMNPQ = a . b (II) Substituindo (I) em (II), temos: AMNPQ = (8 – 2b) . b

AMNPQ = 8b – 2b2

A partir daqui você precisa usar seus conhecimentos sobre funções de segundo grau (capítulo 3). Esta é uma função do tipo f(x) = x 2 + x + c. A única diferença é que, em vez de x, nossa variável é b. Repare o coeficiente do termo que é elevado ao quadrado (b 2) é negativo. Então o gráfico dessa equação é uma parábola com a concavidade

Agora, podemos definir os valores, segundo o enunciado: • h = 20 m; • a largura máxima do silo é de 6 m. Então o lado B = 6 m; • a cada 1 metro de altura, a largura do trapézio diminui 0,5 m. Então, em 2 m, a largura vai diminuir 0,5 . 2 = 1 metro. Com isso, a base menor do trapézio (b) é de 5. Calculando a área da base do prisma (área do trapézio): Q6 + 5 V . 2 h A trapézio = Q Base + base V . 2 A base = = 11 m 2 2 O volume do prisma correspondente do silo é: Vprisma = 11 . 20 = 220 m³ Se uma tonelada de forragem ocupa 2 m³, um silo de 220 m³ comporta 110 toneladas. Resposta: A GE MATEMÁTICA 2017

139


SIMULADO

12. Os triângulos da figura são todos semelhantes, já que todos são retângulos

e, conforme a figura, seus catetos correspondentes são paralelos. Pelos valores CD 12 3 fornecidos no enunciado, a razão de semelhança entre todos eles é: AB = 16 = 4 Aplicando o teorema de Pitágoras ao triângulo ABC, descobrimos a medida do segmento AC (hipotenusa do primeiro triângulo): a2 = b2 + c2 → a2 = 162 + 122 a2 = 256 + 144 a = 400 → a = 20. A cada trecho da horizontal, a distância diminui numa razão de 3/4. A partir daqui você tem de trabalhar com os conceitos de progressão geométrica (PG, capítulo 6). O comprimento do segmento AP equivale à soma dos termos de uma PG infinita, em que a1 = 20 e q = 3/4. Substituindo esses valores na fórmula, obtemos: 20 20 Lim S n = 3 = 1 = 80 n" 3 1– 4 4 Resposta: C

b) O gráfico da função concentração em função do tempo é uma função de segundo grau. No eixo x, temos o tempo, e no y, a concentração. Repare que, na função, t2 tem coeficiente negativo (–0,05t²). Portanto, a parábola tem concavidade voltada para baixo, e a máxima concentração do medicamento ocorre no ponto correspondente ao vértice da parábola. A parábola é simétrica em relação a esse ponto. Então, a coordenada x do vértice é o ponto central entre as duas raízes, o que encontramos pela média aritmética: 10 + 30 = 20 horas 2

Somando 20 horas ao horário em que foi ministrada a primeira dose, chegamos à resposta: 11 + 20 = 7 horas da manhã da terça-feira.

15. O enunciado informa que os pontos (–3 , 3) e (3 , –1) pertencem ao gráfico da função. Portanto, eles obedecem à lei y = ax + b. Calculando o coeficiente angular a da função, temos:

13. A equação reduzida da circunferência é (x – x ) + (y – y ) = r 2

2

c

2

c

em que o centro tem coordenadas (xC , yC ) e o raio é r. Vamos desenvolver os quadrados da expressão acima e comparar com a expressão da circunferência fornecida no enunciado, depois de separarmos os termos semelhantes: x 2 2xx c + x 2c + y 2 2yy c + y 2c = r 2

x² + 2x + y² + my =n As setas indicam termos semelhantes em correspondência nas expressões. Podemos então dizer que 2x = –2xxC → xC = –1 m Da mesma forma, my = –2yyC → yc = 2

Ou seja, o centro da circunferência tem coordenadas S –1, – 2 X . Sabemos também que esse ponto pertence à reta y = –x + 1. Substituindo o valor da abscissa do vértice, temos: y = – (–1) + 1 = 2. m

m

a=

4 Q 3– Q –1 VV ∆y 2 = =–6 =–3 ∆x –3 –3 Q V

2

Substituindo esse valor no a da expressão da função: y = – 3 x + b Para calcular o valor de b, basta substituir as coordenadas de qualquer um dos pontos conhecidos: (–3, 3) ou (3, –1). Substituindo o ponto (–3 . 3): 2 3 = – 3 Q –3 V + b

3=2+b b=3–2=1 2

Então, a = – 3 e b = 1.

Portanto, – 2 = 2 , m = – 4.

Resposta: E

Substituindo o valor de m e as coordenadas do ponto (–3, 4) que também pertence a ela na expressão da circunferência, temos: x² + 2x + y² + my = n (–3)² + 2 . (–3) + 4² + (–4) . 4 = n 9 – 6 + 16 – 16 = n n = 3 Resposta: A

16. A função que relaciona o valor da corrida (y) com a quantidade de quilô-

CAPÍTULO 3

14. a) Substituindo C(t) = 40 na função, determinamos quanto tempo depois da ingestão da dose a concentração do medicamento atinge esse valor: 40 = –0,05t² + 2t + 25 –0,05t² + 2t + 25 – 40 = 0 –0,05t² + 2t – 15 = 0 Resolvendo essa equação por B: ∆ = 4 – 4.(– 0,05) . (–15) = 4 – 3 = 1 –2 ! 1 t = –0, 1 → t1 = 10, t2 = 30.

O primeiro momento em que a concentração de medicamento atinge 40 ppm é a raiz de menor valor: t1 = 10 horas. Se Álvaro tomou o medicamento às 11 horas da manhã de 2ª feira, esse momento corresponde às 21 horas do mesmo dia.

140 GE MATEMÁTICA 2017

metros rodados (x) é uma função do primeiro grau, de forma geral y = ax + b, onde b representa o valor da bandeirada e a, o custo por quilômetro. Vamos substituir os valores fornecidos na forma geral da função e resolver o sistema de equações. Para x = 8 km, temos y = 28,50. Então 28,50 = a . 8 + b Para x = 5 km, y = 19,50. Então, 19,50 = a . 5 + b Montando o sistema de equações: 28,50 = 8a + b 19,50 = 5a + b

{

Subtraindo as duas equações: 28,50 = 8a + b – 19,50 = – 5a – b 9 = 3a – 0 → a = 3 Substituindo esse valor em qualquer uma das equações, encontramos o valor de b, a bandeirada: 28,50 = 3 . 8 + b → b = 28,50 – 24 = 4,50. Resposta: D


17. As funções para o lucro das duas empresas são funções de primeiro 19. Para se determinarem os pontos de intersecção dos gráficos de quaisquer

grau, cujo gráfico é sempre uma reta. Mas repare que os coeficientes angulares (o coeficiente a) são distintos. Então essas retas estão inclinadas em ângulos diferentes em relação ao eixo x e, portanto, se cortam em algum ponto do plano cartesiano. Esse é o ponto em que os lucros serão iguais. O ponto de encontro das duas retas pertence a ambas. Portanto, igualamos suas equações: 3t – 1 = 2t + 9 LA = LB 3t – 2 t = 9 + 1 → t = 10 As duas empresas têm lucro igual no 10º mês. Resposta: D

duas ou mais funções, basta igualar as equações das curvas que elas representam. Para isso, é preciso isolar y do lado esquerdo da equação. Primeiro, vamos trabalhar com a equação da reta: y–2=x+2→y=x+4 1 O ponto de intersecção é aquele em que x = x + 4 Multiplicando os dois lados da equação por x, eliminamos o x do denominador de 1/x: 1 = x² + 4x → x² + 4x – 1 = 0 Resolvendo a equação de 2º grau, temos ∆ = 16 – 4 . 1 . (–1) = 20 → x 1 = –2 + 5 e x 2 = –2 – 5

18. Para resolver a questão, você terá de trabalhar com sistemas de equações.

Substituindo esses valores na equação da reta dada, temos:

Para calcular o valor de f(4), precisamos, antes, escrever a expressão de f(x), ou seja, precisamos determinar os valores dos coeficientes a, b e c. O enunciado fornece as coordenadas de três pontos que pertencem à função f(x): (1, 2), (2, 5) e (3,4). Substituindo os valores de x e y na forma geral de f(x) = ax² + bx + c, montamos um sistema de três equações com três incógnitas: Para (1, 2) → 2 = a . 1² + b . 1 + c → a + b + c = 2 (I) Para (2, 5) → 5 = a . 2² + b . 2 + c → 4a + 2b + c = 5 (II) Para (3, 4) → 4 = a . 3² + b . 3 + c → 9a + 3b + c = 4 (III) Temos então o sistema: a + b + c = 2 (I) 4a + 2b + c = 5 (II) 9a + 3b + c = 4 (III)

{

y 1 = –2 +

5 +4=2+

5 e y 2 = –2 – 5 + 4 = 2– 5

Os pontos de intersecção das duas curvas têm coordenadas Q –2 + 5 , 2 + 5 V e Q –2 – 5 , 2 – 5 V

20. A função f(x) = a . (x – m)² + n é a função de 2º grau em sua forma ca-

nônica, em que m e n correspondem respectivamente às coordenadas x e y do vértice da parábola. O coeficiente a é o mesmo nas duas formas, de modo que a = 2. Calculando as coordenadas do vértice: b 1 1 x v = – 2a = – 2 . 2 = 4

Para obter o valor de y no vértice, basta substituir o valor de x em f(x): 4 48 46 1 2 1 2 y v = 2 S 4 X – 4 + 3 = 16 – 16 + 16 = 16

Para resolvê-lo, podemos isolar c na equação (I) e substituir o resultado nas demais: a+b+c=2→c=2–a–b (II) 4a + 2b + 2 – a – b = 5 → 4a – a + 2b – b = 5 – 2 → 3a + b = 3 (IV) (III) 9a + 3b + 2 – a – b = 4 → 9a – a + 3b – b = 4 – 2 → 8a + 2b = 2 (V)

Somando os três coeficientes, temos:

As expressões (IV) e (V) compõem um novo sistema, agora de duas equações, com apenas duas incógnitas:

21. a) A função que define a reta é uma função de primeiro grau, (y = ax + b).

{

3a + b = 3 (IV) 8a + 2b = 2 (V)

Multiplicando a equação (IV) por (–2), temos:

{

– 6a – 2b = – 6 8a + 2b = 2

Somando as duas obtemos: 2a = –4 → a = –2

46 1 32 46 4 82 41 a + m + n = 2 + 16 + 4 = 16 + 16 + 16 = 16 = 8

Resposta: C

O coeficiente linear (b) é o valor de y no ponto em que a reta cruza o eixo y. Portanto, neste caso, b = 2. Substituindo esse valor na expressão geral da função, temos: y = ax + 2. Para calcular o valor do coeficiente angular (o coeficiente a), substituímos as coordenadas do outro ponto fornecido no enunciado: (–2, 0). 0 = a . (–2) + 2 2a = 2 → a = 1 A equação da reta é y = x + 2.

Substituindo na forma geral de f(x), temos: f(x) = –2x² + 9x – 5

b) A forma geral da parábola é uma função de segundo grau, na forma y = ax² + bx + c. O ponto c corresponde ao valor de y para o qual x = 0. Pelos pontos sabemos que isso ocorre no ponto (0,8). Então, sabemos que c = 8. Os outros pontos fornecidos têm coordenada y = 0. Então, o valor de x, nesses pontos, correspondem às raízes da função: x1 = 2 e x2 = 4. Substituindo esses valores na forma fatorada da função temos: y = a . (x – x1 ) . (x – x2 ) y = a . (x – 2) . (x – 4) → y = a . (x² – 6x + 8) Como o valor de c = 8, temos que a = 1. A equação da parábola é y = x² – 6x + 8

Agora podemos calcular o valor de f(4): f(4) = –2 . 4² + 9 . 4 – 5 = –2 . 16 + 36 – 5 = –32 + 31 f(4) = – 1 Resposta: B

c) Para determinar as coordenadas dos pontos em que as duas curvas se cruzam, basta igualar as duas equações: x² – 6x + 8 = x + 2 x² – 7x + 6 = 0

Agora, basta voltar trocando as incógnitas descobertas nas equações anteriores e determinar os valores de a e b: Em (IV) →3 . (–2) + b = 3 → – 6 + b = 3 → b = 9 Em (I) → –2 + 9 + c = 2 → c = 2 – 9 + 2 → c = –5 Portanto, a = –2, b = 9 e c = –5.

GE MATEMÁTICA 2017

141


SIMULADO

Resolvendo a equação por Bhaskara, obtemos: ∆ = 7² – 4 . 1 . 6 = 49 – 24 = 25 → x = 7 ! 5 → x1 = 1 e x2 = 6 2

Substituindo esses valores na equação da reta ou da parábola, tanto faz, encontramos os valores de y para os dois pontos. Na equação da reta: y=x+2 y1 = 1 + 2 = 3 y2 = 6 + 2 = 8. As coordenadas dos pontos em que a reta e a parábola se cruzam são (1, 3) e (6, 8).

Voltando à equação da parábola, descobrimos o valor de y correspondente a x = 2: y = 2² – 4 . 2 + 3 = –1 O vértice de f(x) é o ponto (2, –1). O mesmo para g(x) = –x² – 4x – 3;

Q –4 V b A coordenada x do vértice: x v = – 2a = – 2 = 2

A coordenada y do vértice: para x = 2, y = – (–2)² – 4(–2) – 3 = 1 O vértice de g(x) é o ponto (–2, 1). Colocando os pontos em um sistema cartesiano, calculamos a distância entre eles:

22. a) Para determinar f(0), basta trocar x por 0 na função:

2 G

f(0) = 4º – 6 . 2º + 8 = 1 – 6 + 8 = 3 f(0) = 3

b) Vamos resolver a equação 4x – 6 . 2x + 8 = 168 . Vamos usar um truque: Sabemos que 4x = (2²) x. E, pela propriedade de potência da potência, sabemos que (2²) x = (2x)². Chamando 2x = t, trocamos a variável da função (ou a incógnita da equação): t² – 6t + 8 = 168 → t² – 6t – 160 = 0 Por Bhaskara: ∆ = 36 – 4 . 1 . (–160) = 676 t=

1

2 -3

6 ! 26 2

-2

y

-1

0

a 0

1

-1 4

2 F

x 3

A distância entre os dois pontos, representada pelo segmento a na figura, é a hipotenusa de um triângulo retângulo de catetos 2 e 4. Aqui entra o teorema de Pitágoras (capítulo 2): a² = 2² + 4² = 4 + 16 = 20 a=

20 = 2 5

Resposta: E

Então, t1 = 16 e t2 = –10 Agora precisamos voltar à incógnita original, trocando t por 2x: t1 = 16 → 2x = 16 → x = log2 16 = 4 t2 = –10 → 2x = – 10 → x = log2 – 10. Este valor é impossível, porque não existe nenhuma potência que eleve 2 e resulte em número negativo. A resposta é x = 4. c) Encontrar os valores de x de modo que f(x) seja menor que zero equivale a encontrar os valores de x para os quais y < 0. Vamos encontrar os valores em que f(x) = 0. De novo, fazendo a troca 2 x = t, a função fica f(t) = t² – 6t + 8. Então, temos t² – 6t + 8 = 0 Resolvendo pela fórmula de Bhaskara, chegamos às raízes t1 = 2 e t2 = 4 O coeficiente de t2 é positivo. Então, a parábola tem concavidade voltada para cima. E os valores de y menores que zero estarão no intervalo entre as duas raízes. Veja:

_ _ _ _ _

+ + + + + 0

1

2

3

+ + + + + 4

5

0 6

Segundo o esboço, f(x) < 0 para valores de t entre 2 e 4 , ou seja, para 2 < t < 4. No entanto, não se esqueça de que t é a incógnita que substituiu 2 x. Agora temos de desfazer a troca: 2 < t < 4 → 2 < 2 x < 4. Isso é o mesmo que 2 1 < 2 x < 2². Pelas propriedades das potências, as relações entre expoentes de bases iguais são as mesmas que as relações entre as bases. Assim, se 21 < 2 x < 2², então 1 < x < 2 O conjunto solução é S = {x ∈ R| 1 < x < 2}.

23. Vamos determinar as coordenadas dos vértices das duas parábolas, pela fórmula para obter a coordenada x do vértice: f(x) = x² – 4x + 3

Q –4 V b x v = – 2a = – –2 = –2

142 GE MATEMÁTICA 2017

CAPÍTULO 4

24. a) Como N corresponde ao número de átomos inicial, ou seja, em t = 0, 0

no gráfico, o ponto em que t = 0 tem coordenadas (0,6) e corresponde a log10N0 = 6,0

b) Aplicando a definição de logaritmo ao valor encontrado na resposta a, temos log10 N0 = 6,0 → N0 = 10 6 = 1 000 000 Portanto, são 1 000 000 de átomos radioativos em t = 0. c) Pela definição de meia-vida, devemos determinar o instante t em que a quantidade de átomos radioativos seja a metade em relação a um instante anterior N conhecido, por exemplo N0 = 106. Assim, log N t = log 20 Aplicando o logaritmo de base 10 a ambos os lados da equação e aproveitando as propriedades operatórias de logaritmos, teremos: N0 log N t = log 2

log Nt = log N0 – log 2 log Nt = 6,0 – 0,3 = 5,7 Pelo gráfico apresentado, o ponto em que log Nt = 5,7 corresponde a t = 6 horas. Esta é a meia-vida do elemento descrito na questão.

25. O valor de e pouco importa, já que é possível chegar à resposta da ques-

tão aplicando apenas as propriedades das operações com potências. Segundo o enunciado, Q0 = 6 000 e, para t = 20 min, Q(20) = 12 000. Substituindo esses valores na expressão matemática da função, temos: 12 000 = 6 000 . e k . 20 → 12 000 / 6 000 = e k . 20 → e 20 . k = 2 O enunciado pede Q(60), e sabemos que 60 = 20 . 3. Então: Q(60) = 6 000 . e60K = 6 000 . (e20k)3 = 6 000 . 23 =48 000 = 4,8 . 104 Resposta: E


26. Determinando os pontos em que f(x) = 0, encontramos os pontos em 29. A função exponencial apresentada tem sua potência em base e, o número de que a curva do gráfico da função cruza o eixo x. 4– x – 2 = 0 → 4– x = 2 → (22) –x = 21 2– 2x = 21 → –2x =1 → x = – 1 2

Euler. O dado apresentado, ln 2 = 0,69, indica o logaritmo natural de 2, ou seja, o logaritmo de 2 na base e. Com isso, temos garantido que tudo estará em bases compatíveis. Metade da quantidade inicial de césio radioativo é Q 0 .

Resposta: C

Substituindo esse valor na função dada, temos:

2

27. A equação é o produto de três fatores: (x + 3), 2

1

x2 – 9

e e log |x² + x – 1| Um produto é igual a zero quando um dos fatores for igual a zero. Vamos analisar cada um dos fatores, igualando-o a zero: • x + 3 = 0 para x = –3. Esta é a primeira raiz da equação. 2

• 2 x –9 = 0 Esta equação não tem raiz; uma potência de base 2, qualquer que seja o expoente, vai resultar um valor maior que zero, sempre. • log |x² + x – 1| = 0 Aplicando a definição de logaritmo, temos: |x² + x – 1| = 10º = 1

–1 –0,023t Cancelando a quantidade inicial em ambos os termos, ficamos com 2 = 2 = e Como ln 2 = 0,69, então e–0,69 = 2–1 Então, e–0,69 = e–0,023t Portanto, –0,69 = –0,023t t = 30 dias. Resposta: B

30. Temos as coordenadas de dois pontos da função f(x): (p, 1) e (q, 2).

Substituindo esses valores na expressão da função: 1 = log k p → p = k1 (I) 2 = log k q → q = k2 (II) O trapézio assinalado no gráfico tem área de 30 u². O comprimento da base menor dele é 1 u e o comprimento da base maior é 2 u. Sua altura h equivale a q – p. Assim: 30 =

Se o módulo de x² + x – 1 é igual a 1, então, sem ser em módulo, a expressão pode ser igual a 1 ou igual a –1: x² + x – 1 = 1 (I) ou x² + x – 1 = –1 (II) Para (I): x² + x – 2 = 0 → ∆= 1 – 4 . 1 . (–2) = 9 → x =

Q0 –0,023 t 2 = Q0 e

–1 ! 3 → x1 = –2 e x2 = 1 2

Para (II): x² + x = 0 → x(x + 1) = 0 → x3 = 0 e x4 = –1 Portanto, a equação tem cinco elementos distintos em seu conjunto solução: S = {–3 , –2, –1, 0, 1} Resposta: E

28. O enunciado fornece a equação que relaciona a magnitude de um

terremoto em relação a um terremoto padrão, que libera energia E0 , e o valor da magnitude do terremoto de 11 de março de 2011. A base do logaritmo é decimal. Substituindo esses valores na equação, obtemos: 9 . 3 = log S E4,5 X 2

10

Precisamos isolar a incógnita E. Para isso, vamos passar para o 1º membro da equação todos os outros elementos que estão no 2º membro, aplicando as propriedades dos logaritmos:

Q1 + 2 V . h 2

h = 20 h = q – p. Então, q – p = 20. Substituindo (I) e (II) temos: k² – k = 20 → k² – k – 20 = 0 Resolvendo a equação, obtemos k1 = 5 e k2 = –4 O valor de k2 não convém, pois k > 0. Então, o valor de k + p – q = 5 + 5 – 25 = –15 Resposta: B

CAPÍTULO 5

31. A questão exige apenas que você se lembre de que ângulos maiores que

360o significam mais de uma volta na circunferência. Nesse caso, para achar o ângulo correspondente, descontadas as voltas a mais, dividimos 1 000° por 360° 1 000° : 360° = 2,777... Duas voltas são 360º . 2 = 720º. Subtraindo esse valor dos 1 000º, temos: 1 000° – 720° = 280°. Isso significa que rodar 1 000° e 280° tem os mesmos valores para seno, cosseno e tangente. Localizando 280° na circunferência trigonométrica, encontramos seu quadrante e identificamos se seu seno é positivo ou negativo:

3 E 9 . 2 = log S 4,5 X 10

27 27 E 4,5 2 = log S 10 4,5 X " 2 = log E – log 10

27 9 36 27 2 = log E– 4, 5 " log E = 2 + 2 = 2

log E = 18 Portanto, a ordem de grandeza do terremoto foi E = 10 joules. Resposta: D 18

O ângulo está no quadrante IV. E todos os ângulos aqui têm seno negativo. Portanto, seno negativo. Resposta: A GE MATEMÁTICA 2017

143


SIMULADO

32. Imaginando o triângulo descrito na circunferência, desenhamos a 36. Em qualquer PA, a soma de termos equidistantes do termo central é situação (lembre que não sabemos o tipo de triângulo em questão, se é retângulo, equilátero etc.) A figura é apenas para conduzir o raciocínio: CA x CA

20 S n = 42 000 = Q a 2 + a 19V . 2

42 000 = (3 800 + a19 ) . 10 4 200 = 3 800 + a19 → a19 = 400 Como a19 = a2 + 17r , então 400 = 3 800 + 17r → 17r = – 3 400 → r = –200 Resposta: B

y z

constante. Assim, na PA com 20 termos da questão, a1 + a20 = a2 + a19 = a3 + a18 = ... Sabemos o valor total e o valor da segunda parcela ( a2 ). Então, temos que

CA

y z x = sen y = sen z = 2R sen x x x Tomando: sen x = 2R " sen x = 2R

Pela lei dos senos, sabemos que:

37.

Fazendo o mesmo com os demais membros da igualdade, temos: k .x . y .z x . y .z k .x . y .z y x z " = 3 2R . 2R . 2R = R 8R 3 R3

1

Cancelando os termos comuns aos dois lados da expressão, temos: k = 8 = 0, 125 Resposta: C

33. Analisando os ângulos somados, você identifica que são suplementares

os ângulos dos pares: 165° e 15°, 155° e 25°, 145° e 35°. Somando os ângulos de cada um desses pares obtemos 180º. Se dois ângulos são complementares, seus senos são iguais e seus cossenos, opostos. Então: i. cos 165° = – cos 15° ii. sen 155° = sen 25° iii. cos 135° = – cos 45° Substituindo esses valores na expressão temos: cos 165° + sen 155° + cos 145° – sen 25° + cos 35° + cos 15° = –cos 15° + sen 25° – cos 35° – sen 25° + cos 35° + cos 15° = 0 Resposta: C

34. Do gráfico obtemos as coordenadas de alguns pontos da curva e que, é claro, obedecem à equação de f(x) = a + b . cos(x). São eles: (0, 5) e (π, 1). Esses são pontos estratégicos, pois cos 0 = 1 e cos π = –1. Substituindo os valores na função temos: 5 = a + bcos 0 → 5 = a + b (I) 1 = a + bcos π → 1 = a – b (II) Resolvemos, então, o sistema de equações, somando I e II: 5 = a + b (I) 1 = a – b (II) 6 = 2a Então a = 6/2 → a = 3 Substituindo esse valor em (I), encontramos b: 5 = 3 + b → b = 2 Voltando o que pede a questão: a . b = 3 . 2 = 6. Resposta: D

CAPÍTULO 6

35. Se os três números estão em PA, então, a razão (r) da PA entre quaisquer dois termos subsequentes é a mesma: r = a2 – a1 = a3 – a2 x + 14 – (5x – 5) = 6x – 3 – (x + 14) → –4x + 19 = 5x – 17 → –9x = – 36 → x = 4 Com isso, os três termos são (5 . 4 – 5 ; 4 + 14 ; 6 . 4 – 3) = (15, 18, 21). A soma desses termos é 54. Resposta: B

144 GE MATEMÁTICA 2017

O primeiro número entre 100 e 400 que tem o algarismo das unidades igual a 4 é 104 ; o próximo é 114; daí vêm 124, 134 e assim por diante. O último termo dessa sequência é 394. Trata-se de uma PA de razão r = 10. Vamos calcular a soma dos termos da sequência (104, 114, 124, 134,... 394). Mas, primeiro, determinar quantos termos tem a sequência. Para isso, usamos o termo geral e substituimos 394 para descobrir a sua posição na sequência: an = 104 + (n – 1) . 10 → 394 = 104 + (n – 1) . 10 290 = (n – 1) . 10 → n – 1 = 29 → n = 30 Agora, basta substituir os valores na fórmula da soma dos n termos da PA: 30 S 30 = Q a 1 + a 30V 2 = Q 104 + 394 V . 15 = 489 . 15 = 7 470 Resposta: E

38. Pela tabela, percebe-se que os valores das projeções da produção para cada ano é uma PA em que a1 = 50,25 e r = 1,25. De 2012 a 2021, temos 10 anos. Para calcular a soma de todas as produções anuais, somamos os 10 primeiros termos da PA (50,25; 51,50; 52,75; ... a10 ). Identificando a10: a10 = 50,25 + 1,25 . 9 = 61,50 toneladas Pela fórmula da soma dos termos de uma PA, temos S 10 =

Q 50, 25 + 61, 5 V . 10

Resposta: D

2

= 558, 75 toneladas.

39. Esta questão exige conhecimentos sobre PA e, também, sobre proprie-

dades das operações com logaritmos (capítulo 4). Em qualquer PA de três termos, a soma dos termos das pontas é sempre o dobro do termo central. Observe os exemplos: (3, 8, 13) → 3 + 13 = 16 = 2 . 8 (12, 9, 6) → 12 + 6 = 18 = 2 . 9 (4, 7, 10) → 4 + 10 = 14 = 2 . 7 Generalizando, a1+ a3 = 2 . a2 No caso da PA do problema, temos então: log2x + log88x =2 . log44x Vamos transformar todos os termos para base 2 utilizando a fórmula da mudança de base e determinar o valor de x: log 2 x +

log 2 8x log 2 4x = 2. log 2 8 log 2 4

Substituindo os valores conhecidos e aplicando as propriedades operatórias dos logaritmos, vamos tentar isolar log2x. log 2 x +

log 2 8x log 2 4x = 2. 2 3


log 2 x +

log 2 8 + log 2 x = log 2 4 + log 2 x 3

Candidato II:

M II =

Q . 4 + 25 . 6 4Q + 150 = =? 10 10

Igualando os denominadores no primeiro termo: Candidato III: M III = 21 . 4 + 18 . 6 = 84 + 108 = 192 = 19, 2 10 10 10

3log 2 x + log 2 8 + log 2 x = 2 + log 2 x 3

4log2x + 3 = 3 . (2 + log2x) → 4log2x + 3 = 6 +3 log2x

A nota do candidato II não está fechada, pois depende de seu desempenho em química. O candidato I é o oponente a ser batido. Vamos escrever a inequação que representa a situação em que o candidato II supera a nota do candidato I:

Isolando log2x no primeiro membro: log2x = 3 → x = 8 A sequência é: Q log 2 8, log 4 32, log 8 64 V ,

S 3,

5 5 4 X S6 X 2 ,2 = 2 , 2 , 2

que é uma PA de razão – 1 2 5

6

5

4

15

A soma é : 3 + 2 + 2 = 2 + 2 + 2 = 2

42. Trata-se de dois casos de arranjo com repetição. No primeiro, devemos

Resposta: B

CAPÍTULO 7

40. Vamos chamar de I o conjunto dos falantes de inglês e de E o conjunto

dos falantes de espanhol. Se, dos 1 200 alunos da escola, 300 não falam nem inglês nem espanhol, então eles não pertencem nem a I, nem a E. Consequentemente, 900 alunos falam inglês ou espanhol, ou seja, pertencem à união dos dois conjuntos. Quando se faz a união de dois conjuntos, não basta somar as quantidades de elementos dos dois conjuntos. Se fizermos isso, estaremos contando duas vezes aqueles elementos que pertencem a ambos os conjuntos (a intersecção de I e E), com os alunos que falam os dois idiomas. Por isso, na linguagem de conjuntos, em que n(P) é a quantidade de elementos de um conjunto P qualquer, a quantidade de elementos na união dos conjuntos I e E é: n(E∪I) = n(E) + n(I) – n(E∩I) Substituindo os valores conhecidos, temos: 900 = 500 + 600 – n(E∩I) → n(E∩I) = 200. Isso quer dizer que 200 alunos falam espanhol e inglês. Portanto, falam somente inglês: 600 – 200 = 400 alunos. Falam somente espanhol: 500 – 200 = 300 alunos. Veja o diagrama de Venn que representa essa situação: I

E 300

200

4Q + 150 > 21,8 → 4Q + 150 > 218 → 4Q > 68 → Q > 17 10 Como as notas só podem assumir valores inteiros, a menor nota maior que 17 é 18. Resposta: A

400

arranjar 10 algarismos (de 0 a 9) em uma senha com 6 dígitos. O número de arranjos possíveis é de 106. Na nova senha, pode-se utilizar 10 algarismos (números de 0 a 9), mais 26 letras maiúsculas, mais 26 letras minúsculas, ou seja, um total de 10 + 26 + 26 = 62 símbolos diferentes, a serem arranjados em senhas de 6 dígitos. O número total de arranjos nesse caso é de 626 arranjos. O coeficiente de melhora é a razão entre os possíveis arranjos pelo método novo e os possíveis pelo método antigo. Pelos nossos cálculos, esse coeficiente é 626 / 106. Resposta: A

43. A partir das informações do enunciado, escrevemos três equações:

a.PA + b . PB + PR = 16PB (I) a.PA + b . PB + PR = 10PB + 5PA (II) a.PA + b . PB + PR = 4PR (III)

Em cada uma delas, a e b são as quantidades de bolas azuis e brancas, respectivamente. Igualando os segundos membros de (I) e (II), obtemos: 16 PB = 10 PB + 5 PA → 6 PB = 5 PA → PA = 6 PB

Igualando os segundos membros de (I) e (III), obtemos 16 PB = 4 PR → PR = 16 = 4 4 Resposta: PA = 65 e PR = 4 PB PB PB

CAPÍTULO 8

300

Se já sabemos que o aluno será sorteado entre os que não falam inglês, o nosso espaço amostral tem 600 alunos: aqueles que não falam qualquer uma das duas línguas (300) somados àqueles que só falam espanhol (300). Destes 600, 300 falam espanhol. Portanto a probabilidade procurada é P = 300 = 1 600 2 Resposta: A

41. Vamos calcular a média ponderada de cada candidato e depois compará-las. A soma dos pesos é 10. Candidato I:

MI =

20 . 4 + 23 . 6 80 + 138 218 = = 10 = 21, 8 10 10

5

44.

Vamos, a partir da figura, obter as coordenadas dos pontos A (Miami), B (Bermudas) e C (San Juan): A (0, 2), B (7, 9) e C (9, 0). Montando a matriz A, a partir desses pontos: 0 2 1 A= 7 9 1 9 0 1 Calculando o determinante de A pela regra de Sarrus: 0 2 1 0 2 det A = 7 9 1 7 9 = (0 + 18 + 0) – (81 + 0 +14) = 18 – 95 = –77 9 0 1 9 0 GE MATEMÁTICA 2017

145


SIMULADO A área do triângulo ABC representado no plano cartesiano equivale à metade do valor absoluto do determinante calculado acima. Portanto, a área S do triângulo ABC é SABC = 1 . | det A | = 1 | – 77 | = 38,5 cm2 2 2 O mapa está desenhado em escala de 1 : 17 000 000. Isso significa que cada 1 cm equivale, na realidade, a 17 000 000 cm. Fazendo a conversão de centímetros para quilômetros, temos que 1 cm no mapa corresponde a 170 km. Então, 1cm² = 170 . 170 km2 = 28 900 km². Já encontramos a área do triângulo no mapa: 38,5cm². Então, o Triângulo das Bermudas tem de área real 38,5 . 28 900 = 1 112 650 km². Resposta: A área total do Triângulo das Bermudas é de 1 112 650 km2

45. A questão exige que você domine dois temas: matrizes e logaritmos.

Por partes: O enunciado informa que cada elemento aij dessa matriz é o valor do logaritmo decimal de (i + j). Em linguagem matemática, temos, então que aij = log (i + j) Se todos os elementos da matriz são o resultado dessa expressão, então podemos definir que a11 = log(1 + 1) = log2 = 0,3 Repare que o valor x aparece em duas posições: a23 e a32. Então, x = a23 = a32 = log(2 + 3) = log5 Podemos escrever 5 = 10 : 2. Então, pelas propriedades dos logaritmos, temos x = log5 = log 10/2 = log10 – log2 Conhecemos todos os valores da expressão final. Então, x = log10 – log2 = 1 – 0,3 = 0,7 Resposta: B

46. Vamos encontrar o valor de cada termo da função A² + x . A + y . B = C

1 1 0 1 1 0 1 2 0 A = 0 1 0 . 0 1 0 = 0 1 0 0 0 1 0 0 1 0 0 1 2

Presidente do Grupo Abril: Walter Longo Diretor de Operações: Fábio Petrossi Gallo Diretor Geral de Publicidade: Rogério Gabriel Comprido Diretor de Planejamento, Controle e Operações: Edilson Soares Diretora de Serviços de Marketing: Andrea Abelleira Diretor de Tecnologia: Carlos Sangiorgio Diretor de Vendas para Audiência: Dimas Mietto Diretora de Conteúdo: Alecsandra Zapparoli

Diretor Editorial - Estilo de Vida: Sérgio Gwercman Diretor de Redação: Fabio Volpe Diretor de Arte: Fábio Bosquê Editores: Ana Prado, Fábio Akio Sasaki, Lisandra Matias, Paulo Montoia Repórter: Ana Lourenço Analista de Informações Gerenciais: Simone Chaves de Toledo Analista de Informações Gerenciais Jr.: Maria Fernanda Teperdgian Designers: Dânue Falcão, Vitor Inoue Estagiários: Guilherme Eler, Paula Lepinski, Sophia Kraenkel Atendimento ao Leitor: Sandra Hadich, Walkiria Giorgino CTI Andre Luiz Torres, Marcelo Augusto Tavares, Marisa Tomas PRODUTO DIGITAL Gerente de Negócios Digitais: Marianne Nishihata Gerentes de Produto: Pedro Moreno e Renata Gomes de Aguiar Analistas de Produto: Elaine Cristina dos Santos e Leonam Bernardo Designers: Danilo Braga, Juliana Moreira, Simone Yamamoto Animação: Felipe Thiroux Estagiário: Daniel Ito Desenvolvimento: Anderson Renato Poli, Cah Felix, Denis V. Russo, Eduardo Borges Ferreira, Elton Prado. Estagiário: Vinicius Arruda COLABORARAM NESTA EDIÇÃO Consultoria: Fabio Marson Texto: Thereza Venturoli Ilustração: 45 Jujubas (capa) Revisão: José Vicente Bernardo www.guiadoestudante.com.br VENDAS DE PUBLICIDADE Andrea Veiga (RJ), Alex Stevens (Internacional e Regional), Ana Paula Moreno (Moda, Decoração e Construção), Cristiano Persona (Financeiro), Daniela Serafim (Tecnologia, Telecom, Educação e Saúde), Selma Souto (Bens de Consumo), William Hagopian (Transporte, Mobilidade, Entretenimento e Turismo) ABRIL BRANDED CONTENT Edward Pimenta VENDAS PARA AUDIÊNCIA Adailton Granado (Processos), Cézar Almeida (Circulação Exame/Femininas), Daniela Vada (Atendimento ao Assinante), Ícaro Freitas (Circulação Veja/ Lifestyle), Luci Silva (Marketing Direto, Relacionamento e Retenção), Marco Tulio Arabe (Estúdio de Criação), Mary Veras (Vendas Corporativas), Rodrigo Chinaglia (e-business), Wilson Paschoal (Vendas de Rede) MARKETING Maurício Panflio (Pesquisa de Mercado), Carolina Bertelli (Femininas), Cinthia Obrecht (Estilo de Vida), Diego Macedo (Abril Big Data), Keila Arciprete (Veja), Leander Moreira (Exame) DIGITAL E MOBILE Sandra Carvalho

PUBLICIDADE SÃO PAULO e informações sobre representantes de publicidade no Brasil e no Exterior: www.publiabril.com.br GE MATEMÁTICA ed. 8 2017 (EAN 789-3614-10450 8) é uma publicação da Editora Abril. Edições anteriores: Venda exclusiva em bancas, pelo preço da última edição em banca mais despesa de remessa. Solicite ao seu jornaleiro. Distribuída em todo o país pela Dinap S.A. Distribuidora Nacional de Publicações, São Paulo. A PUBLICAÇÃO não admite publicidade redacional. SERVIÇO AO ASSINANTE: Grande São Paulo: (11) 5087-2112 Demais localidades: 08007752112 www.abrilsac.com PARA ASSINAR: Grande São Paulo: (11) 3347-2145 Demais localidades: 08007752145 www.assineabril.com.br

Remontando a expressão A² + x . A + y . B = C, agora com as matrizes, temos: 1 2 0 x x o y 0 0 1+x+y 2+x 0 0 1 0 + 0 x 0 + 0 y 0 = 0 1+x+y 0 0 0 1 0 0 x 0 0 y 0 0 1+x+y

2+x 1+x+y 0

0 0 1+x+y

Estabelecendo a igualdade entre quaisquer termos das duas matrizes, encontramos o valor de x. Considerando o elemento a12, por exemplo, temos 2+x=0→x=0–2 →x=–2 Resposta: C

146 GE MATEMÁTICA 2017

Conselho Editorial: Victor Civita Neto (Presidente), Thomaz Souto Côrrea (Vice-Presidente), Alecsandra Zapparoli, Eurípedes Alcântara Giancarlo Civita e José Roberto Guzzo

REDAÇÃO E CORRESPONDÊNCIA – Av. das Nações Unidas, 7221, 18º andar, Pinheiros, São Paulo, SP, CEP 05425-902, tel. (11) 3037-2000.

E obtendo y . B: 1 0 0 y 0 0 y.B= 0 1 0 = 0 y 0 0 0 1 0 0 y

1+x+y 0 0

ROBERTO CIVITA (1936-2013)

DEDOC E ABRIL PRESS Valter Sabino PLANEJAMENTO CONTROLE E OPERAÇÕES Adriana Fávilla, Adriana Kazan, Emilene Pires e Renata Antunes RECURSOS HUMANOS Alessandra de Castro (Desenvolvimento Organizacional), Ana Kohl (Serviços de RH) e Márcio Nascimento (Remuneração e Benefícios)

Obtendo x . A: 1 1 0 x x 0 x . A = x. 0 1 0 = 0 x 0 0 0 1 0 0 x

Portanto, 0 0 0 C= 0 0 0 = 0 0 0

Fundada em 1950

VICTOR CIVITA (1907-1990)

Para adquirir os direitos de reprodução de textos e imagens do Guia do Estudante, acesse www.abrilconteudo.com.br. Tels.: (11) 3990-1329 e (11) 3990-2059 E-mails: atendimentoconteudoabril@abril.com.br e abrilcontent@abril.com.br IMPRESSA NA GRÁFICA ABRIL Av. Otaviano Alves de Lima, 4400, CEP 02909-900 – Freguesia do Ó São Paulo - SP

Presidente AbrilPar: Giancarlo Civita Presidente do Grupo Abril: Walter Longo Diretor Corporativo de Marketing: Tiago Afonso Diretor de Auditoria: Thomaz Roberto Scott Diretor Superintendente da Gráfica: Eduardo Costa Diretora Corporativa de Recursos Humanos: Claudia Ribeiro Diretora de Relações Corporativas: Meire Fidelis Diretora Jurídica: Mariana Macia Presidente Executivo da DGB: Claudio Prado www.abril.com.br


Issuu converts static files into: digital portfolios, online yearbooks, online catalogs, digital photo albums and more. Sign up and create your flipbook.